GEOMETRIA 7

70
74 Geometrie. Cap. 1. Recapitulare şi completări GEOMETRIE Capitolul 1 Recapitulare şi completări l Dan ºi Radu se întîlnesc la Piaþa Flacãra în faþa Bãncii de Economii. Reprezen- taþi punctul de întîlnire a celor doi pentru Vasile, care vrea sã se întîlneascã cu ei. Rezolvare. Pentru a se întîlni cu cei doi Vasile trebuie sã ajungã în punctul marcat pe desen. l Punctul este cea mai simplã figurã geometricã. l Orice figurã geometricã este o mulþime de puncte. l Punctele se reprezintã „l“ sau „ד ºi se noteazã: A, B, A 1 („punctul A unu“), A 2 („A doi“), A¢ („A prim“), A² („A secund“) etc. l Distanþa dintre punctele A, B, se noteazã d(A, B) ºi se mãsoarã cu rigla gradatã. Dacã d(A, B) = 0, punctele A ºi B sînt confundate (A = B). Dacã d(A, B) > 0, punctele A ºi B sînt diferite (A ¹ B). Din viaţă Punctul. Reprezentarea punctelor A B A 1 · Dreapta l O vrabie a zburat de pe o cupolã a bisericii Ciuflea pe o cu- polã a catedralei din Chiºinãu. Reprezentaþi pe o hartã direcþia pe care a zburat vrabia. Rezolvare. Dupã ce se marcheazã pe hartã poziþiile celor douã biserici, se deseneazã cu ajutorul unei rigle dreapta ce conþine cele douã puncte. l Dreapta este o figurã geometricã. l Axioma este o propoziþie matematicã pe care o consi- derãm adevãratã. Ea nu trebuie justificatã logic. Axioma dreptei. Existã o singurã dreaptã, ce conþine douã puncte diferite date. l O dreaptã se noteazã cu o literã micã: a, b, a 1 , a¢, („dreapta a prim“), a² („a secund“) sau cu douã litere mari: AB („dreap- ta A-Be“), CC¢, D¢D². l Dreptele care au un punct comun se numesc drepte con- curente. În desen sînt ilustrate dreptele m ºi EF concurente în punctul P. Se noteazã: m Ç EF = {P}. l Punctele ce nu aparþin unei drepte sînt puncte exterioare dreptei. Punctul P aparţine (F nu aparþine) dreptei m ºi se scrie: P Î m (F Ï m). l Trei sau mai multe puncte conþinute de aceeaºi dreaptã se numesc puncte coliniare. l Punctele E, P, F sînt coliniare. Punctul P se aflã între punctele E ºi F. Se noteazã: EPF (ca în desen) sau FPE. Proprietatea punctelor coliniare. Punctul B se aflã între punctele A ºi C (ABC) dacã ºi numai dacã d(A, C) = d(A, B) + d(B, C). Se mai scrie AC = AB + BC. l Dreptele cu douã puncte comune se numesc confundate. De exemplu, EP ºi PF sînt confundate ºi se noteazã: EP = PF. Din viaţă P m E F

Transcript of GEOMETRIA 7

74 Geometrie. Cap. 1. Recapitulare şi completări

G E O M E T R I E Capitolul 1 Recapitulare şi completări

l Dan ºi Radu se întîlnesc la Piaþa Flacãra în faþa Bãncii de Economii. Reprezen-taþi punctul de întîlnire a celor doi pentru Vasile, care vrea sã se întîlneascã cu ei.Rezolvare. Pentru a se întîlni cu cei doi Vasile trebuie sã ajungã în punctul marcat pe desen.

l Punctul este cea mai simplã figurã geometricã.l Orice figurã geometricã este o mulþime de puncte.l Punctele se reprezintã „l“ sau „ד ºi se noteazã: A, B, A

1 („punctul A unu“), A

2

(„A doi“), A¢ („A prim“), A² („A secund“) etc.l Distanþa dintre punctele A, B, se noteazã d(A, B) ºi se mãsoarã cu rigla gradatã. Dacã d(A, B) = 0, punctele A ºi B sînt confundate (A = B). Dacã d(A, B) > 0, punctele A ºi B sînt diferite (A ¹ B).

Din viaţă

¶ Punctul. Reprezentarea punctelor

A B

A1

· Dreaptal O vrabie a zburat de pe o cupolã a bisericii Ciuflea pe o cu-polã a catedralei din Chiºinãu. Reprezentaþi pe o hartã direcþia pe care a zburat vrabia.Rezolvare. Dupã ce se marcheazã pe hartã poziþiile celor douã biserici, se deseneazã cu ajutorul unei rigle dreapta ce conþine cele douã puncte.

l Dreapta este o figurã geometricã.l Axioma este o propoziþie matematicã pe care o consi-derãm adevãratã. Ea nu trebuie justificatã logic.Axioma dreptei. Existã o singurã dreaptã, ce conþine douã puncte diferite date.l O dreaptã se noteazã cu o literã micã: a, b, a

1, a¢, („dreapta

a prim“), a² („a secund“) sau cu douã litere mari: AB („dreap-ta A-Be“), CC¢, D¢D².l Dreptele care au un punct comun se numesc drepte con-curente. În desen sînt ilustrate dreptele m ºi EF concurente în punctul P. Se noteazã: m Ç EF = {P}.l Punctele ce nu aparþin unei drepte sînt puncte exterioare dreptei. Punctul P aparţine (F nu aparþine) dreptei m ºi se scrie: P Î m (F Ï m).l Trei sau mai multe puncte conþinute de aceeaºi dreaptã se numesc puncte coliniare.l Punctele E, P, F sînt coliniare. Punctul P se aflã între punctele E ºi F. Se noteazã: E–P–F (ca în desen) sau F–P–E.Proprietatea punctelor coliniare. Punctul B se aflã între punctele A ºi C (A–B–C) dacã ºi numai dacã d(A, C) = d(A, B) + d(B, C). Se mai scrie AC = AB + BC.l Dreptele cu douã puncte comune se numesc confundate. De exemplu, EP ºi PF sînt confundate ºi se noteazã: EP = PF.

Din viaţă

P

m

E

F

75Geometrie. Cap. 1. Recapitulare şi completări

E x e r c i ţ i i 1. Alegeţi răspunsul. Care este cea mai simplă figură geometrică?

a) Pătratul b) Cubul c) Punctul d) Cercul2. Completaţi ca în model.

A1 A2

A3

A B

CD

M N

PQ

a) Triunghiul are vîrfurile A1, A2, A3.b) Dreptunghiul are vîrfurile ..................c) Pătratul are vîrfurile .................

3. Se ştie că A(–1, 2).Completaţi pentru punctele: B(.......), C(.......), D(.......),E(.......).

4. Reprezentaţi într-un sistem de axe de coordonate punctele L(–2, 3), M(–1, –2), N(3, 1), P(2, 4), Q(3, –3).5. Notaţi distanţa dintre punctele:

a) M şi P; b) T şi S; c) B şi D; d) E şi R.6. Completaţi după măsurare:

d(A, B) = ..... mm; d(A, C) = ..... mm;d(B, C) = ..... mm.

7. Completaţi propoziţia: Există ................. care să conţină două puncte date diferite.

a) trei drepte; b) o singură dreaptă;

c) două drepte; d) patru drepte.8. Completaţi cum se citesc drep-tele ilustrate: ...................................................................................9. Reprezentaţi punctele coliniare:

a) A, B, C astfel încît A–B–C;b) D, F, G astfel încît D–G–F;c) P, M, N astfel încît P–M–N.

10. Completaţi ca în modelul rezolvat.a) a şi b se intersectează în M se scrie a Ç b = {M};b) m şi n se intersectează în P se scrie .....................;c) c1 şi c2 se intersectează în A se scrie .....................11. Completaţi ca în modelul rezolvat.a) P aparţine a se scrie P Î a, iar C nu aparţine a se scrie C Ï a;b) B aparţine d se scrie ....., iar A nu aparţine d se scrie .............;c) D aparţine m se scrie ....., iar T nu aparţine m se scrie .............;d) H aparţine e se scrie ....., iar M nu aparţine e se scrie .............12. Desenaţi şi enumeraţi dreptele ce conţin cîte 2 dintre punctele necoliniare:

a) D; E; F; b) M; G; T; c) A; I; L.13. Notaţi: a) punctele A şi B sînt confundate;

b) dreptele a şi c sînt confundate.14. Se ştie: d(A, B) = 7 cm; d(A, C) = 4 cm;

d(B, C) = 6 cm; d(A, E) = 5 cm;d(B, E) = 2 cm; d(C, E) = 3 cm.

Controlaţi cu ajutorul distanţelor dacă sînt coliniare punctele: A, B, C; A, B, E; B, C, E; A, C, E.

AB

C

D

E

mm1

A B

C

15. Verificaţi cu ajutorul distanţelor dacă:

AEC

D F

a) A–C–E; b) A–D–E; c) A–D–F; d) A–C–F. 16. Aflaţi numărul maxim de dreptediferite, ce conţin cel puţin douădintre punctele din desen.

17. Aflaţi numărul maxim de drepte diferite ce se pot construi prin cel puţin două puncte, cînd sînt date 7 puncte diferite.18. Aflaţi numărul maxim de drepte diferite ce se pot construi prin cel puţin două puncte, cînd sînt date 7 puncte diferite dintre care 3 puncte sînt coliniare.19. Aflaţi numărul maxim de drepte diferite ce se pot construi prin cel puţin două puncte, cînd sînt date 8 puncte diferite dintre care 4 puncte sînt coliniare.

20. Pentru a ajunge din localitatea A în localitatea D se poate alege unul dintre traseele: ABCD; ACD; ABD.Alegeţi traseul cel mai scurt, dacă se ştiu distanţele:

AB = 15 km; BC = 25 km; CD = 14 km; AC = 30 km; BD = 29 km.

21. Aflaţi numărul maxim de drepte diferite ce se pot construi prin cel puţin două puncte:

a) cînd sînt date 20 de puncte diferite;b) cînd sînt date 30 de puncte diferite dintre care 4 puncte sînt coliniare.

AB

C

D E

76 Geometrie. Cap. 1. Recapitulare şi completări

¸ Planull Pe tablã este reprezentatã dreapta d ºi punctele A, B, C. Care dintre cele trei puncte aparþine dreptei d?Rezolvare. Spre deosebire de reprezentarea ei, dreapta d este nemãrginitã. Prelungind reprezentarea dreptei d, se constatã cã numai punctul C aparþine dreptei d.

Din viaţă

l Dreapta este nemãrginitã. Pe o dreaptã poate fi reprezentatã mulþimea numerelor raþionale (ºi nu numai aceste numere). O dreaptã conþine o infinitate de puncte.l În exerciþiu, tabla reprezintã un plan. Planul este o figurã geometricã cu o infinitate de puncte. Un plan se noteazã cu litere mici ale alfabetului grecesc: a (alfa), b (beta), g (gama), a

1, a

2, b¢, b².

Axiomã. Dacã douã puncte diferite aparþin unui plan, atunci dreapta determinatã de eleeste conþinutã de (inclusã în) plan.l În desen: AB este conþinutã de sau inclusă („Ì“) în planul tablei.Geometria plană. Geometria care studiazã figurile geometrice conþinute de un plan se numeºte geometrie planã (GE Pãmînt (în limba greacã), METRON mãsurare (în limba greacã)).

¹ Semidreaptal În fotografie ascensorul se aflã la nivelul 3. Ilustraþi cu ajutorul unei figuri geome-trice coborîrea ascensorului la nivelul 1.Rezolvare. Semidreapta AB are originea A şi ea ilustreazã cum va coborî ascensorul de la nivelul 3 la nivelul 1.

l Fie A, B ºi C puncte ale dreptei d, A–B–C. [BA este semidreapta închisã cu originea B ºi care conþine toate punctele dreptei d situate de aceeaºi parte a punctului B ca ºi A, inclusiv B. În condiþiile

de mai sus, [BA = {P Î d | P–B–C sau P = B}, iar semidreapta deschisã BC este (BC = {M Î d | A–B–M}. Semidreptele BA şi BC se numesc semidrepte opuse.l Pe [AB se poate reprezenta N (mulþimea numerelor naturale), iar pe (AB se poate reprezenta mulþimea N* (mulþimea numerelor naturale nenule).

º Segmentull Maria a desenat casa din dreapta ajutîndu-se de punctele din stînga. Ce figuri geometrice a obţinut unind cîte două dintre aceste puncte? Care este numărul lor minim?Rezolvare. Unind oricare dintre cele două puncte se obţine un seg-ment. Maria a desenat cel puţin 17 segmente.

DT

Ll Segmentul deschis DL conţine toate punctele situate între D şi L. Se notează (DL).l Segmentul închis DL conţine în plus punctele D şi L. Se notează [DL] = (DL) È {D, L}. D şi L sînt capetele sau extremităţile segmentelor DL (închis sau deschis).

l Se mai scrie: (DL) = {M Î DL | D–M–L}, unde DL este dreapta DL. Evident, (DL) = (DL Ç (LD.l În desen D–T–L. T este punct interior segmentului DL.l Lungimea segmentului DL este d(D, L). Se notează DL = d(D, L).l Segmentele congruente sînt segmentele cu aceeaşi lungime. „º“ se citeşte „congruent cu“.l Congruenţa segmentelor are proprietăţile egalităţii numerelor (reflexivitate, simetrie, tranzitivitate).Axioma adunării segmentelor. Dacă A–B–C, atunci AC = AB + BC.

d

AC

B

Din viaţă

A

B

d CB

A

Din viaţă

a

77Geometrie. Cap. 1. Recapitulare şi completări

E x e r c i ţ i i 1. În desen sînt reprezentate plane. Cum se citeşte nu-mele fiecărui plan?

a a¢ a²

2. Completaţi propoziţia matematică alegînd una din-tre variantele propuse.

Dacă punctele A şi B aparţin planului a, atunci dreapta AB ............................ în planul a.a) nu are alte puncte comune cu;b) este conţinută de;c) mai are cel mult încă un punct comun cu;d) mai are cel mult încă două puncte comune cu.

3. Completaţi ca în model. a) M şi P sînt puncte ale planului a

1. Atunci se

scrie MP Ì a1.

b) A şi D sînt puncte ale planului a2. Atunci se

scrie .............c) B şi C sînt puncte ale planului a

3. Atunci se

scrie .............;d) M şi N sînt puncte ale planului a

4. Atunci se

scrie .............4. Fie D–L–F. Completaţi:

a) semidreapta deschisă DL se notează .....;b) semidreapta închisă DL se notează ......;c) semidreapta închisă LD se notează ......;d) semidreapta deschisă LD se notează ......;

5. Fie dreapta a şi punctele A, B, C, D, E (v. dese-nul).

A EC D FBa

Completaţi ca în model.a) [BA şi [BC sînt semidreptele opuse închise cu originea B.b) .... şi ..... sînt semidreptele opuse închise cu originea C.c) .... şi ..... sînt semidreptele opuse închise cu originea D.d) .... şi ..... sînt semidreptele opuse închise cu originea E.

6. Notaţi segmentul:a) deschis AB; b) închis BC; c) deschis AC;c) închis DE; c) deschis EF; d) închis MN.

7. Completaţi ca în model (v. desenul ex. 5).a) [AB] = [AB ∩ [BA.b) [AC] = ................ c) [BC] = ................d) [CD] = ................ e) [BD] = ................f) [DE] = ................ g) [CE] = ................

8. Completaţi cu „Γ sau „Ï“ (v. desenul ex. 5).a) C ... [BA; b) C ... [BD; c) D ... [BC;d) d ... [CA; e) F ... [DC; f) E ... [FD.

9. În desen segmentele congruente sînt marcate la fel. Completaţi ca în model.

a) [AS] ≡ [DI];b) ....... ≡ .......;c) ....... ≡ .......;d) ....... ≡ ........

10. Punctele A, B, C,D, E, F sînt coliniare caîn desenul ex. 5. AB = 5, AC = 12, CE = 19, DE = 7, EF = 8. Aflaţi: BC, CD, AF.

11. Enumeraţi semidreptele închise şi segmentele în- chise pe care le descoperiţi în desen.

12. Fie A–B–C. Lungimile segmentelor AB şi BC sînt direct proporţionale cu numerele 3 şi 5, iar AC = 24 cm. Aflaţi lungimile segmentelor AB şi BC.13. Aflaţi numărul maxim de segmente determinate de cîte două dintre 10 puncte.14. Aflaţi numărul maximde semidrepte diferite determinate de puncteledin desen.

15. Fie segmentul AB. Marcaţi cu M1 un punct al seg-mentului şi aflaţi numărul segmentelor; marcaţi cu M2 alt punct al segmentului şi calculaţi cîte segmente sînt în acest moment. Se continuă pînă se adaugă punctul M5. Cîte segmente sînt acum?

16. Examinaţi figurile geometrice!Cîte segmente distingeţiîn fiecare situaţie?Adăugaţi încă o figură şinumăraţi segmentele.

A

C G

M

E

N

KL D

I

J FR

S

B

T

A

EC DB A

HC

D E

B

F

G

a) b) c)

78 Geometrie. Cap. 2. Unghiul

Capitolul 2 Unghiul ¶ Unghiul. Definiţie. Elemente. Măsurarea unghiurilorl Anton trebuie să ajungă din B în C cu ajutorul unei hărţi, o busolă şi semnele pe care le găseşte în pădure. Cum va proceda?Rezolvare. Utilizînd harta şi busola, Anton măsoară unghiul for-mat de BA cu direcţia Sud-Nord (SN), şi unghiul format de AB cu SN. În acest fel el va putea alege corect calea de la B spre A şi cea de la A spre C.

Din viaţă

N

S

A

B

C

l La ce se foloseşte firul cu plumb? Ce puteţi spune despre turnul din Pisa (din fotografie)?Rezolvare. Firul cu plumb este folosit de constructori pentru a asigura verticalitatea zidurilor. Tur-nul din Pisa este înclinat, deoarece axa lui formează cu orizontala un unghi ascuţit.

l Unghiul este figura geometrică formată de două semidrepte cu originea comună. Unghiul este reuniunea celor două semidrepte cu aceeaşi origine.l Unghiul DBC notat „ÐDBC“ sau unghiul B notat „ÐB“ are laturile [BC, [BD şi vîrful B. l Unghiul alungit are laturile semidrepte opuse, iar unghiul nul are laturile egale (sau confundate). Unghiul alungit şi unghiul nul sînt unghiuri impro-prii. Celelalte unghiuri sînt unghiuri proprii.l Unghiurile se măsoară cu raportorul în grade (se notează „°“), minute (1° = 60¢) şi secunde (1¢ = 60²). Unghiurile alungite au măsura de 180°, iar cele nule au 0°. Măsurînd cu raportorul, mÐDBC (măsura unghiului DBC este) = 27°.l Unghiurile cu aceeaşi măsură se numesc unghiuri congruente. Dacă unghiurile A şi D sînt congruente, atunci se scrie: ÐA ≡ ÐD.l Unghiurile de 90° se numesc unghiuri drepte şi ele se identifică sau se construiesc cu ajutorul echerului. Laturile unghiului drept sînt per-pendiculare

· Clasificarea unghiurilor

Vîrful un-ghiului B

B

D

CLaturile unghiului B

BD

C

Raportor

Echer

l Unghiul A este mai mic decît unghiul B şi se scrie „ÐA < ÐB“ dacă mÐA < mÐB.l Unghiurile mai mici decît un unghi drept se numesc unghiuri ascuţite.l Unghiurile mai mari decît un unghi drept se numesc unghiuri obtuze.

$�

q

¸ Semiplan. Interiorul, exteriorul unui unghil În desen sînt reprezentate: un rîu, o casă, o biserică, un castel. Care dintre cele trei construcţii se află de aceeaşi parte a rîului? Care dintre ele nu se află de aceeaşi parte a rîului?Rezolvare. Casa şi biserica se află de aceeaşi parte a rîului. Biserica şi castelul nu se află de aceeaşi parte a rîului, casa şi castelul se află de aceeaşi parte a rîului.

Din viaţă

l Fie dreapta d conţinută de planul a. Punctele planului ce sînt de aceeaşi parte a dreptei d sînt conţinute de acelaşi semiplan deschis cu frontiera d.l Unghiul CBD determină trei mulţimi: unghiul CBD (reuniunea semidreptelor închise BC şi BD), interiorul unghiului CBD (IntÐCBD), exteriorul unghiului CBD (ExtÐCBD). IntÐCBD este intersecţia a două semiplane deschise.Axioma de construcţie a unghiului. Fie semidreapta h conţinută de dreapta k. Există un singur unghi congruent cu un unghi dat, cu o latură h şi cealaltă latură conţinută de unul dintre semiplanele cu frontiera dreapta k.

da

BF

G

B

DIntÐCBD

ExtÐCBD

Din viaţă

79Geometrie. Cap. 2. Unghiul

E x e r c i ţ i i 1. Completaţi propoziţia:Unghiul este reuniunea ............................................... cu aceeaşi origine.

a) a două segmente b) a două semidreptec) a două drepte d) a două cercuri.

2. Notarea unghiurilor. Completaţi ca în model.

AB

CD

E

F

GI J

K

LM

a) Unghiul EAF notat „ÐEAF“ sau unghiul A notat „ÐA“.b) Unghiul G......... notat ......................................c) Unghiul K......... notat ........................................d) Unghiul D........ notat .........................................

3. Completaţi propoziţiile ca în model (v. desenul ex. 2).a) A este vîrful, iar [AE şi [AF sînt laturile unghiu-lui EAF.b) B ........................................................................c) C ........................................................................d) D ........................................................................

4. Completaţi propoziţia:Unghiul alungit are laturile ........................................ 5. Completaţi ca în model.

A O B

a) O este vîrful, iar [OA şi [OB sînt laturile unghiu-lui alungit AOB.

C I D

b) ............................................................................E J F

c) ............................................................................M K N

d) ............................................................................6. Completaţi propoziţiile:

a) Unghiul alungit are măsura ................. b) Unghiul nul are laturile ........................... şi mă-sura lui este ......c) Unghiurile nule şi alungite se numesc ..............,

iar celelalte unghiuri se numesc ................d) Unghiurile cu aceeaşi măsură se numesc .........e) Unghiurile A şi B sînt congruente se notează .....................f) Pentru măsurarea unghiurilor se foloseşte ........g) Un unghi ascuţit are .......................................h) Un unghi obtuz are .......................................i) Cu echerul se pot construi ................................j) Laturile unghiului drept sînt ..............................

7. Măsuraţi unghiurile ilustrate şi completaţi relaţiile.

AA B C Da) mÐA = ...... b) mÐB = ...... c) mÐC = ...... d) mÐD = ......

8. Examinaţi desenul şi remarcaţi unghiurile congru-ente (marcate la fel), apoi completaţi relaţiile.

A

B CD E

F

a) ÐA ≡ ......; b) ÐB ≡ ......; c) ÐC ≡ ......9. Construiţi:

a) unghiul A, de 61°; b) unghiul B, de 42°;c) unghiul C, de 39°; d) unghiul B, de 35°.

10. Construiţi:a) unghiul M, de 99°; b) unghiul N, de 111°;c) unghiul P, de 124°; d) unghiul Q, de 132°.

11. Completaţi desenul ca în modelul rezolvat.

a)

AA

ExtÐA

IntÐA

ExtÐA

b)

B

c)

C

c)

D

12. Examinaţi desenulşi enumeraţi toate un-ghiurile proprii ce le identificaţi.

13. Fie M1 Î Int ÐAOB, M2 Î Int ÐAOM1, M3 Î Int ÐAOM2, .... Cîte unghiuri se obţin după ce aţi construit semidreptele OM1, OM2, OM3, OM4, OM5, OM6?

O

AB

O

AB

M1

O

AB

M1M2

A

B

C

D

E

80 Geometrie. Cap. 2. Unghiul

¹ Unghiuri adiacente N

S

A

B

Cl În desen este ilustrată schematic o hartă. Se remarcă localităţile A, B, C şi mai multe drumuri. Cum se poate afla măsura unghiului format de direcţiile BA şi BC cu ajutorul unei busole?Rezolvare. Se măsoară unghiul format de direcţia BA cu direcţia SN şi un-ghiul format de BC cu direcţia SN. Măsura unghiului format de direcţiile BA şi BC este egal cu suma măsurilor unghiurilor formate de SN cu BA şi BC.

l Două unghiuri se numesc adiacente dacă au o latură comună (semidreaptă deschisă) conţinută de interi-orul unghiului format de laturile necomune sau, laturile necomune sînt semidrepte opuse. l Unghiurile adiacente EFG şi GFH se numesc adiacente comple- mentare dacă unghiul EFH este unghi drept. (Remarcaţi cum am ilustrat că unghiul EFH este drept!) l Unghiurile MKL şi LKN se numesc adiacente suplemen- tare dacă semidreptele KM şi KN sînt opuse.Axioma adunării unghiurilor. Dacă C Î IntÐABD, atunci m(ÐABD) = m(ÐABC) + m(ÐCBD).l O teoremă este o propoziţie al cărui adevăr trebuie justificat logic. O teoremă poate fi formulată sub forma: „Dacă IPOTEZA, atunci CONCLUZIA.“l Demonstraţia unei teoreme constă în justificarea logică a teoremei.Teorema unghiurilor adiacente. Dacă unghiurile ABC şi CBD sînt adiacente, atunci m(ÐABD) = m(ÐABC) + m(ÐCBD).Obsevaţii. 1) Ipoteza teoremei este: „unghiurile ABC şi CBD sînt adiacente.“ 2) Concluzia teoremei este: „m(ÐABD) = m(ÐABC) + m(ÐCBD).“ 3) Demonstraţia teoremei unghiurilor adiacente.

Afirmaţii Justificări1) C Î IntÐABD. Ipoteza.

2) m(ÐABD) = m(ÐABC) + m(ÐCBD), q.e.d. (c.c.t.d.). Axioma adunării unghiurilor.l Corolarul este o teoremă care este o consecinţă a altei teoreme.Teorema unghiurilor adiacente are consecinţele:Corolarul 1. Dacă unghiurile EFG şi GFH sînt adiacente complementare, atuncim(ÐEFG) + m(ÐGFH) = 90°.Corolarul 2. Dacă unghiurile MKL şi LKN sînt adiacente suplementare, atunci m(ÐMKL) + m(ÐLKN) = 180°.l Bisectoarea unghiului este semidreapta cu originea în vîrful unghiului, conţinu-tă de interiorul unghiului şi care formează cu laturile unghiului unghiuri congruente.Teorema bisectoarelor unghiurilor adiacente suplementare. Dacă unghiurile ABCşi CBD sînt adiacente suplementare, atunci bisectoarele lor sînt perpendiculare.l Reciproca unei teoreme este o teoremă a cărei ipoteză este concluzia teoremei, iar con-cluzia - ipoteza teoremei. Dacă ipoteza conţine mai multe condiţii, atunci lucrurile sînt mai complicate.Reciproca teoremei bisectoarelor unghiurilor adiacente suplementare. Dacă bisectoarele unghiurilor adiacente AID şi CIB sînt perpendiculare, atunci unghiurile AID şi CIB sînt adiacente suplementare.l Dacă o teoremă este reciproca unei teoreme, atunci aceasta din urmă se mai numeşte teorema directă.l O teoremă de echivalenţă conţine atît teorema directă cît şi teorema reciprocă. În exemplul de mai sus, teorema de echivalenţă esteTeorema bisectoarelor unghiurilor adiacente suplementare. Unghiurile ABC şi CBD sînt adiacente su-plementare dacă şi numai dacă bisectoarele lor sînt perpendiculare.l Demonstraţia unei teoreme de echivalenţă este formată din demonstraţiile celor două teoreme.

Din viaţă

A

B

C

D

K

L

M N

A

O

C

B

E

F

G

H

I

AB

C

D

Ex x yy

81Geometrie. Cap. 2. Unghiul

E x e r c i ţ i i 1. Completaţi propoziţia:Două unghiuri sînt adiacente dacă o latură comună (semidreaptă deschisă) conţinută de ..................... sau, laturile necomune sînt .........................................2. Descrieţi unghiurile adiacente ca în model.

AB

CD

E

FG

I J

K

LMN

PR T

a) Unghiurile adiacente EAN şi FAN au [AN latură comună şi (AN Ì Int ÐEAF.b) Unghiurile adiacente GBP şi PBI au .............................................c) Unghiurile adiacente JCR şi RCK au .............................................d) Unghiurile adiacente LDT şi TDM au .............................................

3. Completaţi propoziţiile:a) Două unghiuri adiacente cu laturile necomune semidrepte opuse se numesc .................................b) Două unghiuri adiacente cu laturile necomune perpendiculare se numesc .................................

4. Examinaţi desenul şi completaţi fiecare caz.

D

A B

C

E

O

G

F

I

L K

JP

N MR

Unghiuri adiacente: .........................Unghiuri adiacente complementare: ......................

Unghiuri adiacente suplementare: .......................Nu sînt unghiuri adiacente: ..............................

5. Unghiurile BAD şi DAC sînt adiacente. Aflaţi:

D

A

B

C

a) mÐBAC, dacă mÐBAD = 38° şi mÐDAC = 56°;b) mÐBAC, dacă mÐBAD = 25° şi mÐDAC = 15°;c) mÐBAC, dacă mÐBAD = 92° şi mÐDAC = 75°.

6. Unghiurile BAD şi DAC sînt adiacente. Aflaţi:a) mÐBAD, dacă mÐBAC = 93° şi mÐDAC = 34°;b) mÐBAD, dacă mÐBAC = 85° şi mÐDAC = 27°;c) mÐBAD, dacă mÐBAC = 97° şi mÐDAC = 56°.

7. Unghiurile MOP şi PON sînt adiacente suplemen-tare. Aflaţi:

M O N

P

a) mÐMOP, dacă mÐPON = 108°;b) mÐPON, dacă mÐMOP = 63°; c) mÐMOP, dacă mÐPON = 77°.

8. Completaţi propoziţia:Bisectoarea unui unghi este semidreapta cu originea în vîrfului unghiului, inclusă în interiorul unghiului şi care .........................................................................9. Construiţi bisectoarea:

a) unghiului A, de 60°; b) unghiului B, de 44°;c) unghiului C, de 58°; d) unghiului D, de 72°.

13. Fie teorema: „Unghiurile ABC şi CBD sunt adia-cente complementare dacă şi numai dacă bisectoarele lor formează un unghi de 45°“.Completaţi teorema directă şi teorema reciprocă. 14. a) Formulaţi teorema bisectoa-relor unghiurilor adiacente suple-mentare. b) Completaţi ipoteza şi concluzia teo-

remei descriind desenul.Ipoteza. Unghiurile ........... şi ........... sînt adiacente ...................; [ID este bisectoarea .........................;[ID este bisectoarea .........................Concluzia. mÐ......... = 90°. c) Completaţi demonstraţia.1) mÐAIB + ......... = 180° (Ipoteza).2) mÐAID = ......... = x (Ipoteza). ...............................

10. Fie teorema: „Dacă două unghiuri sînt adiacente complementare, atunci suma măsurilor lor este 90°“.Ipoteza teoremei este: „două unghiuri sînt adiacente complementare“.Concluzia teoremei este: „suma măsurilor lor este 90°“.Formulaţi ipoteza şi concluzia teoremei: „Dacă două

unghiuri sînt adiacente complementare şi unul dintre ele are 30°, atunci celălalt unghi are 60°“.11. Formulaţi ipoteza şi concluzia teoremei: „Dacă un unghi are 64°, atunci bisectoarea formează cu la-turile lui unghiuri de 32°“.12. Formulaţi şi rezolvaţi un exerciţiu asemănător exerciţiului 11.

I

AB

C

D

Ex x yy

82 Geometrie. Cap. 2. Unghiul

º Unghiuri opuse la vîrfl În desen este ilustrată intersecţia a două străzi. Se ştie măsura unuia dintre unghiuri. Cum se pot afla măsurile celorlalte unghiuri?Rezolvare. Se aplică proprietatea unghiurilor adiacente suplementare de trei ori şi se află măsurile celor trei unghiuri.

l Unghiurile ABC şi DEF se numesc suplementare dacă suma măsurilor lor este egală cu 180°.l Dacă două unghiuri sînt suplementare, atunci fiecare dintre ele este suplementul celuilalt.Teorema unghiurilor suplementare. Două unghiuri sînt congruente dacă şi numai dacă au acelaşi suple-ment.l Două unghiuri cu laturile respectiv semidrepte opuse se numesc unghiuri opuse la vîrf. De exemplu, dreptele AC şi BD se intersectează în punctul Oformînd două perechi de unghiuri opuse la vîrf: ÐAOB şi ÐCOD; ÐAOD şiÐBOC.Teorema unghiurilor opuse la vîrf. Fie A–O–C, punctele B şi D situate de o parte şi de alta a dreptei AC. Unghiurile AOB şi COD sînt opuse la vîrf dacă şi numai dacă ÐAOB ≡ ÐCOD.a) Teorema directă: Dacă ÐAOB şi ÐCOD sînt unghiuri opuse la vîrf, atunci ÐAOB ≡ ÐCOD.

Afirmaţii Justificări1) Unghiurile AOB şi AOD sînt adiacente suplementare. Ipoteza.2) Unghiurile AOD şi DOC sînt adiacente suplementare. Ipoteza.3) Unghiurile AOB şi COD au acelaşi suplement. 1) şi 2).4) ÐAOB ≡ ÐCOD, q.e.d. Teorema unghiurilor suplementare.

b) Teorema reciprocă: Dacă A–O–C, punctele B şi D situate de o parte şi de alta a dreptei AC şi ÐAOB ≡ ÐCOD, atunci ÐAOB şi ÐCOD sînt unghiuri opuse la vîrf.

Afirmaţii Justificări1) m(ÐAOB) = m(ÐCOD) = x. Ipoteza.2) Unghiurile AOD şi DOC sînt adiacente suplementare. Ipoteza.3) m(ÐAOD) = 180° – x. Teorema unghiurilor adiacente suplementare.4) B–O–D, q.e.d. Măsura unghiului alungit.

l Două drepte sînt perpendiculare, dacă se intersectează astfel, încît unul dintre unghiu-rile formate de ele este drept. Dreptele a şi b sînt perpendiculare se notează: „a ^ b“. l Mijlocul unui segment este punctul segmentului egal depăr- tat de capetele sale. l Mediatoarea segmentului AB cu mijlocul P este dreapta ce conţine punctul P şi este perpendiculară pe dreapta AB. l Mediatoarea unui segment se poate construi prin plierea unei foi de hîrtie transparentă.

l Bisectoarea unui unghi se poate construi prin plierea uneifoi de hîrtie transparente.l Unghiurile ascuţite ABC şi DEF se numesc complementare dacă suma măsurilor lor este egală cu 90°.l Dacă două unghiuri sînt complementare, atunci fiecare dintre ele este complementul celuilalt.Teorema unghiurilor complementare. Două unghiuri ascuţite sînt congruente dacă şi numai dacă au acelaşi complement.

A

O

B

D

C

a

b

A BP

m

Din viaţă

Construcţii

83Geometrie. Cap. 2. Unghiul

E x e r c i ţ i i 1. Completaţi propoziţiile:

a) Două unghiuri sînt suplementare dacă suma măsurilor lor este .....................b) Un unghi este suplementul altui unghi dacă cele două unghiuri sînt ........................c) Două unghiuri sînt ............................ dacă şi numai dacă au acelaşi suplement.

2. Unghiurile A şi B sînt suplementare. Aflaţi:

BAC

D

E

F

x180° – x

a) mÐB, dacă mÐA = 46°;b) mÐB, dacă mÐA = 89°;c) mÐB, dacă mÐA = 71°;d) mÐB, dacă mÐA = 67°.

3. Aflaţi măsura suplementului unui unghi de:a) 28°; b) 37°; c) 135°; d) 109°.

4. Completaţi propoziţiile:a) Două drepte se intersectează formînd două perechi de unghiuri .................................b) Unghiurile opuse la vîrf sînt .............................

5. Examinaţi desenul şi identificaţi unghiurile opuse la vîrf.

D

AB

CP EF

G HI NK

L MJ

a) b) c)6. Enumeraţi perechile de unghiuri congruente ilus-trate la exerciţiul 5.7. Aflaţi măsurile celorlalte unghiuri dacă:

S T

V UO

a) mÐVOU = 69°; b) mÐVOU = 31°;c) mÐVOU = 96°; d) mÐVOU = 127°.

8. Completaţi propoziţiile:a) Două drepte se numesc perpendiculare, dacă se intersectează formînd .................................b) Dacă dreptele d şi m sînt perpendiculare se scrie ............c) Mijlocul unui segment este punctul segmentului egal ......................................................d) Mediatoarea unui segment este perpendiculara

pe dreapta ce conţine segmentul ....................................

9. Construiţi mediatoatele segmentelor:

A B C D

E F

10. Completaţi propoziţiile:a) Două unghiuri ascuţite se numesc complemen-tare, dacă suma măsurilor lor este ........................b) Un unghi este complementul altui unghi, dacă cele două unghiuri sînt .........................c) Două unghiuri ascuţite sînt ............................ dacă şi numai dacă au acelaşi complement.

11. Unghiurile A şi B sînt complementare. Aflaţi:

AC

D

xB E

F

90° – x

a) mÐB, dacă mÐA = 25°;b) mÐB, dacă mÐA = 48°;c) mÐB, dacă mÐA = 76°;d) mÐB, dacă mÐA = 52°.

12. Aflaţi măsura complementului unui unghi de:a) 34°; b) 43°; c) 22°; d) 73°.

13. Construiţi, aplicînd metoda plierii hîrtiei, media-toarea unui segment de: 5 cm, 11 cm, 13 cm, 17 cm. 14. Construiţi, aplicînd metoda plierii hîrtiei, bisec-toarea unui unghi de: 25°, 37°, 71°, 97°.15. Examinaţi desenul! Cu ajutorul unuia dintre un-ghiurile ilustrate, construiţi:

27°

A

49° B 53° C

35°

D

a) un unghi de 63°;Cu ajutorul echerului se construieşte complemen-tul unghiului A.b) un unghi de 41°;c) un unghi de 37°;d) un unghi de 55°.

84 Geometrie. Cap. 2. Unghiul

16. Cu ajutorul unuia dintre unghiurile ilustrate (v. ex. 15), construiţi cu echerul şi rigla un unghi de 143°.17. Examinaţi desenul şi remarcaţi unghiurile drepte!

39°

AO

B

CD

E

Aflaţi măsurile unghiurilor: AOB, COD, AOE.18. Construiţi două unghiuri cu acelaşi comple-ment după ce descoperiţi astfel de unghiuri în figura geometrică de mai sus.19. Unghiurile AOB şi BOC sînt adiacente, m(ÐAOB) = 150°. Care este măsura maximă ce o poate avea un-ghiul BOC?

OA

B

150°

20. Unghiurile AOB şi BOC sînt adiacente suplemen-tare, măsura primului fiind de 3 ori mai mare decît a celuilalt. Aflaţi:

a) măsurile celor două unghiuri;b) măsura unghiului format de bisectoarele lor;

c) măsurile unghiurilor formate de bisectoare cu laturile unghiurilor.

21. Unghiurile AOB şi BOC sînt adiacente comple-mentare, măsura unuia fiind de 4 ori mai mare decît a celuilalt. Aflaţi:

a) măsurile celor două unghiuri;b) măsura unghiului format de bisectoarele lor;c) măsurile unghiurilor formate de bisectoare cu laturile unghiurilor.

22. Două unghiuri sînt suplementare. Aflaţi măsurile lor dacă măsura unuia este de 5 ori mai mare decît a celuilalt.23. Două unghiuri sînt complementare. Aflaţi măsurile lor dacă măsura unuia este de 8 ori mai mare decît a celuilalt.24. Teorema unghiurilor cu acelaşi suplement:Două unghiuri sînt congruente dacă şi numai dacă au acelaşi suplement.Teorema directă. Dacă două unghiuri sînt congruente, atunci ele au acelaşi suplement.Formulaţi teorema reciprocă.25. Teorema unghiurilor cu acelaşi complement:Două unghiuri ascuţite sînt congruente dacă şi numai dacă au acelaşi complement.Teorema directă. Dacă două unghiuri ascuţite sînt congruente, atunci ele au acelaşi complement.Formulaţi teorema reciprocă.

26. a) Unghiurile AOB şi BOC sînt adiacente comple-mentare. Aflaţi măsurile celor două unghiuri, dacă ele sînt direct proporţionale cu 2 şi 3.

b) Două unghiuri sînt complementare. Aflaţi mă-surile celor două unghiuri, dacă ele sînt direct proporţionale cu 2 şi 7.27. a) Unghiurile AOB şi BOC sînt adiacente suple-mentare. Aflaţi măsurile celor două unghiuri, dacă ele sînt direct proporţionale cu 3 şi 7. b) Două unghiuri sînt suplementare. Aflaţi măsurile celor două unghiuri, dacă ele sînt direct proporţionale cu 7 şi 8.28. a) Două unghiuri sînt adiacente suplementare. Aflaţi măsurile celor două unghiuri, dacă ele sînt in-vers proporţionale cu 3 şi 7. b) Două unghiuri sînt complementare. Aflaţi mă-surile celor două unghiuri, dacă ele sînt invers proporţionale cu 7 şi 8.29. Formulaţi şi demonstraţi aplicînd metoda „Afirmaţii-Justificări“ teorema bisectoarelor unghiu-rilor adiacente complementare.

30. Formulaţi şi demonstraţi aplicînd metoda „Afir-maţii-Justificări“:

a) teorema unghiurilor cu acelaşi complement;b) teorema unghiurilor cu acelaşi suplement.

Măsurarea unghiurilor în construcţii

Alte instrumente pentru măsurareaşi construirea unghiurilor

85Geometrie. Cap. 2. Unghiul

E VA L U A R EI I I

1. Examinaţi desenul!

a) În desen este ilustrat unghiul ADC. Aşezaţi co-rect literele.b) Enumeraţi elementele unghiului ADC.

2. Măsuraţi şi scrieţi care este măsura unghiului L

L3. Identificaţi unghiurile adiacente:

D A

B

C E

MK

FP

NL R

4. Construiţi bisectoarea unghiului

G

J

I

5. Examinaţi desenul şi enumeraţi perechile de un-ghiuri opuse la vîrf.

B

A

C

D

I

6. Aflaţi:a) suplementul unui unghi de 127°;b) complementul unui unghi de 46°.

7. Aflaţi măsurile a două unghiuri adiacente comple-mentare, dacă măsura unuia este cu 26° mai mare decît măsura celuilalt.8. Aflaţi măsurile a două unghiuri suplementare, dacă măsurile lor sînt direct proporţionale cu 7 şi 13.9. Aflaţi măsurile a două unghiuri complementare, dacă măsurile lor sînt invers proporţionale cu 17 şi 13.10. Cîte perechi de unghiuri

A

B

C

D EFadiacente complementare des-

coperiţi în desen?

1. Examinaţi desenul!

a) În desen este ilustrat unghiul BGL. Aşezaţi co-rect literele.b) Enumeraţi elementele unghiului BGL.

2. Măsuraţi şi scrieţi care este măsura unghiului M

M3. Identificaţi unghiurile adiacente:

E B

F

D A

IL

CT

OS P

4. Construiţi bisectoarea unghiului

L

K

M

5. Examinaţi desenul şi enumeraţi perechile de un-ghiuri opuse la vîrf.

M

Q

N

P

O

6. Aflaţi:a) suplementul unui unghi de 135°;b) complementul unui unghi de 54°.

7. Aflaţi măsurile a două unghiuri adiacente comple-mentare, dacă măsura unuia este cu 18° mai mare decît măsura celuilalt.8. Aflaţi măsurile a două unghiuri suplementare, dacă măsurile lor sînt direct proporţionale cu 19 şi 11.9. Aflaţi măsurile a două unghiuri complementare, dacă măsurile lor sînt invers proporţionale cu 23 şi 7.10. Cîte perechi de unghiuri

O

P

U

R STadiacente complementare des-

coperiţi în desen?

1

1

1

1

1

1

1

1

1

1

86 Cap. 3. Cercul

Capitolul 3 Cercul

Din viaţă

¶ Cercul. Definiţie. Elemente

· Poziţiile unei drepte faţă de un cerc

l Cercul de centru O şi rază R este mulţimea punctelor planului aflate la distanţa R de centru O. Notaţie: C(O, R) este cercul de centru O şi rază R.l Cercul se construieşte cu ajutorul compasului.l Elementele unui cerc sînt: centrul cercului, raza cercului.l Interiorul C(O, R) este Int C(O, R) = {M Î planului | OM < R}.l Discul circular de centru O şi rază R, D(O, R) este D(O, R) = {M apar-ţine planului | OM £ R}.l Exteriorul C(O, R) este Ext C(O, R) = {M Î planului | OM > R}.l Coardă a unui cerc este un segment determinat de două puncteale cercului.l Diametru al unui cerc este o coardă ce conţine centrul cercului.Diametrele cercului de rază R au lungimea 2R. Capetele unui di-ametru sînt puncte diametral opuse.l Arc mic al unui cerc este intersecţia cercului cu inte-riorul unui unghi cu vîrful în centrul cercului.l Arc mare al unui cerc este intersecţia cercului cu exterio-rul unui unghi cu vîrful în centrul cercului.l Un arc al unui cerc cu capetele puncte diametral opuseeste un semicerc.

l Fie C(O, R) conţinut de planul a. Punctele planului aflate la distanţa R de Oaparţin cercului; cele aflate la distanţă mai mică decît R aparţin IntC(O, R);cele aflate la distanţă mai mare decît R aparţin ExtC(O, R).

l Fie dreapta d şi M Ï d. P Î d este proiecţia ortogo- nală a punctului M pe dreapta d şi se scrie P = drd M, dacă MP ^ d.

l Fie C(O, R) şi dreapta d. Dreapta d este: a) exterioară cercului, dacă nu are puncte comune cu cercul; b) tangentă cercului, dacă are un punct comun şi numai unul cu cercul; c) secantă, dacă are două puncte comunte cu cercul.

Teorema dreptei exterioare cercului. Dreapta d este exterioară C(O, R) dacăşi numai dacă distanţa de la O la d este mai mare decît R, adică d(O, d) > R.Teorema dreptei tangente cercului. Dreapta d este tangentă C(O, R) dacă şinumai dacă distanţa de la O la d este egală cu R, adică d(O, d) = R.Teorema dreptei secante cercului. Dreapta d este secantă C(O, R) dacă şi numai dacă distanţa de la O la d este mai mică decît R, adică d(O, d) < R.

l În montaj sînt prezentate cîteva situaţii din viaţă. Ce figură geometrică plană recunoaşteţi în ele?Rezolvare. Niciuna dintre construcţii nu poate fi realizată fără cerc.

Compas

ExtC(O, R)

RO

I n t C ( O , R )

O

A r c m i c

A r c m a r e

Unghiul O

Diametru

Puncte diame-tral opuse

Coardă

Semicerc

RO

A

B

C

P

M

d

O

d

d(O, d)123123R

d123

123 Rd(O, d)

d

123 R

123d(O, d)

dreaptă exterioară

dreaptă tangentă

dreaptă secantă

87Cap. 3. Cercul

O

A r c m i c

A r c m a r e

Unghiul O

Diametru

Puncte diame-tral opuse

Coardă

Semicerc

E x e r c i ţ i i 1. Completaţi propoziţiile:

a) Cercul de centru A şi rază r este mulţimea punc-telor planului .....................b) Cercul de centru B şi rază d se notează ............ c) Cercul de centru C şi rază 6 cm se notează ............

2. Interiorul cercului. Completaţi propoziţiile:a) Int C(I, a) = ................................................;b) Int C(A, d) = ................................................;c) Int C(C, r) = ................................................;d) Int C(B, m) = ................................................

3. Discul circular. Completaţi propoziţiile:a) D(A, m) = ................................................;b) D(B, n) = ................................................;c) D(C, r) = ................................................;d) D(F, p) = ................................................

4. Exteriorul cercului. Completaţi propoziţiile:a) Ext C(I, a) = ................................................;b) Ext C(A, d) = ................................................;c) Ext C(C, r) = ................................................;d) Ext C(B, m) = ................................................

5. Construiţi:a) C(A, 3 cm); b) C(B, 5 cm); c) C(C, 4,3 cm);d) C(D, 2,5 cm); e) C(E, 3,7 cm).

6. Construiţi C(O, 48 mm) şi coarda:a) AB de 5 cm; b) AC de 6 cm; c) BD de 64 mm;d) DE de 7 cm; e) DF de 3 cm; f) DM de 96 mm.

7. Construiţi C(O, 65 mm) şi coarda:a) AB de 42 mm, arcul mic şi arcul mare AB;b) CD de 34 mm, arcul mic şi arcul mare CD;c) EF de 51 mm, arcul mic şi arcul mare EF;d) MN de 37 mm, arcul mic şi arcul mare MN;e) KL de 67 mm, arcul mic şi arcul mare KL.

8. Construiţi C(O, 44 mm) şi punctele diametral opuse:

a) A, B; b) C, D; c) E, F; d) K, L; e) M, N.9. Examinaţi figura geometrică şi descrieţi poziţia fie-cărui punct faţă de cerc.

AO

B

C

D

E

10. Notează:a) proiecţia punctului A pe dreapta d;b) proiecţia punctului B pe dreapta m;c) proiecţia punctului C pe dreapta n;d) proiecţia punctului D pe dreapta p;e) proiecţia punctului E pe dreapta a.

11. Completaţi desenul cu proiecţiile punctelor A,B, C, D, E, F pe dreapta d.

A

B

C

D

E

F

d

12. Notaţi:a) dreapta AB este perpendiculară pe dreapta d;b) dreapta CD este perpendiculară pe dreapta m;c) dreapta EF este perpendiculară pe dreapta n;d) dreapta KL este perpendiculară pe dreapta p;e) dreapta MN este perpendiculară pe dreapta e.

13. Completaţi propoziţiile:a) O dreaptă este exterioară unui cerc, dacă ea .....................b) O dreaptă este tangentă unui cerc, dacă ea .....................c) O dreaptă este secantă unui cerc, dacă ea .....................

14. Examinaţi figura geometrică şi recunoaşteţi po-ziţiile dreptelor faţă de cerc.

O

a c d

15. Fie C(I, 8 cm), d(I, a) este distanţa de la dreapta a la centrul cercului I. Recunoaşteţi poziţia dreptei:

a) m faţă de C(I, 8 cm) dacă d(I, m) = 3 cm;b) n faţă de C(I, 8 cm) dacă d(I, n) = 10 cm;c) p faţă de C(I, 8 cm) dacă d(I, p) = 8 cm;d) k faţă de C(I, 8 cm) dacă d(I, k) = 0;e) h faţă de C(I, 8 cm) dacă d(I, h) = 8 cm.

88 Cap. 3. Cercul

¸ Poziţiile relative a două cercuri

l Două cercuri sînt exterioare dacă discurile circulare determinate de ele nu aupuncte comune.Teorema cercurilor exterioare. C(O, R) şi C(O¢, R¢) sînt exterioare dacă şinumai dacă distanţa dintre centrele lor OO¢ este mai mare decît R + R¢.

l Două cercuri sînt tangente exterioare dacă discurile circulare determinate de eleau un punct comun şi numai unul.Teorema cercurilor tangente exterioare. C(O, R) şi C(O¢, R¢) sînt tangente ex-terioare dacă şi numai dacă distanţa dintre centrele lor OO¢ = R + R¢.

l Două cercuri sînt secante dacă au două puncte comune.Teorema cercurilor secante. C(O, R) şi C(O¢, R¢) sînt secante dacă şi numai dacă distanţa dintre centrele lor |R – R¢| < OO¢ < R + R¢.

l Două cercuri sînt tangente interioare dacă discurile circulare determinate de ele aupuncte comune şi cercurile au un punct comun şi numai unul.Teorema cercurilor tangente interioare. C(O, R) şi C(O¢, R¢) sînt tangente interioare dacă şi numai dacă distanţa dintre centrele lor OO¢ = |R – R¢|.

l Două cercuri sînt interioare dacă unul dintre discurile circulare determinate de ele esteinclus în celălalt şi cercurile nu au puncte comune.Teorema cercurilor interioare. C(O, R) şi C(O¢, R¢) sînt interioare dacă şi numai dacădistanţa dintre centrele lor OO¢ < |R – R¢|.

l Două cercuri sînt concentrice dacă centrele lor coincid.Teorema cercurilor concentrice. C(O, R) şi C(O¢, R¢) sînt concentrice dacă şi numai dacădistanţa dintre centrele lor OO¢ = 0.

O O¢14243678678R R¢

O O¢14243678 678R R¢

O O¢14243678678R R¢

OO¢678678

R

O O¢678678R

R¢678

O O¢678678R

R¢678

l Mediatoarea segmentului AB

l Perpendiculara din M pe dreapta d

l Bisectoarea unui unghi

¹ Construcţii geometrice

A B A B A B

Se construieşte: un arc al C(A, R); un arc al C(B, R) şi se obţin punctele M, N; MN este mediatoarea [AB].

M

N

M

N

A Bd

Md

Md

M

A Bd

M

A Bd

M Md

A B

89Cap. 3. Cercul

E x e r c i ţ i i 1. Completaţi propoziţiile:

a) Două cercuri se numesc exterioare, dacă ........... .....................b) Două cercuri se numesc tangente exterioare, dacă ................................c) Două cercuri se numesc secante, dacă ................................

2. Completaţi propoziţiile:a) Două cercuri se numesc tangente interioare, dacă ................................b) Două cercuri se numesc interioare, dacă ................................c) Două cercuri se numesc concentrice, dacă ................................

3. Examinaţi figurile geometrice şi descrieţi poziţiile relative ale celor două cercuri.

O O¢142436786783 2

4

a)

OO¢678678

3

2

b)

O O¢14243678 6783 2

6

c)4. Examinaţi figurile geometrice şi denumiţi poziţiile relative ale celor două cercuri.

O O¢6786783

2678

0,4a)

O O¢6786783

2678

1 b)

O O¢142436786783 2

5c)

5. Construiţi:a) C(A, 3 cm), C(B, 5 cm), AB = 6 cm;b) C(A, 3 cm), C(B, 5 cm), AB = 8,5 cm;c) C(A, 3 cm), C(B, 5 cm), AB = 8 cm;d) C(A, 3 cm), C(B, 5 cm), AB = 1,2 cm;e) C(A, 3 cm), C(B, 5 cm), AB = 2 cm;f) C(A, 3 cm), C(B, 5 cm), AB = 0.

6. Recunoaşteţi fiecare dintre poziţiile relative ale cer-curilor construite la ex. 5.7. Completaţi propoziţiile:

a) C(O, R) şi C(O¢, R¢) sînt exterioare dacă şi numai dacă distanţa dintre centrele lor OO¢ este ..........................b) C(O, R) şi C(O¢, R¢) sînt tangente exterioare dacă şi numai dacă distanţa dintre centrele lor ................................c) C(O, R) şi C(O¢, R¢) sînt secante dacă şi numai dacă distanţa dintre centrele lor ..........................

8. Completaţi propoziţiile:a) C(O, R) şi C(O¢, R¢) sînt tangente interioare dacă şi numai dacă distanţa dintre centrele lor ...............b) C(O, R) şi C(O¢, R¢) sînt interioare dacă şi numai dacă distanţa dintre centrele lor ........................c) C(O, R) şi C(O¢, R¢) sînt concentrice dacă şi numai dacă distanţa dintre centrele lor .................

9. Fie C(A, 9 cm), C(B, 6 cm). Denumiţi poziţia unui cerc faţă de celălalt, dacă:

a) AB = 42 mm; b) AB = 151 mm;c) AB = 26 mm; d) AB = 3 cm;e) AB = 15 cm; f) AB = 0.

10. Construiţi mediatoarea segmentului:a) AB = 64 mm; b) CD = 52 mm;c) EF = 74 mm; d) GH = 58 mm;e) KL = 85 mm; f) MN = 75 mm.

11. Construiţi cu rigla şi compasul perpendicularele din A şi B pe dreapta d.

AB

d

12. Construiţi cu rigla şi compasul bisectoarea:a) unghiului A de 49°;b) unghiului B de 113°;c) unghiului C de 97°;d) unghiului D de 173°;e) unghiului E de 89°.

90 Geometrie. Cap. 4. Triunghiul

Capitolul 4 Triunghiul

Din viaţă

¶ Triunghiul. Definiţie. Elemente

· Clasificarea triunghiurilor după unghiuri

l Fie punctele necoliniare A, B, C. Triunghiul ABC este reuniunea segmentelor AB, BC, AC. Notaţie: DABC = [AB] È [BC] È [AC].l Triunghiul ABC are: laturile AB, BC, AC (fiecare latură este un segment); vîrfurile A,B, C; unghiurile BAC, ABC, ACB. Vîrfului A i se opune latura BC; lui B i se opune la-tura AC; lui C i se opune latura AB. Laturii AB i se opune vîrful C; laturii BC i se opu-ne vîrful A; laturii AC i se opune vîrful B. Laturii AB i se opune unghiul C; lui AC i seopune unghiul B; lui BC i se opune unghiul A. Unghiului A i se opune latura BC; lui B i seopune latura AC; lui C i se opune latura AB. Unghiurile A şi B sînt alăturate laturii AB; unghiurile A şi C sînt alăturate laturii AC; unghiurile B şi C sînt alăturate laturii BC.l Interiorul triunghiului ABC este Int ABC = Int ÐA Ç Int ÐB.l Exteriorul triunghiului ABC este Ext ABC = Ext ÐA È Ext ÐB.l Lungimile laturilor triunghiului ABC sînt: AB = c, BC = b, AC = a. Perimetrul triunghiului ABC este suma lungimilor laturilor lui şi P = a + b + c. Semiperimetrul triunghiului ABC este p = 0,5P.

l Triunghiul ascuţitunghic are toate unghiurile ascuţite. Triunghiul FGH este ascuţitunghic.l Triunghiul obtuzunghic are un unghi obtuz.Triunghiul KLM este obtuzunghic.l Triunghiul dreptunghic are un unghi drept.Triunghiul ABC este dreptunghic, are unghiul A drept, [AB] şi [AC] sînt catetele triunghiului, iar [BC] este ipotenuza triunghiului.

l În montaj sînt prezentate cîteva situaţii din viaţă. Ce figură geometrică plană stă la baza semnelor de circulaţie din imagine şi poate fi recunoscută în celelalte construcţii?Rezolvare. Semnele de circulaţie ilustrate au la bază triunghiuri. Triunghiu-rile se recunosc uşor şi în celelalte construcţii.

AB

C

Int ABC

Ext ABCExt A

BC

Ext ABC

F

GH

K

L M A B

C

Cat

etă

Catetă

Ipotenuză

¸ Clasificarea triunghiurilor după laturi

Triunghiul isoscel are două laturi congruente. Triunghiul echilateral are toate laturile congruente. Triunghiul scalen (oarecare) are laturile de lungimi diferite.

Laturicongruente

Baza

VîrfultriunghiuluiM

N P

¹ Construcţii geometrice

Construcţia triunghiului echilateral

Construcţia triunghiului isoscel

91Geometrie. Cap. 4. Triunghiul

E x e r c i ţ i i 1. Completaţi propoziţia:Triunghiul determinat de punctele necoliniare M, N, P este ..................................2. Completaţi egalităţile după model:

a) DABC = [AB] È [BC] È [AC];b) DDEF = ..........................................;c) DGHI = ...........................................;d) DJKL = ............................................;e) DPQR = ...........................................

3. Completaţi desenele ca în model

AB

C

Int ABC

Ext ABCExt A

BC

Ext ABC D E

L

Int .......

Ext .......Ext .

......

Ext .......

KM

N

Int .......

Ext .......Ext .

......

Ext ....... E

F

G

Int .......Ext .......Ex

t .....

..

Ext .......4. Completaţi:

a) Triunghiul DEF are laturile ............................. şi unghiurile ....................................b) Triunghiul GHI are laturile ............................. şi unghiurile ....................................c) Triunghiul JKL are laturile ............................. şi unghiurile ....................................d) Triunghiul MNP are laturile ............................. şi unghiurile ....................................

5. Completaţi:a) Fie triunghiul DCA. Vîrfului D i se opune latura ......; vîrfului C i se opune latura ......; vîrfului A i se opune latura ......; laturii DC i se opune vîrful ....; laturii CA i se opune vîrful ....; laturii AD i se opune vîrful ....; unghiului D i se opune latura ......; unghiurile D şi C sînt alăturate laturii ....b) Fie triunghiul BEF. Vîrfului B i se opune latura ......; vîrfului E i se opune latura ......; vîrfului F i se opune latura ......; laturii BE i se opune vîrful ....; laturii EF i se opune vîrful ....; laturii BF i se opune vîrful ....; unghiului B i se opune latura ......; unghiurile B şi E sînt alăturate laturii ....c) Fie triunghiul BEF. Vîrfului B i se opune latura ......; vîrfului E i se opune latura ......; vîrfului F i

se opune latura ......; laturii BE i se opune vîrful ....; laturii EF i se opune vîrful ....; laturii BF i se opune vîrful ....; unghiului B i se opune latura ......; unghiurile B şi E sînt alăturate laturii ....

6. Completaţi egalităţile după model:a) Int ABC = Int ÐA Ç Int ÐB;b) Int DIR = ..............................;c) Int EFG = ..............................;d) Int HKL = ..............................;e) Int MNP = ..............................

7. Completaţi egalităţile după model:a) Ext ABC = Ext ÐA Ç Ext ÐB;b) Ext DIR = ..............................;c) Ext EFG = ..............................;d) Ex HKL = ..............................;e) Ext MNP = ..............................

8. Perimetrul triunghiului:a) DEF este P = DE + EF + DF. Semiperimetrul este p = 0,5P.b) GHK este P = ................. Semiperimetrul este p = .........c) GHK este P = ................. Semiperimetrul este p = .........

9. Recunoaşteţi triunghiurile ascuţitunghice, dreptun-ghice şi obtuzunghice.

B

A

CD

FK

M N

PE

G

H

R

S

TI

J

L

10. Completaţi denumirile laturilor triunghiului drept-unghic (v. ex. 9):

a) MNP; b) RST.11. Completaţi denumirile elementelor triunghiurilor isoscele:

B

A

CD F

KM

N

P

G RL

12. Construiţi un triunghi echilateral cu laturile:a) 4 cm; b) 5,3 cm; c) 3,8 cm; d) 4,5 cm.

13. Construiţi un triunghi isoscel cu:a) baza de 4 cm şi laturile congruente de 3,6 cm;b) baza de 5 cm şi laturile congruente de 3,5 cm;c) baza de 3,8 cm şi laturile congruente de 4 cm;d) baza de 3,2 cm şi laturile congruente de 5 cm.

92 Geometrie. Cap. 4. Triunghiul

º Medianele triunghiului

l O mediană a unui triunghi este un segment determinat de un vîrf al triunghiului şi mijlocul latu-rii opuse lui. Orice triunghi are trei mediane.

¼ Înălţimile triunghiului

A

B

C

F

D E

K

L

M

GG G

» Bisectoarele triunghiului

l O bisectoare a unui triunghi este un segment cu mulţimea punctelor interioare egală cu intersecţia bisectoarei unui unghi al triunghiului cu interiorul triunghiului. Orice triunghi are trei bisectoare.

A

B

C

l O înălţime a unui triunghi este un segment determinat de un vîrf al triunghiului şi proiecţia sa pe dreapta ce conţine latura opusă lui. Înălţimi se numesc atît lungimile acestor segmente, cît şi dreptele ce le conţin.

A

B C

F

D E

P

K

L M

½ Mediatoarele triunghiului

l Mediatoarele unui triunghi sînt mediatoarele laturilor triunghiului.

A

B C

F

D E

M

K L

Triunghiul ascuţitunghic

Triunghiul dreptunghic

Triunghiul obtuzunghic

Triunghiul ascuţitunghic

Triunghiul dreptunghic

Triunghiul obtuzunghic

93Geometrie. Cap. 4. Triunghiul

E x e r c i ţ i i 1. Completaţi propoziţia:O mediană a unui triunghi este un segment determi-nat de ..................................2. Construiţi medianele triunghiurilor:

3. Construiţi medianele triunghiurilor aplicînd proce-deul plierii hîrtiei.

4. Completaţi propoziţia:O bisectoare a unui triunghi este un segment deter-minat de intersecţia dintre o bisectoare a unui unghi al triunghiului cu ..................................5. Construiţi bisectoarele triunghiurilor:

6. Construiţi bisectoarele triunghiurilor pliind hîrtia.

7. Pentru triunghiurile de mai sus, încercaţi să con-struiţi un cerc cu centrul în punctul de intersecţie al bisectoarelor, tangent laturilor triunghiului.8. Completaţi propoziţiile:a) O înălţime a unui triunghi este un segment determi-nat de un vîrf al triunghiului şi proiecţia sa ..................................................................b) Înălţimi ale triunghiului se numesc atît lungimile .....................................................................................9. Construiţi înălţimile triunghiurilor dreptunghice.

10. Construiţi înălţimile triunghiurilor ascuţitunghice.

11. Construiţi înălţimile triunghiurilor obtuzunghice.

12. Construiţi înălţimile triunghiurilor dreptunghice de mai sus (v. ex. 9) aplicînd procedeul plierii hîrtiei.13. Construiţi înălţimile triunghiurilor ascuţitunghice de mai sus (v. ex. 10) aplicînd procedeul plierii hîrtiei.14. Construiţi înălţimile triunghiurilor obtuzunghice de mai sus (v. ex. 11) aplicînd procedeul plierii hîrtiei.

94 Geometrie. Cap. 4. Triunghiul

15. Completaţi propoziţia:Mediatoarele unui triunghi sînt mediatoarele laturi-lor ...............16. Construiţi mediatoarele triunghiurilor dreptun-ghice.

17. Construiţi mediatoarele triunghiurilor ascuţitun-ghice.

18. Construiţi mediatoarele triunghiurilor obtuzun-ghice.

19. Construiţi mediatoarele triunghiurilor dreptun-ghice de mai sus (v. ex. 16) aplicînd procedeul plierii hîrtiei.20. Construiţi mediatoarele triunghiurilor ascuţitun-ghice de mai sus (v. ex. 17) aplicînd procedeul plierii hîrtiei.21. Construiţi mediatoarele triunghiurilor obtuzun-ghice de mai sus (v. ex. 18) aplicînd procedeul plierii hîrtiei.22. Identificaţi elementele triunghiurilor ilustrate în desen.

A B

C

D E

F

K

L

M

23. Identificaţi elementele triunghiurilor ilustrate în desen.

A B

C

D E

F

K

L

M

24. Identificaţi elementele triunghiurilor ilustrate în desen.

A B

C

D E

F

K

L

M

25. Identificaţi elementele triunghiurilor ilustrate în desen.

A B

C

D E

F

K

LM

26. Caracterizaţi după laturi triunghiurile ilustrate.

A B

C

DE

F M

N P

95Geometrie. Cap. 4. Triunghiul

27. Examinaţi înălţimile triunghiului

A B

C

D

dreptunghic ABC.Enumeraţi înălţimile triunghiului şiidentificaţi punctul lor de intersecţie.28. Descoperiţi în rezolvarea unuiadintre exerciţiile anterioare înălţimi-le unui triunghi ascuţitunghic.Care este poziţia intersecţiei lorfaţă de triunghi?29. Descoperiţi în rezolvarea unuia dintre exerciţiile anterioare înălţimile unui triunghi obtuzunghic.Care este poziţia intersecţiei lor faţă de triunghi?30. Descoperiţi în rezolvarea unuia dintre exerciţiile anterioare mediatoarele unui triunghi dreptunghic.Care este poziţia intersecţiei lor faţă de triunghi?31. Descoperiţi în rezolvarea unuia dintre exerciţiile anterioare mediatoarele unui triunghi ascuţitunghic.Care este poziţia intersecţiei lor faţă de triunghi?32. Descoperiţi în rezolvarea unuia dintre exerciţiile anterioare mediatoarele unui triunghi obtuzunghic.Care este poziţia intersecţiei lor faţă de triunghi?33. Construiţi medianele şi bisectoarele unui triunghi echilateral cu laturile de 5 cm. Ce constataţi? 34. Construiţi medianele şi înălţimile unui triunghi

echilateral cu laturile de 4 cm. Ce constataţi?35. Construiţi bisectoarele şi mediatoarele unui tri-unghi echilateral cu laturile de 6 cm. Ce constataţi?36. Completaţi teorema:Dacă un triunghi este echilateral, atunci medianale şi bisectoarele lui .....37. Completaţi teorema:Dacă un triunghi este echilateral, atunci medianale şi înălţimile lui .....38. Completaţi teorema:Dacă un triunghi este echilateral, atunci bisectoarele lui sînt conţinute de .............................. lui.39. Completaţi teorema:Dacă un triunghi este echilateral, atunci medianele lui sînt conţinute de .............................. lui.40. Construiţi bisectoarea şi mediana din vîrful triun-ghiului isoscel cu baza de 4 cm şi laturile congruente de 3,5 cm. Ce constataţi?41. Construiţi bisectoarea şi înălţimea din vîrful triun-ghiului isoscel cu baza de 5 cm şi laturile congruente de 4,3 cm. Ce constataţi?42. Construiţi mediatoarea bazei şi bisectoarea din vîrful triunghiului isoscel cu baza de 6 cm şi laturile congruente de 5 cm. Ce constataţi?

43. Fie triunghiul ABC. Se marchează puncte ale la-turii BC: M1, M2, ..., M15,. Completaţi în fiecare dintre situaţiile ilustrate numărul triunghiurilor.

A

B C

A

B CM1

......... triunghi .......... triunghiuri

B

A

CM1 M2B

A

CM1 M2 M15...........

......... triunghiuri ............. triunghiuri44. Fie triunghiul ABC. Se marchează puncte pe laturi şi se construiesc triunghiuri ca în desen. Pentru fie-

care situaţie aflaţi numărul triunghiurilor. Cîte triun-ghiuri se obţin după 10 paşi?

A

B CA

B C

A

B CA1

B1C1

A1

B1C1

A2

B2

C2

45. Aflaţi perimetrul unui triunghi echilateral cu două dintre laturi de lungimi 3x + 7 şi 5x + 3.46. Aflaţi perimetrul unui triunghi isoscel cu baza de lungime x + 3 şi celelalte laturi 4x + 3 şi 2x + 8.

96 Geometrie. Cap. 4. Triunghiul

¾ Construcţia triunghiului

Triunghiul isoscel cu lungimilelaturilor congruente şi lungimeabazei date.

Baza

Triunghiul scalen cu lungimilelaturilor date.

Triunghiul dreptunghic cu lungimile catetelor date. Se construieşte un unghi

drept şi una dintre catete

Triunghiul cu lungimile a două laturi şi măsura un-ghiului format de ele date.

32°

32°

Se construiesc unghiul şi una dintre laturi.

32° 32°

Triunghiul cu lungimea a două laturi şi măsurile un-ghiurilor alăturate ei date.

39°, 44°

Se construiesc latura şi unghiurile alăturate ei.

39° 44°39° 44° 39° 44°

97Geometrie. Cap. 4. Triunghiul

E x e r c i ţ i i 1. Construiţi un triunghi isoscel cînd este dată lungi-mea bazei şi lungimea uneia dintre laturile congru-ente.

a) Baza Altă latură

b) Baza Altă latură

c) Baza Altă latură

d) Baza Altă latură

e) Baza Altă latură

2. Construiţi un triunghi isoscel cînd este dat unghiul din vîrf şi lungimea uneia dintre laturile congruente.

Una dintre laturile congruentea)

Una dintre laturile congruenteb)

Una dintre laturile congruentec)

Una dintre laturile congruente

d)

3. Construiţi un triunghi isoscel cînd este dată măsura unghiului din vîrf şi lungimea uneia dintre laturile congruente.

a) 17°, Una dintre laturile congruente

b) 21°, Una dintre laturile congruente

c) 24°, Una dintre laturile congruente

d) 24°, Una dintre laturile congruente

e) 38°, Una dintre laturile congruente

4. Construiţi un triunghi isoscel cînd este dată măsura unghiului opus bazei şi lungimea uneia dintre laturile congruente.

a) 67°, una dintre laturile congruente 4,3 cm;b) 78°, una dintre laturile congruente 3,8 cm;c) 95°, una dintre laturile congruente 5,7 cm;d) 103°, una dintre laturile congruente 4,5 cm;e) 110°, una dintre laturile congruente 5,1 cm.

5. Construiţi un triunghi dreptunghic cu lungimile ca-tetelor date.

a) Lungimile catetelor: 3 cm, 4,5 cm.b) Lungimile catetelor: 3,5 cm, 4,2 cm.c) Lungimile catetelor: 5,1 cm, 4,4 cm.d) Lungimile catetelor: 3,3 cm, 5,2 cm.e) Lungimile catetelor: 4,8 cm, 5,5 cm.

6. Construiţi un triunghi isoscel cînd sînt date lungi-mile laturilor.

a) 4 cm, 4 cm, 4,5 cm.b) 3,5 cm, 3,5 cm, 2,8 cm.c) 4,3 cm, 4,3 cm, 3,3 cm.d) 4,8 cm, 4,8 cm, 3,1 cm.e) 5,1 cm, 5,1 cm, 4,8 cm.

7. Construiţi un triunghi scalen cînd sînt date lungi-mile laturilor.

a) 3 cm, 5,3 cm, 6,5 cm.b) 2,5 cm, 3,8 cm, 4,2 cm.c) 2,9 cm, 4,6 cm, 3,7 cm.d) 3,4 cm, 5,6 cm, 5,3 cm.

8. Construiţi un triunghi scalen cînd sînt date lungi-mile a două laturi şi măsura unghiului format de ele.

a) 3,5 cm, 2,5 cm, 78°.b) 4,7 cm, 5 cm, 55°.c) 5,8 cm, 4,5 cm, 62°.d) 6,3 cm, 3,1 cm, 83°.e) 2,9 cm, 5,9 cm, 51°.

9. Construiţi un triunghi dreptunghic cînd se dau lun-gimea unei catete şi măsura unghiului format de ea cu ipotenuza.

a) 71 mm, 37°.b) 67 mm, 28°.c) 58 mm, 66°.d) 35 mm, 25°.e) 45 mm, 43°.

10. Construiţi un triunghi cînd se dau lungimea unei laturi şi măsurile unghiurilor formate de ea cu cele-lalte laturi.

a) 34 mm, 36°, 36°.b) 22 mm, 54°, 54°.c) 43 mm, 46°, 46°.d) 37 mm, 34°, 34°.

11. Construiţi un triunghi scalen cînd se dau lungimea unei laturi şi măsurile unghiurilor formate de ea cu celelalte laturi.

a) 52 mm, 43°, 29°.b) 25 mm, 35°, 62°.c) 49 mm, 55°, 38°.

98 Geometrie. Cap. 4. Triunghiul

12. Construiţi un triunghi dreptunghic cu lungimea unei catete de 46 mm şi unghiul format de ea cu ipo-tenuza, congruent cu complementul unghiului de 35°.13. Construiţi C(A, 2,5 cm) şi coarda BC = 2,5 cm. Ce tip de triunghi este ABC?

A B

C

14. Construiţi C(O, 2,3 cm), diametrul BC şi coarda AB = 3 cm. Ce tip de triunghi este ABC? (Ce măsură are unghiul A?)

O

B

C

A

15. Construiţi un triunghi isoscel cu laturile congru-ente de 3,8 cm şi unghiul opus bazei suplementul un-ghiului de 134°.16. Construiţi un triunghi echilateral cu perimetrul de 6,3 cm.17. Construiţi un triunghi isoscel cu perimetrul egal cu 90 mm şi baza de 22 mm.18. Construiţi un triunghi echilateral, ştiind că lungi-mile a două laturi, în centimetri, sînt 3x + 5 şi 5x – 4.

19. Construiţi un triunghi dreptunghic cu:a) lungimea ipotenuzei 4,8 cm şi o catetă de 3 cm;b) lungimea ipotenuzei 3,6 cm şi un unghi de 27°.

20. a) Dreptele a şi b sînt perpendiculare pe dreapta AB. Cercetaţi dacă dreptele a şi b sînt concurente.

A

a b

Bb) Se poate construi un triunghi cu o latură de 5 cm şi unghiurile ce le formează cu celelalte două drepte?

21. a) Dreptele a şi AB formează un unghi de 93°, iar dreptele b şi AB formează un unghi de 95°. Cercetaţi dacă dreptele a şi b sînt concurente.

93°A

a b

B95°

b) Se poate construi un triunghi cu o latură de 4 cm şi unghiurile ce le formează cu celelalte două respectiv de 58° şi 135°?

22. a) Comparaţi lungimea unei laturi a triunghiului MNP cu suma lungimilor celelalte două. Ce relaţii există între ele?

M N

P

b) Se poate construi un triunghi cu lungimile la-turilor 3 cm, 7 cm şi 9 cm?

23. În desen dreapta EM descompune triunghiulDEF în două triun-ghiuri: unul ascuţit-unghic, celălalt obtuz-unghic.

b) Se poate descompune un triunghi ascuţitunghic în triunghiuri ascuţitunghice?

D

E F

M

99Geometrie. Cap. 4. Triunghiul

E VA L U A R EI I I

1. Completaţi egalitatea

DDFG = ......................................... D F

G

2. Enumeraţi elementeletriunghiului: D

N P E F

Ka) isoscel DNP;b) dreptunghic EFK.

3. Construiţi triunghiul:a) isoscel BFJ cu baza FJ = 2,3 cm şi BF = 3,4 cm;b) echilateral ACG cu AC = 3,5 cm.

4. Construiţi medianele triunghiului BDF.B

D F

A

G

K

5. Construiţi bisectoarele triunghiului AGK.6. Construiţi înălţimile triunghiului ABC.

A

B C

F

D E

7. Construiţi mediatoarele triunghiului DEF.8. Construiţi înălţimile triunghiului GJK.

K

GJ

N

L

M

9. Construiţi mediatoarele triunghiului LMN.

1. Completaţi egalitatea

DPRN = ......................................... R

NP

2. Enumeraţi elementeletriunghiului:

D L

PA

B J

a) isoscel ABJ;b) dreptunghic DLP.

3. Construiţi triunghiul:a) isoscel CDN cu baza DN = 3,1 cm şi CD = 4,1 cm;b) echilateral BVH cu BV = 2,9 cm.

4. Construiţi medianele triunghiului BDF.B

D F

A

G

K

5. Construiţi bisectoarele triunghiului AGK.6. Construiţi înălţimile triunghiului ABC.

A

B C

F

D E

7. Construiţi mediatoarele triunghiului DEF.8. Construiţi înălţimile triunghiului GJK.

K

GJ

NL

M

9. Construiţi mediatoarele triunghiului LMN.

1

1

1

1

11

1

11

2

100 Geometrie. Cap. 5. Congruenţa triunghiurilor

Capitolul 5 Congruenţa triunghiurilor ¶ Definiţia congruenţei triunghiurilor

· Criterii de congruenţă a triunghiurilor oarecare

l Fie punctele necoliniare A, B, C. Triunghiurile ABC, ACB, BCA, BAC, CAB şi CBA sînt mulţimi egale de puncte. DABC = DACB = DBCA = DBAC = DCAB = DCBA = [AB] È [BC] È [AC].l Triunghiul ABC este congruent cu triunghiul DEF şi se scrie „DABC ≡ DDEF“, dacă: [AB] ≡ [DE], [BC] ≡ [EF], [AC] ≡ [DF],ÐBAC ≡ ÐEDF, ÐABC ≡ ÐDEF, ÐACB ≡ ÐEFD.l Atenţie! Între elementele triunghiurilor ABC şi DEF care se compară prin congruenţă se stabileşte o corespondenţă care se descoperă uşor dacă se ţine cont: A D, B E, C F.Triunghiurile ABC şi DEF au laturile omoloage: AB şi DE; BC şi EF; AC şi DF. Triunghiurile ABC şi DEF au unghiurile omoloage: BAC şi EDF; ABC şi DEF; ACB şi DFE.l La stabilirea congruenţei a două triunghiuri ordinea literelor triunghiului al doilea depinde de ordinea literelor primului triunghi.l Dacă două triunghiuri sînt congruente, atunci ele pot fi suprapuse conform corespondenţei dintre ele.l Două triunghiuri sînt congruente, dacă laturile lor omoloage sînt congruente şi unghiurile lor omoloage sînt congruente.Teorema congruenţei triunghiurilor. Congruenţa triunghiurilor are proprietăţile egalităţii numerelor (reflexivitatea, simetria, tranzitivitatea).

Criteriul LUL, Axiomă. Dacă două triunghiuri au cîte două laturi şi unghiurile formate de ele respectiv congruente, atunci ele sînt congruente. (LUL de la Latură-Unghi-Latură!)Model de aplicare a criteriului LULIpoteza. Desenul şi relaţiile marcate în desen.Concluzia. DABC ≡ DDEF.

Criteriul ULU, Teoremă. Dacă două triunghiuri au cîte două unghiuri şi laturile alăturate lor respectiv congruente, atunci ele sînt congruente. (ULU de la Unghi-Latură-Unghi!)Model de aplicare a criteriului ULUIpoteza. Desenul şi relaţiile marcate în desen.Concluzia. DABC ≡ DDEF.

Criteriul LLL, Teoremă. Dacă două triunghiuri au laturile respectiv congruente, atunci ele sînt congru-ente. (LLL de la Latură-Latură-Latură!)Model de aplicare a criteriului LLLIpoteza. Desenul şi relaţiile marcate în desen.Concluzia. DABC ≡ DDEF.Teorema triunghiului isoscel. Un triunghi este isoscel dacă şi numai dacă are două unghiuri congruente. Teorema directă Teorema reciprocăIpoteza. Desenul şi relaţia marcată în desen.Concluzia. ÐMNP ≡ ÐMPN.

AB

C F

ED

A B

C

E D

F Conform LUL, [AB] ≡ [DE] (Ipoteza), [BC] ≡ [EF] (Ipo-teza) şi ÐBAC ≡ ÐEDF (Ipo-teza) implică DABC ≡ DDEF, c.c.t.d.

A

B

C D

E

F

Conform ULU [AC] ≡ [DE] (Ipoteza), ÐBAC ≡ ÐEDF(Ipoteza) şi ÐACB ≡ ÐDFE (Ipoteza) implică DABC ≡ DDEF, c.c.t.d.

A B

C F

DE

Conform LLL [AB] ≡ [DE] (Ipoteza), [BC] ≡ [EF] (Ipo-teza) şi [AC] ≡ [DF] (Ipoteza) implică DABC ≡ DDEF, c.c.t.d.

M

N P

Conform LUL, [MN] ≡ [MP] (Ipoteza), [MP] ≡ [MN] (Ipo-teza) şi ÐNMP ≡ ÐPMN (Ipo-teza) implică DMNP ≡ DMPN, de unde ÐMNP ≡ ÐMPN.

M

N P

Ipoteza. Desenul şi rela- ţia marcată în desen.Concluzia. [MN] ≡ [MP].

101Geometrie. Cap. 5. Congruenţa triunghiurilor

E x e r c i ţ i i 1. Formulaţi încă 3 nume pentru triunghiul:

a) DEF; b) GHI; c) JKL; d) MNP.2. Completaţi propoziţiile:

a) Triunghiul ABD este congruent cu triunghiul CEF dacă ...............................................................................................................................................b) Triunghiul AHJ este congruent cu triunghiul CBD dacă ...............................................................................................................................................c) Triunghiul KLT este congruent cu triunghiul MPG dacă .............................................................................................................................................

3. Completaţi definiţiile:a) DADF ≡ DBCE dacă ...............................................................................................b) DGIK ≡ DJLM dacă ...............................................................................................c) DNPR ≡ DSTQ dacă ...............................................................................................

4. Examinaţi desenul şi completaţi relaţiile:

DL

K A

J

M

a) DDKL ≡ .......................;b) DKDL ≡ .......................;c) DDLK ≡ .......................;d) DKLD ≡ .......................

5. Se ştie că DGIK ≡ DJLM. Completaţi congruenţele ţinînd cont de corespondenţele:

G J, I L, K M. a) DGKI ≡ .......................;b) DKIG ≡ .......................;c) DKGI ≡ .......................;d) DIKG ≡ .......................;e) DIGK ≡ .......................

6. Completaţi ca în model.a) DADF ≡ DBCE. Triunghiurile ADF şi BCE au laturile omoloage: AD, BC; AF, BE; DF, CE şi un-ghiurile omoloage: DAF, CBE; ADF, BCE; AFD, BEC.b) DGIK ≡ DJLM. Triunghiurile .......... şi .......... au laturile omoloage: ......, ......; ......, ......; ......, ...... şi unghiurile omoloage: .........., ..........; .........., ..........; .........., ..........

c) DNPR ≡ DSTQ. Triunghiurile .......... şi .......... au laturile omoloage: ......, ......; ......, ......; ......, ...... şi unghiurile omoloage: .........., ..........; .........., ..........; .........., ..........

7. Completaţi proprietăţile congruenţei triunghiurilor conform modelului.

a) DADF ≡ DADF; dacă DADF ≡ DBCE, atunci DBCE ≡ DADF.b) DGIK ≡ D........; dacă DGIK ≡ DJLM, atunci .......................c) DNPR ≡ D........; dacă DNPR ≡ DSTQ, atunci .......................

8. Completaţi criteriul LUL.Dacă două triunghiuri au cîte .................................................. respectiv congruente, atunci ele sînt con-gruente.9. Completaţi conform modelului de aplicare a LUL.

A D

E

IJ

L

a) Conform LUL, [AD] ≡ [IJ] (Ipoteza), [DE] ≡ [IL] (Ipoteza) şi ÐADE ≡ ÐJIL (Ipoteza) implică DADE ≡ DIJL, c.c.t.d.

N F

B

PH

E

b) Conform LUL, [.....] ≡ [.....] (Ipoteza), [.....] ≡ [.....] (Ipoteza) şi Ð......... ≡ Ð......... (Ipoteza) implică D......... ≡ D........., c.c.t.d.

G D

C

VI

S

c) Conform LUL, [.....] ≡ [.....] (Ipoteza), [.....] ≡ [.....] (Ipoteza) şi Ð......... ≡ Ð......... (Ipoteza) implică D......... ≡ D........., c.c.t.d.

M C

L

X

Q

Y

d) Conform LUL, [.....] ≡ [.....] (Ipoteza), [.....] ≡ [.....] (Ipoteza) şi Ð......... ≡ Ð......... (Ipoteza) implică D......... ≡ D........., c.c.t.d.

102 Geometrie. Cap. 5. Congruenţa triunghiurilor

10. Completaţi criteriul ULU.Dacă două triunghiuri au cîte ............................................. respectiv congruente, atunci ele sînt congru-ente.11. Completaţi conform modelului de aplicare a ULU.

A D

E

IJ

L

a) Conform ULU, [AD] ≡ [IJ] (Ipoteza), ÐADE ≡ ÐJIL (Ipoteza) şi ÐDAE ≡ ÐIJL (Ipoteza) implică DADE ≡ DIJL, c.c.t.d.

N F

B

PH

E

b) Conform ULU, [.....] ≡ [.....] (Ipoteza), Ð......... ≡ Ð......... (Ipoteza) şi Ð......... ≡ Ð......... (Ipoteza) implică D......... ≡ D........., c.c.t.d.

G D

C

VI

S

c) Conform ULU, [.....] ≡ [.....] (Ipoteza), Ð......... ≡ Ð......... (Ipoteza) şi Ð......... ≡ Ð......... (Ipoteza) implică D......... ≡ D........., c.c.t.d.

M C

L

X

Q

Y

d) Conform ULU, [.....] ≡ [.....] (Ipoteza), Ð......... ≡ Ð......... (Ipoteza) şi Ð......... ≡ Ð......... (Ipoteza) implică D......... ≡ D........., c.c.t.d.

12. Completaţi criteriul LLL.Dacă două triunghiuri au ................... respectiv con-gruente, atunci ele sînt congruente.13. Completaţi conform modelului de aplicare a LLL.

A D

E

IJ

L

a) Conform LLL, [AD] ≡ [IJ] (Ipoteza), [DE] ≡ [IL] (Ipoteza) şi [AE] ≡ [JL] (Ipoteza) implică DADE ≡ DIJL, c.c.t.d.

N F

B

PH

E

b) Conform LLL, [.....] ≡ [.....] (Ipoteza), [.....] ≡ [.....] (Ipoteza) şi [.....] ≡ [.....] (Ipoteza) implică D......... ≡ D........., c.c.t.d.

G D

C

VI

S

c) Conform LLL, [.....] ≡ [.....] (Ipoteza), [.....] ≡ [.....] (Ipoteza) şi [.....] ≡ [.....] (Ipoteza) implică D......... ≡ D........., c.c.t.d.

M C

L

X

Q

Y

d) Conform LLL, [.....] ≡ [.....] (Ipoteza), [.....] ≡ [.....] (Ipoteza) şi [.....] ≡ [.....] (Ipoteza) implică D......... ≡ D........., c.c.t.d.

14. Completaţi teorema triunghiului isoscel:Un triunghi este isoscel dacă şi numai dacă are ....... ................. congruente.15. Triunghiul isoscel ABC are baza BC. Aflaţi mÐB dacă:

a) mÐC = 34°; b) mÐC = 27°; c) mÐC = 56°. 16. Construiţi triunghiul isoscel ABC cu baza BC = 4 cm, dacă:

a) mÐB = 54°; b) mÐC = 49°; c) mÐB = 62°. 17. Examinaţi desenul şi enumeraţi relaţiile de con-gruenţă dintre triunghiuri ca în model.

a) Triunghiurile ABC şi ABD au: C

A B

D

latura comună AB ([AB] ≡ [AB]),[AC] ≡ [AD] (raze), [BC] ≡ [BD] (raze).

b)

G

E F

H

c)

K

I J

L

d)

P

M N

Q

103Geometrie. Cap. 5. Congruenţa triunghiurilor

18. Ţinînd cont de relaţiile marcate, completaţi de-monstraţia că [DF] ≡ [LK].

D E

F

JK

L

Conform LUL, [.....] ≡ [.....] (Ipoteza), [.....] ≡ [.....] (Ipoteza) şi Ð......... ≡ Ð......... (Ipoteza) implică D......... ≡ D........., de unde [.....] ≡ [.....], c.c.t.d.

19. Aplicînd definiţia cer- D

A C

B

cului şi un criteriu de con-gruenţă a triunghiurilor, demonstraţi că ÐADC ≡ ÐABC.20. Ţinînd cont de re-

A

B

O

C

D

laţiile din desen, de-monstraţi că:[AC] ≡ [BD] şiÐOBD ≡ ÐOAC.

21. Ţinînd cont de re-

A

B

O

C

D

laţiile din desen şi deproprietatea unghiuri-lor opuse la vîrf, de-monstraţi că:[AD] ≡ [BC],ÐODA ≡ ÐOBC şiÐDAO ≡ ÐBCO.

22. Completaţi desenul ex. 21 şi demonstraţi că: [AB] ≡ [CD], ÐABO ≡ ÐCDO şi ÐBAO ≡ ÐDCO.

23. Ţinînd cont de re-

A

B

O

C

D

laţiile din desen şi deproprietatea unghiuri-lor opuse la vîrf, de-monstraţi că:[AD] ≡ [BC],ÐODA ≡ ÐOCB şiÐDAO ≡ ÐCBO.

24. În desen sînt marcate segmentele congruente. De-monstraţi că:

[AB] ≡ [DC], ÐABC ≡ ÐBCD.A

B C

D

25. Demonstraţi că triunghiul isoscel BDC cu baza DC are unghiurile D şi C congruente.26. Demonstraţi că triunghiul BDC cu unghiurile D şi C congruente este isoscel cu baza DC.27. În desen sînt marcate segmentele congruente. De-monstraţi că:

a) [AB] ≡ [DC];b) ÐBAO ≡ ÐCDO;c) ÐABC ≡ ÐDCB.

A

B

C

D

O

28. Punctul O este mijlocul A

B

C

D

O

comun al segmentelor ACşi BD. Demonstraţi că:

a) [AD] ≡ [BC];b) ÐDAO ≡ ÐBCO ;c) [AB] ≡ [CD];d) ÐBAD ≡ ÐBCD.

104 Geometrie. Cap. 5. Congruenţa triunghiurilor

l Reciproca teoremei triunghiului isoscel se poate demonstra aplicînd criteriul ULU. l O consecinţă a teoremei triunghiului isoscel esteTeorema triunghiului echilateral. Un triunghi este echilateral dacă şi numai dacă are unghiurile con-gruente.Teorema medianelor corespunzătoare laturilor congruente. Medianele corespunzătoare laturilor con-gruente ale unui triunghi isoscel sînt congruente.

Ipoteza teoremei este ilustratăîn desen.Concluzia. [BD] ≡ [CE].

Teorema bisectoarelor corespunzătoare unghiurilor congruente. Bisectoarele corespunzătoare unghiu-rilor congruente ale unui triunghi isoscel sînt congruente.

Ipoteza teoremei este ilustratăîn desen.Concluzia. [BD] ≡ [CE].

Teorema medianei corespunzătoare bazei triunghiului isoscel. Mediana din vîrful triunghiului isoscel este bisectoare şi înălţime a triunghiului.

Ipoteza teoremei este ilustratăîn desen.Concluzia. ÐBAD ≡ ÐCAD, mÐADB = 90°.

¸ Congruenţa triunghiurilor dreptunghice (1)

Criteriul CC. Dacă două triunghiuri dreptunghice au catetele respectiv congruente, atunci ele sînt congruente. (CC de la Catetă-Catetă!)Evident, acest criteriu este un caz particular al axiomei LUL.Proprietatea mediatoarei segmentului. Teoremă. Punctele mediatoarei segmentului sînt egal depărtate de capetele segmentului.Ipoteza teoremei este ilustratăîn desen.Concluzia. [PA] ≡ [PB].

Criteriul CU. Dacă două triunghiuri dreptunghice au cîte o catetă şi ununghi alăturat ei respectiv congruente, atunci ele sînt congruente. (CU de la Catetă-Unghi ascuţit!)Evident, acest criteriu este un caz particular al criteriului ULU.

D E

F

A B

C

D E

F

A B

C

A B

P

M

dDemonstraţieConform CC, [MP] ≡ [MP] (catetă comună), [MA] ≡ [MB] (Ipoteza) implică DMAP ≡ DMBP, de unde [PA] ≡ [PB], c.c.t.d.

A

B C

E D

DemonstraţieConform LUL, [AB] ≡ [AC] (Ipoteza), [AD] ≡ [AE] (Ipo-teza), ÐBAD ≡ ÐCAE (unghi comun) implică DBAD ≡ DCAE, de unde [BD] ≡ [CE], c.c.t.d.

A

B C

E D

DemonstraţieConform ULU, [AB] ≡ [AC] (Ipoteza), ÐBAD ≡ ÐCAE (unghi comun), ÐABD ≡ ÐACE (Teorema triunghiului isoscel) implică DBAD ≡ DCAE, de unde [BD] ≡ [CE], c.c.t.d.

DemonstraţieConform LLL, [AB] ≡ [AC] (Ipoteza), [AD] ≡ [DA] (latură comună), [BD] ≡ [DC] (Ipoteza) implică DBAD ≡ DDAC, de unde ÐBAD ≡ ÐCAD, mÐADB = 90° (congruent cu suplementul său), c.c.t.d.

A

B CD

105Geometrie. Cap. 5. Congruenţa triunghiurilor

E x e r c i ţ i i 1. Completaţi teorema triunghiului echilateral:Un triunghi este echilateral dacă şi numai dacă are .............................................2. Măsuraţi medianele triunghiurilor ilustrate:

A

B CD

EF

M

N P

I

J KR

ST

U

VX

Triunghiul ABC

Triunghiul MNP

Triunghiul IJK

AD = MR = IU =BE = NS = JV =CF = PT = KX =

a) Comparaţi lungimile medianelor triunghiului scalen ABC.b) Comparaţi lungimile medianelor triunghiului isoscel MNPc) Comparaţi lungimile medianelor triunghiului echilateral IJK.

3. Completaţi proprietatea medianelor triunghiului isos-cel:Medianele corespunzătoare laturilor congruente ale triunghiului isoscel sînt .............................4. Completaţi proprietatea medianelor triunghiului echilateral:Medianele triunghiului echilateral sînt ....................5. Triunghiul isoscel ABC cu baza BC are medianele corespunzătoare laturilor congruente BE şi CF. Aflaţi:

a) CF, dacă BE = 3,2 cm;b) BE, dacă CF = 7,6 cm;c) CF, dacă BE = 13,7 cm;d) BE, dacă CF = 32,5 cm.

6. Triunghiul echilateral ABC are medianele AD, BE şi CF. Aflaţi:

a) CF şi BE, dacă AD = 68 mm;b) AD şi BE, dacă CF = 54 mm;c) AD şi CF, dacă BE = 98 mm;d) BE şi CF, dacă AD = 61 mm.

7. Măsuraţi bisectoarele triunghiurilor ilustrate.

A

B CD

EF

M

N P

I

J KR

ST

U

VX

Triunghiul ABC

Triunghiul MNP

Triunghiul IJK

AD = MR = IU =BE = NS = JV =CF = PT = KX =

a) Comparaţi lungimile bisectoarelor triunghiului scalen ABC.b) Comparaţi lungimile bisectoarelor triunghiului isoscel MNP.c) Comparaţi lungimile bisectoarelor triunghiului echilateral IJK.

8. Completaţi proprietatea bisectoarelor triunghiului isoscel:Bisectoarele unghiurilor congruente ale triunghiului isoscel sînt .............................9. Completaţi proprietatea bisectoarelor triunghiului echilateral:Bisectoarele triunghiului echilateral sînt .................10. Triunghiul isoscel ABC cu baza BC are bisec-toarele corespunzătoare laturilor congruente BE şi CF. Aflaţi:

a) CF, dacă BE = 3,6 cm;b) BE, dacă CF = 8,5 cm;c) CF, dacă BE = 12,8 cm;d) BE, dacă CF = 16,2 cm.

11. Triunghiul echilateral ABC are bisectoarele AD, BE şi CF. Aflaţi:

a) CF şi BE, dacă AD = 35 mm;b) AD şi BE, dacă CF = 74 mm;c) AD şi CF, dacă BE = 62 mm;d) BE şi CF, dacă AD = 75 mm.

12. Construiţi triunghiul isoscel ABC cu baza BC şi mediana corespunzătoare ei AD. Controlaţi dacă:

a) AD este bisectoare; b) AD este înălţime.13. Formulaţi teorema medianei corespunzătoare ba-zei triunghiului isoscel.

106 Geometrie. Cap. 5. Congruenţa triunghiurilor

14. Completaţi criteriul CC.Dacă două triunghiuri dreptunghice au catetele ........................................., atunci ele sînt congruente.15. Completaţi proprietatea mediatoarei segmentului.Punctele mediatoarei segmentului sînt .....................................................16. Completaţi criteriul CU.Dacă două triunghiuri dreptunghice au cîte o catetă şi un ........................................., atunci ele sînt con-gruente.17. Fie segmentul AB şi M un punct al mediatoarei lui. Aflaţi:

a) MA, dacă MB = 3,6 cm;b) MB, dacă MA = 57 mm;

c) MA, dacă MB = 6,8 cm;d) MB, dacă MA = 84 mm.

18. Construiţi triunghiul dreptunghic ABC cu ipote-nuza BC, dacă:

a) AB = 2,3 cm şi AC = 41 mm;b) AB = 3,7 cm şi AC = 33 mm;c) AB = 26 mm şi AC = 3,5 cm.

19. Construiţi triunghiul dreptunghic ABC cu ipote-nuza BC, dacă:

a) AB = 5,1 cm şi mÐB = 41°;b) AB = 43 mm şi mÐB = 21°;c) AB = 2,7 cm şi mÐB = 36°;d) AB = 39 mm şi mÐB = 28°.

20. Construiţi triunghiul isoscel ABC cu baza BC de:a) 34 mm şi mediana AD de 4,1 cm; b) 4,5 cm şi bisectoarea AD de 54 mm;c) 39 mm şi înălţimea AD de 6,1 cm.

Cîte triunghiuri pot fi construite în fiecare situaţie şi ce relaţie există între ele?21. Construiţi triunghiul isoscel ABC cu baza BC de 54 mm şi suma măsurilor unghiurilor alăturate bazei 74°.

22. Aplicînd un criteriu de congruenţă a triunghiurilor oarecare, demonstraţi că triunghiurile dreptunghi-ce ABC (cu ipotenuza BC) şi DEF (cu ipotenuza EF) cu [AB] ≡ [DE] şi [AC] ≡ [DF] sînt congruente.23. Aplicînd un criteriu de congruenţă a triunghiurilor oarecare, demonstraţi că triunghiurile dreptunghice ABC (cu ipotenuza BC) şi DEF (cu ipotenuza EF) cu [AB] ≡ [DE] şi ÐB ≡ ÐE sînt congruente.

24. DABC ≡ DDEF, [AM] este o mediană a triunghi-ului ABC, [DN] este o mediană a triunghiului DEF. Demonstraţi că [AM] ≡ [DN].25. DABC ≡ DDEF, [AM] este o înălţime a triun-ghiului ABC, [DN] este o înălţime a triunghiului DEF. Demonstraţi că [AM] ≡ [DN].26. DABC ≡ DDEF, [AM] este o bisectoare a triun-ghiului ABC, [DN] este o bisectoare a triunghiului DEF. Demonstraţi că [AM] ≡ [DN].27. Triunghiul isoscel ABC cu baza BC şi triunghiul isoscel DEF cu baza EF au bazele congruente şi [AM] ≡ [DN], unde M şi N sînt mijloacele bazelor. Demonstraţi că DABC ≡ DDEF.

A

B CM

D

E FN

28. D este mijlocul [BC], AD ^ BC. Demonstraţi că [PB] ≡ [PC].

B CD

P

29. D este mijlocul [BC], [PB] ≡ [PC]. Demonstraţi că AD ^ BC.

B CD

P

107Geometrie. Cap. 5. Congruenţa triunghiurilor

E VA L U A R EI I I

1. Cu ajutorul triunghiu-

A

F

H D

C J

rilor ilustrate completaţi: DAFH ≡ .............., dacă: ............................................................................. 2. Identificaţi triunghiurile congruente conform LUL.

A

B

C

D

E

F

G

H

I

J

KL

3. Identificaţi triunghiurile congruente conform ULU.

A

BF

G

I

KC

D E HJ

L

4. Identificaţi triunghiurile congruente conform LLL.

A

BF

GC

H J

LDE

I

K

5. Completaţi demonstraţia M

A

B C

D

congruenţei segmentelor AB şi CD:Conform .........., [AM] ≡ [....], [BM] ≡ [....] şi ÐAMB ≡.......... (opuse ............) implică DAMB ≡ .........., de unde [AB] ≡ [CD], c.c.t.d.

6. Cu ajutorul relaţiilor M

A

B C

D

marcate, completaţi demon-straţia că [AB] ≡ [CD].Conform .........., [AM] ≡ [....], [BM] ≡ [....] şi ÐAMB ≡.......... (opuse ............) implică DAMB ≡ .........., de unde ............................, c.c.t.d.

7. Cu ajutorul relaţiilor marcate, de-

C

A B

Dmonstraţi că ÐACB ≡ ÐADB.8. Se ştie că C–O–A, D–O–B. De-

A

B

OC

D

monstraţi că:a) [AD] ≡ [BC];b) ÐDAO ≡ ÐCBO;c) ÐDAB ≡ ÐCBA.

1. Cu ajutorul triunghiu-

B

D

Q D

A I

rilor ilustrate completaţi: DBDQ ≡ .............., dacă: ............................................................................. 2. Identificaţi triunghiurile congruente conform LUL.

I

S

C

A

D

F

N

O

P

E

ML

3. Identificaţi triunghiurile congruente conform ULU.

A

FD

T

I

NC

B E GR

P

4. Identificaţi triunghiurile congruente conform LLL.

I

ES

FC

A D

JGB

M

K

5. Completaţi demonstraţia O

E

F G

K

congruenţei segmentelor EF şi KG:Conform .........., [OF] ≡ [....], [OE] ≡ [....] şi ÐEOF ≡.......... (opuse ............) implică DEOF ≡ .........., de unde [EF] ≡ [KG], c.c.t.d.

6. Cu ajutorul relaţiilor I

E

F G

L

marcate, completaţi demon-straţia că [EF] ≡ [GL].Conform .........., [EI] ≡ [....], [FI] ≡ [....] şi ÐEIF ≡.......... (opuse ............) implică DEIF ≡ .........., de unde ............................, c.c.t.d.

7. Cu ajutorul relaţiilor marcate, de-

G

D F

Emonstraţi că ÐDEF ≡ ÐDGF.8. Se ştie că M–I–P, N–I–R. De-

P

R

IM

N

monstraţi că:a) [MR] ≡ [NP];b) ÐMRI ≡ ÐNPI;c) ÐMRP ≡ ÐNPR.

1

1

1

1

1

1

1

3

108 Geometrie. Cap. 6. Paralelism

Capitolul 6 Paralelism ¶ Definiţia dreptelor paralele

¸ Unghi exterior. Unicitatea perpendicularei

l Dreptele a şi c (v. desenul!) sînt concurente în punctul M şi se no-tează a Ç c = {M}.l Dreptele conţinute de acelaşi plan care nu au puncte comune se nu-mesc drepte paralele. a şi b (v. desenul!) sînt paralele şi se noteazăa || b sau a Ç b = Ø.l Dreptele care au două puncte comunte sînt drepte confundate (sau egale). De exemplu MN = c.Axioma paralelelor (Postulatul lui Euclid). Fiind date o dreaptă şi un punct exterior ei, există o singură dreaptă paralelă cu dreapta dată, care conţine punctul dat. De exemplu, conform Postulatului lui Euclid (PE), dreapta a este singura dreaptă paralelelă cu dreapta b, care conţine punctul M.l Atenţie! Postulatul lui Euclid este Axioma paralelelor pe care în expunerea noastră o acceptăm ca fiind adevărată.l Geometria euclidiană este teoria matematică în care Postulatul lui Euclid este considerat adevărat.

Din viaţă

l Examinaţi harta Chişinăului de la p. 74 şi daţi: două exemple de străzi ale căror direcţii sînt concurente; două exemple de străzi ale căror direcţii nu se intersectează (sînt paralele).Rezolvare. Direcţiile străzilor Bucureşti şi Vasile Alecsandri sînt concurente, ca şi direcţia străzii Vasile Alec-sandri şi cea a bd. Ştefan cel Mare.Direcţiile străzilor Bucureşti şi Şciusev nu se intersectează, ca şi direcţia străzii Bucureşti şi cea a bd. Ştefan cel Mare.

MN

a

b

c

l Suplementul unui unghi al triunghiului, adiacent acestui unghi este un unghi exterior tri-unghiului.Teorema unghiului exterior (1). Un unghi exterior unui triunghi este mai mare decît ori-care dintre unghiurile triunghiului neadiacente acestui unghi.

Ipoteza. Triunghiul ABC;ÐACM este exterior triun-ghiului.Concluzia. ÐA < ÐACM şiÐABC < ÐACM.

Teorema unicităţii perpendicularei coborîte dintr-un punct pe o dreaptă. Există o singură dreaptă perpendiculară pe o dreaptă, ce conţine un punct dat, exterior acestei drepte.l Procedeul de demonstraţie „prin reducere la absurd“ porneşte de la presupunerea că concluzia este falsă şi prin raţionament se deduce că ipoteza (sau o condiţie din ipoteză) este falsă sau că o propoziţie matematică deja demonstrată este falsă.

DemonstraţieSe construieşte [BD] care are mijlocul P, ce coincide cu mijlocul [AC]. Conform LUL, [AP] ≡ [PC] (construcţie), [PB] ≡ [PD] (construcţie), ÐAPB ≡ ÐCPD (unghiuri opuse la vîrf) implică ∆APB ≡ ∆CPD, de unde ÐA ≡ ÐACD, (1). D Î Int ÐACM şi (1) implică ÐA < ÐACM. Analog se demonstrează că ÐABC < ÐACM, c.c.t.d.

A

MB C

D

P

· Unghiuri formate de două drepte cu o secantă

l Dreptele a şi b formează cu secanta c (v. desenul!): perechile de un-ghiuri alterne interne M2 şi N8, M3 şi N5; perechile de unghiuri alter-ne externe M1 şi N7, M4 şi N6; perechile de unghiuri corespondente M1 şi N5, M2 şi N6, M3 şi N7, M4 şi N8; unghiuri interne de aceeaşi parte a secantei M2 şi N5, M3 şi N8; unghiuri externe de aceeaşi parte a secantei M1 şi N6, M4 şi N7.

a c

b

M

N

12 3

4

56 7

8

109Geometrie. Cap. 6. Paralelism

E x e r c i ţ i i 1. Notaţi:

a) dreptele a şi b se intersectează în P;b) dreptele c şi d se intersectează în Q;c) dreptele e şi f se intersectează în R;d) dreptele e şi f se intersectează în M.

2. Completaţi propoziţiile:a) Dreptele conţinute de acelaşi plan care nu au puncte comune se numesc ................................b) Dreptele d şi e sînt paralele se notează .......... sau ................

3. Completaţi Axioma paralelelor:Fiind date o dreaptă şi un punct exterior ei, există ............................................................................., care conţine punctul dat.4. Examinaţi desenul şi completaţi:

m d

n

B

C

45 2

3

67 8

1

a) Perechile de unghiuri alterne interne sînt ..........b) Perechile de unghiuri alterne externe sînt .........c) Perechile de unghiuri corespondente sînt ..........d) Unghiurile interne de aceeaşi parte a secantei sînt ..........e) Unghiurile externe de aceeaşi parte a secantei sînt ..........

5. Completaţi:a) Unghiuri alterne interne:

B

A

C12 3

4

12 3

4

A2 ,......; .........b) Unghiuri alterne externe:.........................................c) Unghiuri corespondente: ...............................d) Unghiuri interne de aceeaşi parte a secantei: .........6. Examinaţi desenul! Recunoaşteţi tipurile de un-ghiuri formate de două drepte cu o secantă ajutaţi de completarea din desenul mic. Completaţi:

A

B C D1

1

1 2

a) Unghiurile A1 şi B1 (dreptele AC şi BC cu se-canta AB) sînt .......................b) Unghiurile A1 şi C2 (dreptele AB şi CD cu se-canta ......) sînt .......................c) Unghiurile A1 şi C1 (dreptele AB şi BC cu se-canta ......) sînt .......................d) Unghiurile B1 şi C1 (dreptele AB şi AC cu se-canta ......) sînt .......................

7. Examinaţi desenul şi completaţi propoziţiile:A

B C

D

12 34O

a) Unghiurile A şi C (dreptele AB şi CD cu secanta AC) sînt .......................b) Unghiurile B şi D (dreptele AB şi CD cu secanta .......) sînt .......................c) Unghiurile A şi O1 (dreptele AB şi BD cu secanta .......) sînt .......................d) Unghiurile C şi O4 (dreptele DO şi DC cu se-canta .......) sînt .......................e) Unghiurile D şi O2 (dreptele AC şi DC cu se-canta .......) sînt .......................

8. Completaţi propoziţiile:a) Suplementul unui unghi al triunghiului, ............ ................ este un unghi exterior triunghiului.b) Un unghi exterior unui triunghi este ............ decît oricare dintre unghiurile triunghiului ................................c) Există o singură dreaptă perpendiculară pe o dreaptă, ce conţine un punct dat, ................................................

9. Aplicaţi teorema unghiului exterior ca în model:

A

B C1 1

1

a) ÐBAC < ÐB1; b) ÐBAC < .......; c) ÐABC < .......; d) ÐABC < .......;e) ÐACB < .......; f) ÐACB < .......

110 Geometrie. Cap. 6. Paralelism

¹ Construcţii

l O consecinţă a teoremei de existenţă a paralelelorpermite construirea dreptelor paralele.Teorema perpendicularelor pe o dreaptă. Dacădouă drepte diferite sînt perpendiculare pe a treiadreaptă, atunci cele două drepte sînt paralele.

Ipoteza. Dreapta d ^ a şi M Î d. Concluzia. d este unica perpen-diculară pe a din M.

Teorema de existenţă a dreptelor paralele. Dacă două drepte formează cu o secantă două unghiuri al-terne interne congruente, atunci dreptele sînt paralele.Teorema directă (Necesitatea)Ipoteza. Dreptele a, b şisecanta c; ÐM1 ≡ ÐN1.Concluzia. a || b.

Teorema reciprocă (Suficienţa)Ipoteza. Dreptele a, b şisecanta c; a || b.Concluzia. ÐM1 ≡ ÐN1.

DemonstraţiePresupunem că dreapta e conţine M şi este perpendiculară pe a, (1). Conform teoremei unghiului exterior, ÐMBA < ÐA, (2). (1) contrazice (2), prin urmare, presupunerea este falsă. Deci d este unica perpendiculară pe a din M, c.c.t.d.

a

b

cM

N

P1

1

DemonstraţiePresupunem că a Ç b = {P}. Atunci, ÐN1 este exterior triunghiului MPN. Conform teoremei un-ghiului exterior, ÐM1 < ÐN1, (1). (1) contrazice ÐM1 ≡ ÐN1 (Ipoteza). Prin urmare, presupunerea este falsă. Deci a || b, c.c.t.d.DemonstraţiePresupunem că mÐM1 ≠ mÐN1, (1). Se construieşte dreapta d, M Î d, astfel încît ÐTMN ≡ ÐN1, (2). Conform teoremei directe, d || b. Conform Postu-latului lui Euclid (PE), d = a, (3). (2) şi (3) implică ÐM1 ≡ ÐN1, ceea ce contrazice (1). Prin urmare, presupunerea este falsă. Deci ÐM1 ≡ ÐN1, c.c.t.d.

a

b

cM

N

1

1

d

T

º Teorema dreptelor paralele. ConsecinţeExerciţiu rezolvat. Fie a || b, secanta c şi mÐM2 = 17°. Aflaţi măsurile celorlalteunghiuri.Rezolvare. Notaţii prescurtate pentru teoremele aplicate: Teorema Unghiurilor Opuse la Vîrf (TUOV), Teorema Unghiurilor Adiacente Suplementare (TUAS),Teorema de Existenţă a Dreptelor Paralele (TEDP). (Atenţie! anunţaţi notaţiileprescurtate la începutul sau pe parcursul unei rezolvări sau demonstraţii!)Conform mÐM2 = 17°, TUOV implică mÐM4 = 17°, TUAS implică mÐM1 = 163° şi mÐM3 = 163°. Conform mÐM2 = 17°, TEDP implică mÐN6 = 17°, (1), TUAS implică mÐN5 = 163°, mÐN8 = 17° şi mÐN7 = 163°.Rezolvarea exerciţiului permite formularea şi sugerează demonstrarea următoarei teoreme.Teorema paralelelor. Două drepte sînt paralele dacă şi numai dacă formează cu o secantă o pereche de unghiuri alterne interne congruente sau o pereche de unghiuri alterne externe congruente sau o pereche de unghiuri corespondente congruente sau o pereche de unghiuri interne de aceeaşi parte a secantei suplementare sau o pereche de unghiuri externe de aceeaşi parte a secantei suplementare.Corolar. Dacă două drepte sînt paralele, atunci ele formează cu o secantă unghiuri: alterne interne congruente, alterne externe congruente, corespondente congruente, interne de aceeaşi parte a secantei suplementare sau externe de aceeaşi parte a secantei suplementare.Tranzitivitatea paralelismului dreptelor. Teoremă. Dacă două drepte diferite sînt paralele cu a treia dreaptă, atunci ele sînt paralele.

a

b

cM

N

1 d2 3

4

56 7

8

Md

a

e

AB

111Geometrie. Cap. 6. Paralelism

E x e r c i ţ i i 1. Completaţi propoziţiile ce rezultă din unicitatea perpendicularei dintr-un punct pe o dreaptă:

a) Trei drepte perpendiculare dintr-un punct pe o dreaptă ................................b) Patru drepte perpendiculare dintr-un punct pe o dreaptă ................................

2. Completaţi teorema de existenţă a paralelelor:a) Dacă două drepte formează cu o secantă două unghiuri alterne interne congruente, atunci dreptele ...............................b) Dacă două drepte sînt paralele, atunci ele formează cu o secantă două unghiuri alterne in-terne .....................................

3. Completaţi următoarele consecinţe ale teoremei de existenţă a paralelelor:

a) Dacă două drepte diferite sînt perpendiculare pe a treia dreaptă, atunci cele două drepte sînt ........b) Două drepte sînt paralele dacă şi numai dacă for-mează cu o secantă o pereche de unghiuri alterne interne ............... sau o pereche de unghiuri alterne externe .............. sau o pereche de unghiuri corespon-dente ............... sau o pereche de unghiuri interne de aceeaşi parte a secantei .............. sau o pereche de unghiuri externe de aceeaşi parte a secantei ...........c) Dacă două drepte sînt paralele, atunci ele formează cu o secantă unghiuri: alterne interne ................., alterne externe ................., corespondente ............., interne de aceeaşi parte a secantei ................ sau ex-terne de aceeaşi parte a secantei .....................d) Dacă două drepte diferite sînt paralele cu a treia dreaptă, atunci ele sînt .........................

4. Aflaţi măsurile unghiurilor: B1, B2, B3, C1, C2, C3.m d

n

B

C

380° 1

2

12 3

30°

5. Aflaţi măsurile unghiurilor:B1, B2, B3, C1, C2, C3.

m

d

n

B

C

343° 1

2

12

3137°

6. Se ştie că a || b. Aflaţi măsurile celorlalte unghiuri, dacă:

a

d

b

B

C

34 1

2

12

34

a) mÐB2 = 21°; b) mÐB1 = 53°; c) mÐC1 = 57°;d) mÐB3 = 107°; e) mÐC2 = 11°; f) mÐC2 = 3°.

7. Se ştie că a || b. Aflaţi măsurile celorlalte unghiuri (v. desenul):

a

b

BC D

E

77°

M

102°85°

41°

a) formate de paralele cu secanta MB;b) formate de paralele cu secanta MC;c) formate de paralele cu secanta MD;d) formate de paralele cu secanta ME.

8. Aplicînd teorema paralelelor, stabiliţi în fiecare si-tuaţie adevărul propoziţiei „a || b“.

a) m

d

n

48°

132°

...............

b) m

d

n 35°

145° ...............

c) m

d

n

67°

67°

...............

112 Geometrie. Cap. 6. Paralelism

d) m

d

n133°

132°

...............

e)

md

n31°

29°

...............

9. Construiţi perpendicularele în punctele A, B, C, D, E, F pe dreapta d.

A B C D E Fd

10. Construiţi paralelele cu dreapta a prin punctele B, C, D, E, F.

B

CD

E

Fa

11. Construiţi paralelele

A

B C

prin vîrfurile triunghiului ABC cu laturile triunghiu-lui, opuse lor.

12. Patrulaterul ABCD are latu- A

B C

Drile opuse paralele. Aflaţi măsu-rile unghiurilor A, D, C dacă:

a) mÐB = 24°; b) mÐB = 38°.13. Se dau măsurile unghiurilor A şi B ale triunghiului ABC, CD || AB. Aflaţi măsurile unghiurilor C1 şi C2 în fiecare situaţie ca în model.a) Deoarece CD || AB, mÐC1 = mÐA = 34° (un-ghiuri alterne interne) şi mÐC2 = mÐB = 75° (un-ghiuri corespondente).

A

B C

D

E

34°

75° 12

b)

A

B C

D

E

50°

82° 12

c)

A

B C

D

E

41°

101°1

2

d)

A

B C

D

E

53°

63° 12

113Geometrie. Cap. 6. Paralelism

14. Aflaţi x ştiind că m || n.

m

d

n

5x + 19°

7x + 3°

15. Aflaţi x cu ajutorul datelor din desen.

md

n

3x + 13°

36x + 11°

57°

123°

16. Aflaţi x şi y cu ajutorul datelor din desen.

md

n7x + 5°

14x + 7°107°

73°

9y + 13°

17. Aflaţi măsura unghiului APB cu ajutorul datelor din desen ştiind că m || n || r.

m

d

n

117°

129°

P

A

Br

18. Aflaţi măsura unghiului ABC cu ajutorul datelor din desen.

m

d

n

133°

156°C

A

Br

19. În desen AB || CK || EJ || FH şi AD || FK || GI. Completaţi măsurile celorlalte unghiuri din desen.

62°73°

A

B

A

C D E F G

H

I

J

K

L

20. Aflaţi măsura unghiului APB cu ajutorul datelor din desen ştiind că m || n.

m

d

35°

128°

P

A

Bn

21. Aflaţi x şi măsura unghiului APB cu ajutorul date-lor din desen ştiind că m || n.

m

d

5x + 6°

8x + 2°

P

A

Bn

14x + 3°

22. Aflaţi x şi măsura unghiului ABC cu ajutorul da-telor din desen.

md

n

19x + 3°

11x + 2°C

A

B

7x +

114 Geometrie. Cap. 6. Paralelism

» Triunghiul. Consecinţe ale teoremei paralelelorTeorema unghiului exterior (2). Măsura unui unghi exterior al triunghiului esteegală cu suma măsurilor unghiurilor triunghiului, neadiacente unghiului exterior.Suma măsurilor unghiurilor unui triunghi. Teoremă. Suma măsurilor unghiuri-lor unui triunghi este 180°.

Unghiurile triunghiului echilateral. Teoremă. Un triunghi este echilateral dacă şi numai dacă are un-ghiurile de 60°.Unghiurile triunghiului dreptunghic. Teoremă. Un triunghi este dreptunghic dacă şi numai dacă are două unghiuri complementare.Unghiurile triunghiului dreptunghic isoscel. Teoremă. Un triunghi este dreptunghic isoscel dacă şi nu-mai dacă are două unghiuri de 45°.

A

B C

1

2 23 1

M

N P

1

2

2

3

31

2 312 3

¼ Congruenţa triunghiurilor dreptunghice (2)

Criteriul IU. Dacă două triunghiuri dreptunghice au ipotenuzele şi un unghiascuţit respectiv congruente, atunci ele sînt congruente. (IU de la Ipotenuză-Unghi ascuţit!)Teorema se demonstrează aplicînd proprietatea unghiurilor triunghiului dreptunghic şi apoi criteriul ULU.Criteriul IC. Dacă două triunghiuri dreptunghice au ipotenuzele şi o catetă respectiv congruente, atunci ele sînt congruente. (IC de la Ipotenuză-Catetă!)

Ipoteza. Triunghiu-rile ABC şi DEF sînt dreptunghice.[BC] ≡ [EF],[AC] ≡ [DF].Concluzia. DABC ≡ DDEF.

Înălţimile triunghiului isoscel. Teoremă. Un triunghi este isoscel dacă şi numai dacă are două înălţimi congruente.Teorema directăIpoteza. Triunghiul ABC, [AB] ≡ [AC],[BD] şi [CE] înălţimi.Concluzia. [BD] ≡ [CE].

Teorema reciprocăIpoteza. Triunghiul ABC, [BD] ≡ [CE],[BD] şi [CE] înălţimi.Concluzia. [AB] ≡ [AC].

D E

F

A B

C

Demonstraţie. Se construiesc triunghiurile AMB şi DEN, A este mijlocul [CM], D este mijlocul [FN]. Conform CC, [AB] ≡ [AB] (catetă comună), [AC] ≡ [AC] (construcţie) implică DABC ≡ DABM, de unde [BC] ≡ [BM], (1). Ana-log se demonstrează [FE] ≡ [NE], (2). Ipoteza, (1) şi (2) implică [BC] ≡ [BM] ≡ [FE] ≡ [NE], (3). Conform LLL, ipoteza şi (3) implică DCMB ≡ DFNE, de unde ÐC ≡ ÐF, (4). Conform IU, ipoteza şi (3) implică DABC ≡ DDEF, c.c.t.d.

D E

F

A B

C

M N

A

B CDE

Demonstraţie. Conform proprietăţii triunghiului isoscel, ÐBCD ≡ ÐCBE, (1). Conform IU, ipoteza şi (1) implică DBCD ≡ DCBE, c.c.t.d.

A

B CDE

Demonstraţie. Conform IC, ipoteza implică DBCD ≡ DCBE, (1). (1) implică ÐBCD ≡ ÐCBE, (2).(2) implică [AB] ≡ [AC], c.c.t.d.

1

2 3

115Geometrie. Cap. 6. Paralelism

E x e r c i ţ i i 1. Completaţi propoziţiile:

a) Măsura unui unghi exterior al triunghiului este egală cu ......................................................................................................................... ........................b) Suma măsurilor unghiurilor unui triunghi este ........c) Un triunghi este echilateral dacă şi numai dacă are unghiurile de .........

2. Împăturiţi o hîrtie pe care desenaţi triunghiul oarec-are ABC, astfel încît să arătaţi că suma măsurilor ung-hiurilor lui este 180°.

3. Fie x, y, z măsurile un- A

B C

x

y z

ghiurilor triunghiului ABC (v. desenul). Aflaţi: a) x, dacă y = 37°, z = 78°;b) y, dacă x = 89°, z = 65°;c) z, dacă x = 75°, y = 39°;d) x, dacă y = 72°, z = 45°.4. Triunghiul isoscel A

B Cx

y

x

ABC are baza BC. Aflaţi măsura unghiu-lui A, dacă:

a) x = 55°;b) x = 26°;c) x = 71°;d) x = 80°.

5. Triunghiul isoscel A

B Cx

y

x

ABC are baza BC.Aflaţi măsura unghiu-rilor congruente, dacă:

a) y = 42°; b) y = 38°;c) y = 76°;d) y = 102°.

6. Completaţi propoziţiile:a) Un triunghi este dreptunghic dacă şi numai dacă are ............................................b) Un triunghi este dreptunghic isoscel dacă şi nu-mai dacă are .....................................

7. Triunghiul dreptun-

A

B

C

y

x

ghic ABC are baza BCşi măsurile ascuţite x, y (v. desenul). Aflaţi:

a) y, dacă x = 33°; b) x, dacă y = 23°;c) y, dacă x = 73°;d) x, dacă y = 41°.

8. Completaţi propoziţiile:a) (IU) Dacă două triunghiuri dreptunghice au ipo-tenuzele şi un unghi ascuţit respectiv congruente, atunci ............................................b) (IC) Dacă două triunghiuri dreptunghice au ipo-tenuzele şi o catetă respectiv congruente, atunci ............................................c) (Înălţimile triunghiului isoscel) Un triunghi este isoscel dacă şi numai dacă are ..............................

9. Triunghiul isoscel ABC are baza BC şi înălţimile BD, CE (v. desenul). Aflaţi: A

B CDE

a) BD, dacă CE = 5,8 cm;b) CE, dacă BD = 27 mm;c) BD, dacă CE = 3,6 cm;d) CE, dacă BD = 48 mm.

10. Triunghiul ABC are înălţimile BD şi CE (v. de-senul). Aflaţi: A

B CDE

a) AB, dacă AC = 2,7 cm;b) AC, dacă AB = 78 mm;c) AB, dacă AC = 6,2 cm;d) AC, dacă AB = 62 mm.

11. Construiţi triunghiul isoscel ABC cu un unghi de 101° şi o latură de 34 mm. Cîte astfel de triunghiuri necongruente există?12. Construiţi triunghiul dreptunghic isoscel ABC cu ipotenuza BC, dacă AC = 3,8 cm.13. Construiţi triunghiul dreptunghic isoscel ABC cu

ipotenuza BC, dacă BC = 28 mm.14. Construiţi triunghiul dreptunghic ABC cu ipote-nuza BC, dacă BC = 4,6 cm şi mÐB = 75°.15. Construiţi triunghiul dreptunghic ABC cu ipote-nuza BC, dacă BC = 5 cm şi AC = 3,1 cm.

16. Aflaţi măsurile unghiurilor ascuţite ale unui tri-unghi dreptunghic, dacă ele sînt:

a) direct proporţionale cu 2 şi 3;b) invers proporţiuonale cu 3 şi 7.

Formulaţi un exerciţiu asemănător pentru un triunghi

oarecare.17. Construiţi triunghiul isoscel ABC cu un unghi de 35° şi o latură de 68 mm. Cîte astfel de triunghiuri necongruente există?

116 Geometrie. Cap. 6. Paralelism

½ Proprietăţi ale triunghiurilor dreptunghice

l Punctele M şi N sînt simetrice faţă de punctul O, dacă O este mij-locul segmentului MN.Mediana din vîrful unghiului drept. Teoremă. Un triunghi este dreptunghic dacă şi numai dacă lungimea unei mediane este jumătate din lungimea laturii corespunzătoare acelei mediane.Teorema directăIpoteza. Triunghiul ABCeste dreptunghic în A,[CD] ≡ [DB].Concluzia. AD = 0,5BC.

Teorema reciprocăIpoteza. Triunghiul ABC,D mijlocul [BC],[AD] ≡ [BD] ≡ [DC].Concluzia. ÐCAB estedrept.

l Punctele A şi B sînt simetrice faţă de dreapta d, dacă d este mediatoarea segmentului AB.Reciproca teoremei mediatoarei. Dacă un punct al planului este egal depărtat de capetele unui segment, atunci punctul aparţine mediatoarei segmentului.Ipoteza. Desenul şi relaţi-ile marcate.Concluzia. d ^ AB.

Proprietatea Mediatoarei. Teoremă. Mediatoarea unui segment este mulţimea punctelor unui plan, egal depărtate de capetele segmentului.Proprietăţile mediatoarei bazei. Teoremă. Un triunghi este isoscel dacă şi numai dacă are o axă de simetrie. Axa de simetrie a triunghiului isoscel este mediatoarea bazei.l Cercul ce conţine vîrfurile unui triunghi este cercul circumscris triunghiului.Intersecţia mediatoarelor. Teoremă. Intersecţia mediatoarelor unui triunghi este centrul cer-cului circumscris triunghiului.Proprietatea mijlocului ipotenuzei. Teoremă. Un triunghi este dreptunghic dacă şinumai dacă centrul cercului circumscris lui aparţine unei laturi.Teorema triunghiului dreptunghic cu un unghi de 30°. Un triunghi dreptunghic are un unghi de 30° dacă şi numai dacă lungimea unei catete este jumătate din lungimea ipotenuzei.

Proprietatea bisectoarei. Teoremă. Un punct al interioru-lui unui unghi aparţine bisectoarei unghiului dacă şi numai dacă este egal depărtat de laturile unghiului.

A B

C

D

E Demonstraţie. Se construieşte triunghiul CDE astfel încît A şi E sînt simetrice faţă de punctul D, (1). Conform LUL, ipoteza, (1) şi ÐCDE ≡ ÐBDA implică DCDE ≡ DBDA, de unde ÐECD ≡ ÐABD, (2), ÐCED ≡ ÐDAB, (3), [CE] ≡ [AB], (4). BC este secantă dreptelor CE şi AB, (5). Conform Teoremei Paralelelor (TP), (2) şi (5) implică CE || AB, (6). Conform TP, (6) implică CE ^ AC, (7). Conform CU, ipoteza, (4) şi (7) implică DCEA ≡ DABC, de unde [BC] ≡ [AE], (8). Ipoteza şi (8) implică AD = 0,5BC, c.c.t.d.

M NO

A B

C

D

Demonstraţie. Conform Teoremei Triunghiului Isoscel, ipoteza implică mÐDAB = mÐABD = x şi mÐCAD = mÐACD = y, (1).Conform Sumei Măsurilor Unghiurilor Triunghiului, (1) implică 2x + 2y = 180° sau x + y = 90°, (2). Conform Adunării Unghiurilor, (2) implică ÐCAB este drept, c.c.t.d.

A B

P

M

d

Demonstraţie. Ipoteza implică [PM] este mediana cores-punzătoare bazei triunghiului isoscel PAB, (1). Conform Teoremei Corespunzătoare Bazei Triunghiului Isoscel, d ^ AB, c.c.t.d.

A Bd

A B

CM

Sugestii de demonstrare. Dacă [AM este bisectoare, se aplică IU şi rezultă [CM] ≡ [MB]. Dacă [CM] ≡ [MB], se aplică IC şi rezultă [AM este bisectoare.

A B

C

D

30°

117Geometrie. Cap. 6. Paralelism

E x e r c i ţ i i 1. Completaţi propoziţia:Punctele A şi B sînt simetrice faţă de punctul M, dacă M este mijlocul ........................2. Construiţi simetricul:

AB

CD

E

F

HG

a) punctului A faţă de punctul B;b) punctului C faţă de punctul D;c) punctului E faţă de punctul F;d) punctului G faţă de punctul H.

3. Completaţi propoziţia:Un triunghi este dreptunghic dacă şi numai dacă lungimea unei mediane este jumătate din lungimea laturii ............................................................4. Triunghiul dreptun-

A

B

C

D

ghic ABC are baza BCşi mediana AD. Aflaţi: a) AD, dacă BC = 72 mm;b) BC, dacă AD = 3,2 cm;c) AD, dacă BC = 68 mm;d) BC, dacă AD = 4,2 cm.5. Completaţi propoziţia:Un triunghi este dreptunghic dacă şi numai dacă lungimea .............................................6. Completaţi propoziţia:Punctele M şi N sînt simetrice faţă de dreapta a, dacă a este mediatoarea ...........................................7. Construiţi axa de simetrie:

A B

C D

E

F

H

G

a) a segmentului AB;

b) a segmentului CD;c) a segmentului EF; d) a segmentului GH.

8. Completaţi propoziţiile:a) (Mediatoarea) Dacă un punct al planului este egal depărtat de capetele unui segment, atunci punctul aparţine ............................................b) Mediatoarea unui segment este mulţimea punc-telor unui plan, egal ............................................c) Cercul ce conţine vîrfurile unui triunghi este ..............................d) Intersecţia mediatoarelor unui triunghi este centrul cercului ...............................................e) Un triunghi este dreptunghic dacă şi numai dacă centrul cercului circumscris lui .............................f) Un triunghi dreptunghic are un unghi de 30° dacă şi numai dacă lungimea unei catete este .......................................................................................

9. Construiţi triunghiul dreptunghic ABC cu ipote-nuza BC şi mediana AD (raza cercului circumscris), dacă:

a) AD = 3 cm şi BC = 4 cm;b) AD = 23 mm şi BC = 17 mm;c) AD = 3,4 cm şi BC = 5,1 cm;d) AD = 4,2 cm şi BC = 3,5 cm.

10. Construiţi triunghiul dreptunghic ABC cu ipote-nuza BC şi mÐB = 30°, dacă:

a) AC = 3,4 cm; b) AC = 43 mm;c) AC = 4,5 cm; d) AC = 48 mm.

11. Construiţi C (O, R), triunghiul oarecare (scalen) ABC înscris în el (vîrfurile sînt puncte ale cercului) şi perpendicularele prin O pe laturile triunghiului. Executaţi construcţia pentru fiecare valoare a lui R şi precizaţi ce sînt perpendicularele prin O.

a) R = 5 cm;b) R = 39 mm;c) R = 6,2 cm;d) R = 45 mm.

12. Construiţi C (I, r), triunghiul oarecare (scalen) ABC cu laturile tangente acestui cerc şi semidreptele cu originea în cîte un vîrf al triunghiului, care conţin I. Executaţi construcţia pentru fiecare valoare a lui r şi precizaţi ce sînt semidreptele pentru triunghi.

a) r = 2 cm; b) r = 23 mm;c) r = 2,6 cm; d) r = 28 mm.

118 Geometrie. Cap. 6. Paralelism

¾ Linia mijlocie a triunghiului

l Un segment determinat de mijloacele a două laturi ale unui triunghieste o linie mijlocie a triunghiului.Teorema liniei mijlocii (1). O linie mijlocie a unui triunghi este paralelă cu una dintre laturile triunghiului şi lungimea ei este jumătate din lungimea acelei laturi.

Demonstraţie. Fie triunghiul NLC, astfel încît L este simetricul lui P faţă de N, (1). Conform LUL, [AN] ≡ [NC], [PN] ≡ [NL], ÐANP ≡ ÐCNL (opuse la vîrf) implică DANP ≡ DCNL, de unde [AP] ≡ [LC], (2), ÐPAN ≡ ÐNCL, (3). Ipoteza şi (2) implică [PB] ≡ [LC], (4). (3) implică AB || LC, de unde ÐPBC ≡ ÐPCL (alterne interne cu secanta PC), (5). Conform LUL, (4), (5) şi [PC] latură comună implică DPBC ≡ DCLP, de unde [BC] ≡ [LP], (6),

ÐPCB ≡ ÐLPC, (7). (7) implică PN || BC, iar (1) şi (6) implică PN (lungimea liniei mijlocii) = 0,5BC, c.c.t.d.Teorema liniei mijlocii (2). Un segment cu o extremitate mijlocul unei laturi a unui triunghi şi cealaltă ex-tremitate aparţinînd altei laturi a triunghiului este linie mijlocie a acestuia dacă şi numai dacă este paralelă cu latura a treia a triunghiului.Observaţie. Segmentele paralele sînt conţinute de drepte paralele.Concurenţa înălţimilor triunghiului. Teoremă. Înălţimile unui triunghi sau dreptele ce le conţin sînt con-curente în punctul numit ortocentrul triunghiului. Observaţii. Ortocentrul triunghiului se notează cu H (cînd această literă nu este ocupată). În cazul triunghiu-lui dreptunghic teorema este evidentă, deoarece ortocentrul este vîrful unghiului drept.

Demonstraţie. Se construiesc: prin A, PN || BC, (1); prin B, PM || AC, (2); prin C, MN || AB. (1) implică ÐBAC ≡ ÐPBA (alterne interne cu secanta AB), (3). (2) implică ÐABC ≡ ÐPAB (alterne interne cu secanta AB), (4). Conform ULU, [AB] latură comună, (3) şi (4) implică DABC ≡ DBAP, de unde [AP] ≡ [BC], (5). La fel se obţine, [AN] ≡ [BC], (6). (5) şi (6) implică [AP] ≡ [AN], (7). (1) şi ipoteza implică AD ^ PN, (8). (7) şi (8) implică AD este mediatoare a triunghiului PMN, (9). În acelaşi mod se demonstrează că celelalte mediatoare ale triunghiului PMN sînt BE şi CF, (10). Conform proprietăţii mediatoarelor triunghiului, (9) şi (10) implică: dreptele AD, BE şi CF sînt concurente, c.c.t.d.

Concurenţa medianelor triunghiului. Teoremă. Medianele triunghiului sînt concurente în centrul de greutate al triunghiului şi centrul de greutate se află pe fiecare mediană de două ori mai aproape de mij-locul laturii decît de vîrful triunghiului opus acelei laturi.

Demonstraţie. Se construiesc: G intersecţia medianelor [AD] şi [BE]; linia mijlocie DE; ND || CG || ME; [MN]; F, intersecţia [CG cu AB, (1). Conform Teoremei liniei mijlocii 2 (TLM2), (1) implică [MN] este linie mijlocie a tri-unghiului ABG, (2). Conform TLM2, (2) implică [MP] este linie mijlocie a triunghiului AFG, (3). Conform TLM1, (1) şi (2) implică DE || MN şi [DE] ≡ [MN], (4). Conform ULU, (4) implică DDEG ≡ DMNG, de unde G este mijlocul [MD] şi [NE], (5). Conform TLM2, (1) şi (5) implică [PG] este linie mijlocie a triunghiului MND, (6). Conform LUL, (3) şi (6) implică DPNG ≡ DPMF, de unde MF || BG, (7). Conform TLM2, (1) şi (7) implică [FM] este o

linie mijlocie a triunghiului ABG, de unde [CF] este a treia mediană a triunghiului ABC. (1) şi (5) implică AG = 2GD şi BG = 2GE. În acelaşi mod se demonstrează că CG = 2GF, c.c.t.d.Triunghiul echilateral. Teoremă. Medianele, bisectoarele, înălţimile şi mediatoarele triunghiului echila-teral sînt concurente în centrul cercului circumscris triunghiului.

A

BM

NP

CA

B

NP

C

L

A

B CD

EF

M

NP

A

B CD

EFG

M

NP

119Geometrie. Cap. 6. Paralelism

E x e r c i ţ i i 1. Completaţi propoziţia:Un segment determinat de mijloacele a două laturi ale unui triunghi este ........................2. Construiţi liniile mijlocii ale triunghiurilor:

A

B C

D

E

F

H

G

I

3. Recunoaşteţi în situaţiile ilustrate liniile mijlocii:

D

A

B C E

F

H

G

I

M NP

QR

Q

4. Completaţi propoziţia:O linie mijlocie a unui triunghi este ................ cu una dintre laturile triunghiului şi lungimea ei este ................................... acelei laturi.5. Triunghiul ABC are mijloace-

A

B

C

P

M

N

le laturilor M, N, P. Aflaţi lungi-mile laturilor triunghiului MNP,dacă:a) AB = 3,4 cm, BC = 36 mm,AC = 24 mm;b) AB = 7 cm, BC = 82 mm,AC = 42 mm;c) AB = 4,2 cm, BC = 48 mm,AC = 27 mm.6. Triunghiul ABC are mijloacele laturilor M, N, P. Aflaţi lungimile laturilor triunghiului ABC, dacă:

a) MN = 1,3 cm, NP = 15 mm, MP = 21 mm;b) MN = 1,6 cm, NP = 21 mm, MP = 17 mm;c) MN = 2,5 cm, NP = 31 mm, MP = 19 mm; d) MN = 2,9 cm, NP = 34 mm, MP = 43 mm.

7. Triunghiul ABC are mijloacele laturilor M, N, P. Aflaţi perimetrul triunghiului MNP, dacă:

a) PABC = 7,8 cm; b) PABC = 10,5 cm;

c) PABC = 104 mm; d) PABC = 16,4 mm.8. Triunghiul ABC are mijloacele laturilor M, N, P. Aflaţi perimetrul triunghiului ABC, dacă:

a) PMNP = 2,9 cm; b) PMNP = 37 mm;c) PMNP = 5,3 cm; b) PMNP = 45 mm.

9. Desenul ilustrează liniile mijlocii ale triunghiurilor ABC, DEF, HGI. Completaţi:

D

A

B C E

F

H

G

I

M PS

R U

V

NQ

T

a) MN || AC, ............., ...............;b) .............., ............., ...............;c) .............., ............., ...............

10. Triunghiul ABC are mijloa-

A

B

C

N

M

cele laturilor AB şi AC, respec-tiv M şi N. Aflaţi măsurile un-ghiurilor M şi N ale triunghiu-lui AMN, dacă:a) mÐB = 38°, mÐC = 29°;b) mÐB = 109°, mÐC = 21°;c) mÐB = 57°, mÐC = 93°.11. Triunghiul ABC are mijloa-

A

B

C

N

M

cele laturilor AB şi AC, respec-tiv M şi N. Aflaţi măsurile un-ghiurilor B şi C ale triunghiu-lui ABC, dacă:a) mÐM = 76°, mÐN = 33°;b) mÐM = 22°, mÐN = 134°;c) mÐM = 59°, mÐN = 41°.12. Triunghiul ABC are mijloa-

A

B

C

P

M

N

cele laturilor M, N, P. Comple-taţi congruenţele de triunghiurica în model:a) [MP] ≡ [BN] (Ipoteza şi o pro-prietate a liniei mijlocii), [NP]≡ [BM] (Ipoteza şi o proprietate a liniei mijlocii) şi [MN] latură

120 Geometrie. Cap. 6. Paralelism

comună implică (LLL) DMNP ≡ DNMB.b) [MN] ≡ [......] (Ipoteza şi o proprietate a liniei mijlocii), [PM] ≡ [.....] (Ipoteza şi o proprietate a liniei mijlocii) şi [.....] latură comună implică (LLL) DMNP ≡ DPAM.c) [MN] ≡ [......] (Ipoteza şi o proprietate a liniei mijlocii), [NP] ≡ [.....] (Ipoteza şi o proprietate a li-niei mijlocii) şi [.....] latură comună implică (LLL) DMNP ≡ DCPN.

13. Completaţi propoziţia:Un segment cu o extremitate mijlocul unei laturi a unui triunghi şi cealaltă extremitate aparţinînd al-tei laturi a triunghiului este linie mijlocie a acestuia dacă şi numai dacă este .......................................... a triunghiului.14. Aplicînd teorema reconstituită la ex. 13, construiţi liniile mijlocii cînd aveţi mijlocul unei laturi a triun-ghiului.

D

A

B C E

F

M

Q

K

H

G

I

TJ

X

L

15. Triunghiul ABC este isoscel cu baza BC, triun-ghiul DEF este echilateral şi triunghiul GHI estedreptunghic cu ipotenuza HI.

D

A

B C

E

F

HG

I

Construiţi liniile mijlocii ale fiecărui triunghi şi stabiliţi ce tip de triunghi formează.16. Completaţi propoziţiile:

a) Înălţimile unui triunghi sau dreptele ce le conţin sînt ......................................................................... ............................................b) Medianele triunghiului sînt concurente în .................................... şi centrul de greutate se află pe fiecare mediană ...........................................................................................................................................................................c) Medianele, bisectoarele, înălţimile şi media-toarele triunghiului echilateral sînt concurente în ..............................................................................

17. În desen sînt ilustrate cercurile circumscrise triun-ghiului ascuţitunghic ABC şi triunghiului obtuzunghic DEF.

DA

B

C

E

F

a) Construiţi mediatoarele triunghiului ABC şi tri-unghiul MNP format de liniile mijlocii ale triun-ghiului ABC. Ce sînt mediatoarele triunghiului ABC pentru triunghiul MNP?b) Construiţi mediatoarele triunghiului DEF şi tri-unghiul QRS format de liniile mijlocii ale triunghi-ului DEF. Ce sînt mediatoarele triunghiului ABC pentru triunghiul QRS?

18. Triunghiul ABC are centrul de greutate G, G1 este centrul de greutate al triunghiului DEF, G2 al triun-ghiului KLM. Verificaţi în fiecare caz proprietatea centrului de greutate:

a) AG = 2GP; b) DG1 = 2G1Q; c) MG2 = 2G2R.

A B

C

D E

F

K

L

MG G1

G2P Q R

121Geometrie. Cap. 6. Paralelism

19. Triunghiul ABC are mijloacele A

B C

M P

N

laturilor M, N, P. Aflaţi perimetrultriunghiului ABC, dacă MN = 3,5 cm şi laturile triunghiului MNP sînt direct proporţionale cu 2, 4, 5.

20. Triunghiul ABC are mijloacele laturilor M, N, P. Aflaţi perimetrul triunghiului MNP, dacă AB = 4 cm şi laturile triunghiului ABC sînt invers proporţionale cu 3, 4, 5.21. Triunghiul ABC are măsurile unghiurilor direct proporţionale cu 4, 5, 6. Aflaţi măsurile unghiurilor triunghiului ABC.

22. Triunghiul dreptunghic ABC

A

B

C

are măsurile unghiurilor ascuţitedirect proporţionale cu 7 şi 8.Aflaţi măsurile unghiurilor ascu-ţite ale triunghiului ABC.

23. Triunghiul dreptunghic ABC

A

B

CM

NPare ipotenuza BC şi mijloacelelaturilor M, N, P. Demonstraţi că triunghiul MNP este dreptunghic.

24. Triunghiul echilateral ABC

A

B

CM

NP

şi mijloacele laturilor M, N, P. Demonstraţi că triunghiul MNP este echilateral.

25. Triunghiul isoscel ABC

A

B

CM

NPşi mijloacele laturilor M, N,P. Demonstraţi că triunghiulMNP este isoscel.

26. Triunghiul dreptunghic ABC

A

B

C

are măsurile unghiurilor ascuţiteinvers proporţionale cu 4 şi 11.Aflaţi măsurile unghiurilor ascu-ţite ale triunghiului ABC.

27. Triunghiul ABC are mijloacele laturilor M, N, P. Aflaţi perimetrul triunghiului MNP, dacă AB = 6 cm şi laturile triunghiului MNP sînt invers proporţionale cu 4, 5, 6.28. Triunghiul ABC are centrul A

B C

D EG

de greutate G şi medianele con-gruente BD şi CE.

a) Demonstraţi că triunghiul GBC este isoscel.b) Demonstraţi că DBCE ≡ DCBD.c) Demonstraţi că triunghiulABC este isoscel.

29. Triunghiul ABC are mijloacele laturilor M, N, P. Aflaţi perimetrul triunghiului MNP, dacă AB = 6 cm şi laturile triunghiului MNP sînt invers proporţionale cu 4, 5, 6.

30. Două dintre laturile unui triunghi isoscel are lungimile 6 cm şi 5 cm. Aflaţi perimetrul triunghiului format de liniile mijlocii. Studiaţi toate cazurile po-sibile.31. Perimetrul unui triunghi

A

B

CM

NP

isoscel este de 18 cm şi lun-gimea unei laturi este 8 cm.Aflaţi perimetrul triunghiu-lui format de liniile mijlociişi lungimile laturilor acestui triunghi.Studiaţi toate cazurile posibile.32. Triunghiul isoscel DEF are măsurile a două un-ghiuri direct proporţionale cu numerele 2 şi 8. Aflaţi măsurile unghiurilor triunghiului studiind toate ca-zurile posibile. 33. Triunghiul isoscel IJK are măsurile a două un-ghiuri invers proporţionale cu numerele 1 şi 7. Aflaţi măsurile unghiurilor triunghiului studiind toate ca-zurile posibile.34. Construiţi triunghiul dreptunghic cu un unghi de 30°, dacă mediana din vîrful unghiului drept are lungimea 47 mm.

122 Geometrie. Cap. 6. Paralelism

¿ Inegalităţi între elementele triunghiului

Teorema inegalităţilor între elementele triunghiului. O latură a unui triunghi este mai mică decît altă latură a triunghiului dacă şi numai dacă unghiul opus ei este mai mic decît unghiul opus celeilalte laturi.

Teorema directă (Necesitatea) Ipoteza. AB < AC. Concluzia. ÐACB < ÐABC.Demonstraţie. Fie M intersecţia [AC cu C (A, AB). Atunci AMB este triunghi isos-cel cu baza MB, (1). Conform teoremei triunghiului isoscel, (1) implică mÐABM = mÐAMB = x, (2). (1) implică M Î IntÐABC, de unde ÐABM < ÐABC, (3). Conform teoremei unghiului exterior, ÐACB < ÐAMB, (4). (3) şi (4) implică ÐACB < ÐABC, c.c.t.d.

Teorema reciprocă (Suficienţa) Ipoteza. ÐACB < ÐABC. Concluzia. AB < AC.Demonstraţie. Fie M intersecţia [AC cu C (A, AB). Atunci AMB este triunghi isoscel cu baza MB, (1). Con-form teoremei triunghiului isoscel, (1) implică mÐABM = mÐAMB = x, (2). (2) implică x = 0,5(180° − mÐBAC), (3). Suma măsurilor unghiurilor unui triunghi, proprietatea mediei aritmetice şi ipoteza implică:180° − mÐBAC = mÐABC + mÐACB = 2mÐACB, (4). (3) şi (4) implică x > mÐACB, (5). (5) implică: M Î IntÐABC, (6). (1) şi (6) implică AB < AC, c.c.t.d.l Perpendiculara dintr-un punct pe o dreaptă este segmentul de-terminat de punct şi proiecţia lui pe dreaptă. Oblică este oricare alt segment determinat de punctul dat şi un punct al dreptei.Corolar. 1) Ipotenuza unui triunghi dreptunghic este mai lungă decît oricare dintre catetele lui.2) Perpendiculara coborîtă dintr-un punct pe o dreaptă este mai scurtă decît oricare dintre oblicele coborîte din acel punct pe aceeaşi dreaptă.3) Latura opusă unghiului obtuz este cea mai lungă latură a triunghiului obtuzunghic.l Distanţa de la un punct la o dreaptă este egală cu lungimea perpendicularei coborîte din acel punct pe dreaptă.l Distanţa dintre două drepte paralele este egală cu lungimea perpendicularei coborîte dintr-un punct oarecare al unei drepte pe cealaltă dreaptă.Inegalitatea triunghiului. Teoremă. Lungimea unei laturi a unui triunghi este mai mică decît suma lungimilor celorlalte două laturi ale triunghiului.

Ipoteza. Triunghiul ABC din desen. Concluzia. BC < AB + AC.Demonstraţie. Fie M intersecţia [BA cu C (A, AC). Atunci AMC este triunghi isoscel cu baza MC, (1). Conform teoremei triunghi-ului isoscel, (1) implică mÐACM = mÐAMC = x, (2). (1) implică A Î IntÐBCM, de unde ÐBMC < ÐBCM, (3). Conform teoremei inegalităţilor între elementele triunghiului, (3) implică: BC < BM, (4). (1) şi (4) implică BC < AB + AC, c.c.t.d.

Teoremă. Triunghiul dreptunghic în A are înălţimea AD, bisectoarea AE şi mediana AM. Atunci E este între D şi M (D−E−M).Inegalitatea liniilor poligonale cu aceleaşi capete. Teoremă. 1) Fie patrulaterul ABCD ca în desen. Atunci BC + CD < AB + AD.2) Fie situaţia ilustrată în desen. Atunci EK + KL + LJ < EF + FG + GH + HI + IJ.

A

B

CM

x

x

Perp

endi

cula

OblicăOblică

d

M

PProiecţia punctului M pe dreapta d, prdM.

AB

C

M

x

x

A

B

C

D1)

E

FHG

IK L

J2)

Distanţa dintre drep-tele paralele a şi b

a

b

123Geometrie. Cap. 6. Paralelism

E x e r c i ţ i i 1. Completaţi propoziţiile:

a) Un unghi exterior unui triunghi este mai mare decît oricare dintre unghiurile triunghiului ............. acestui unghi.b) O latură a unui triunghi este mai mică decît altă latură a triunghiului dacă şi numai dacă un-ghiul opus ei este ................................................... celeilalte laturi.

2. Ordonaţi măsurile unghiurilor fiecărui triunghi:A

B C

D

E

F

H

G

I

3. Completaţi propoziţia:Ipotenuza unui triunghi dreptunghic este .................decît oricare dintre catetele lui.4. Recunoaşteţi cea mai lungă dintre laturile fiecărui triunghi:

D

A

B C E

F

H

G

I

5. Completaţi propoziţia:Latura opusă unghiului obtuz este ...................... latură a triunghiului obtuzunghic.6. Recunoaşteţi cea mai lungă dintre laturile fiecărui triunghi:

D

A

BC E

F

H

G

I

7. Completaţi propoziţiile:a) Perpendiculara dintr-un punct pe o dreaptă este segmentul determinat de punct şi ..........................b) Oblică este oricare alt segment determinat de punctul dat şi ...................... al dreptei.

8. Construiţi proiecţiile punctelor pe dreapta d:

DA

B

C

E

d

9. Comparaţi perpendiculara cu o oblică din fiecare punct:

FH D

AB C

Md

N P

10. Completaţi propoziţia:Distanţa de la un punct la o dreaptă este egală cu lungimea perpendicularei coborîte .............................11. Examinaţi desenul ex. 8 şi recunoaşteţi distanţa de la fiecare punct la dreapta d.12. Completaţi propoziţia:Distanţa dintre două drepte paralele este egală cu ............................ coborîte dintr-un punct oarecare al uneia dintre ele pe cealaltă dreaptă.13. Executaţi construcţia care permite măsurarea distanţei dintre două drepte paralele.

a b c d m

n

124 Geometrie. Cap. 6. Paralelism

14. Completaţi Inegalitatea triunghiului:Lungimea unei laturi a unui triunghi este mai mică decît ..........................................................................15. Scrieţi inegalitatea triunghiului pentru două dintre laturile triunghiurilor ilustrate.

K

H

G

I

J

L

16. Construiţi bisectoarea, mediana şi înălţimea din vîrful unghiului drept. Constataţi care este ordinea pe ipotenuză a capetelor celor trei segmente.

D

A

B C E

F

H

G

I

17. Completaţi propoziţia:Triunghiul dreptunghic în A are înălţimea AN, bisec-toarea AP şi mediana AM. Atunci ..... este între ... şi ..... (.....−....−.....).18. Comparaţi prin măsurare liniile poligonale ce au aceleaşi capete:

D

A

BC

E

F

H

G

I

J

K

L

19. Comparaţi prin măsurare liniile poligonale ce au aceleaşi capete:

D

A

B

C E

F H

G

I

J

K

L

M

NP

Q

R

S

TU

20. Aflaţi măsurile unghiurilor unui triunghi isoscel, dacă bisectoarele unghiurilor congruente formează un unghi de 106°.21. Aflaţi măsurile unghiurilor unui triunghi isoscel, dacă înălţimile corespunzătoare laturilor congruente formează un unghi de 112°.22. Construiţi un triunghi isoscel cu înălţimile con-gruente de 36 mm şi baza de 43 mm.23. Construiţi un triunghi isoscel cu medianele con-gruente de 42 mm şi baza de 39 mm.

24. Construiţi un triunghi isoscel cu medianele con-gruente de 60 mm şi unghiul format de ele drept.25. Construiţi un triunghi isoscel cu o latură de 15 mm şi alta de 30 mm.26. Construiţi un triunghi dreptunghic cu mediana din vîrful unghiului drept de 3 cm, dacă această mediană formează cu o catetă un unghi de 37°.27. Două unghiuri ale unui triunghi au 46° şi 86°. Aflaţi măsurile unghiurilor formate de cîte două dintre bisectoare.

28. Construiţi un triunghi isoscel cu o latură de 23 mm şi un unghi de 108°.29. Construiţi un triunghi isoscel cu bisectoarele con-gruente de 28 mm şi unul dintre unghiurile congru-ente de 54°.30. Construiţi un triunghi isoscel cu bisectoarele con-gruente de 25 mm şi un unghi de 48°. Cîte astfel de triunghiuri necongruente există?31. Construiţi un triunghi isoscel cu înălţimile con-gruente de 31 mm şi un unghi de 56°. Cîte astfel de triunghiuri necongruente există?

32. Construiţi un triunghi cu lungimile laturilor în centimetri, dacă două dintre laturi au lungimile, res-pectiv, de 1 cm şi 4 cm. Cîte astfel de triunghiuri necongruente există?. Cîte astfel de triunghiuri necon-gruente există?33. Triunghiul dreptunghic ABC are ipotenuza BC şi un unghi de 15°. Demonstraţi că înăţimea AD este 0,25 BC.

A B

DC

125Geometrie. Cap. 6. Paralelism

E VA L U A R EI I I

1. Unghiurile M1 şi P1 forma- M

P

11

ab

te de dreptele a şi b cu secan-ta MP sînt unghiuri ................................................................

2. Construiţi prin punctele A şi B dreptele paralele cu dreapta d.

AB

d

3. Recunoaşteţi dreptele paralele:a

b57°

57° 43,1°

43° c

d

4. Examinaţi desenul şi aflaţi măsurile x şi y ştiind că a || b.

x

52° c

d

A

B

y

5. Triunghiul ABC are unghiul A de 49° şi unghiul B de 74°. Ce măsură are unghiul C?6. Triunghiul isoscel ABC cu baza BC are unghiul A de 84° şi perimetrul de 12 cm. Aflaţi măsurile ce-lorlalte unghiuri şi perimetrul triunghiului format de liniile mijlocii ale triunghiului. 7. Construiţi triunghiul dreptunghic ABC cu ipote-nuza BC = 5,2 cm şi unghiul B de 32°.8. Fie punctele A, B, C. Stabiliţi dacă punctele A, B, C sînt coliniare, dacă:

a) AB = 10 cm, BC = 8 cm, AC = 12 cm;b) AB = 15 cm, BC = 12 cm, AC = 27 cm.

9. Construiţi triunghiul dreptunghic ABC cu ipo-tenuza BC, unghiul B de 30° şi mediana AD = 3,6 cm.10. Construiţi:

a) triunghiul dreptunghic ABC cu ipotenuza BC = 8,6 cm şi unghiul B de 30°;b) triunghiul isoscel ABC cu baza BC = 3,5 cm şi una dintre înălţimile congruente de 2,2 cm.

1. Unghiurile K1 şi L1 forma- K

L

1

1

ab

te de dreptele a şi b cu secan-ta KL sînt unghiuri ................................................................

2. Construiţi prin punctele D şi F dreptele paralele cu dreapta a.

D

F

d

3. Recunoaşteţi dreptele paralele:m

n 121,2°

121°

40°

40° e

f

4. Examinaţi desenul şi aflaţi măsurile x şi y ştiind că a || b.

x50°

a

b

A

B

y

5. Triunghiul ABC are unghiul A de 43° şi unghiul B de 82°. Ce măsură are unghiul C?6. Triunghiul isoscel ABC cu baza BC are unghiul A de 78° şi perimetrul de 14 cm. Aflaţi măsurile ce-lorlalte unghiuri şi perimetrul triunghiului format de liniile mijlocii ale triunghiului. 7. Construiţi triunghiul dreptunghic ABC cu ipote-nuza BC = 4,7 cm şi unghiul B de 53°.8. Fie punctele A, B, C. Stabiliţi dacă punctele A, B, C sînt coliniare, dacă:

a) AB = 13 cm, BC = 9 cm, AC = 21 cm;b) AB = 14 cm, BC = 11 cm, AC = 21 cm.

9. Construiţi triunghiul dreptunghic ABC cu ipo-tenuza BC, unghiul B de 30° şi mediana AD = 2,7 cm.10. Construiţi:

a) triunghiul dreptunghic ABC cu ipotenuza BC = 6,4 cm şi unghiul B de 30°;b) triunghiul isoscel ABC cu baza BC = 4,6 cm şi una dintre înălţimile congruente de 3,9 cm.

1

1

1

1

1

1

1

1

1

1

126 Patrulatere

Capitolul 7 Patrulatere ¶ Poligoane

¸ Unghiurile patrulaterului

l Poligonul este o linie poligonală (frîntă) închisă fărăautointersecţii.

Suma măsurilor unghiurilor patrulaterului. Teoremă. Suma măsurilor unghiurilor unuipatrulater convex este 360°.Sugestie de demonstraţie. Diagonala BD descompunte Int ABCD în Int ABC şi Int ABD.Se aplică Axioma Adunării Unghiurilor şi Suma Măsurilor Unghiurilor Triunghiului.

· Elementele patrulaterului

A

BC

D

E

F

G

H

I

J

K

L

MN O

P

QR

T

S

U V

X

Y

A1A2

A6

A5

A4 A3

Patrulaterulconvex ABCD

Patrulaterulconcav EFGH

Pentagonulconvex IJKLM

Pentagonulconcav NOPQR

Hexagonulconvex STUVXY

Hexagonul con-cav A1A2A3A4A5A6

Linie poligonalăfără autointersecţii

Linie poligonalăcu autointersecţie

l Poligonul cu: 7 laturi se numeşte heptagon; cu 8 laturi se numeşte octogon; cu 9 laturi se numeşte nonagon; cu 10 laturi se numeşte decagon; cu 12 laturi se numeşte dodecagon; cu n laturi − poligon cu n laturi.l Diagonală a unui poligon este un segment determinat de două vîrfuri necon-secutive ale poligonului.

Vîrfuri ne-consecutive

Vîrfuri conse-cutive

Diagonală

A

BC

D

Int ABCDExt ABCD

Ext ABCD

Ext ABCDExt ABCD

A

BC

DVîrfuri opuse

A

B C

D

Vîrfuri conse-cutive

Vîrfuri conse-

cutive

Vîrf

uri

con-

secu

tive

A

BC

DLaturiopuse

A

B C

D

Laturi con-secutive

Vîrfuri consecutive

Latu

ri co

n-se

cutiv

e

A

BC

D

Diagonale

A

BC

D

Bimediane

N

P

Q

M

A

BC

Dx

yt

z

u

v

¹ Patrulatere particulare

l Paralelogramul este patrulaterul cu laturile opuse paralele. Trapezul este patrulaterul cu o pereche de laturi paralele. Patrulaterul ortodiagonal are diagonalele perpendiculare.

Paralelogram TrapezPatrulater orto-diagonal

A B

CD

E F

GH

I

J

K

L

A Bd

127Patrulatere

E x e r c i ţ i i 1. Completaţi propoziţiile:

a) Poligonul este .......................................... fără autointersecţii.b) Patrulaterul are ..................... laturi.c) Pentagonul are ..................... laturi.d) Hexagonul are ..................... laturi.e) Heptagonul are ..................... laturi.f) Octogonul are ..................... laturi.g) Nonagonul are ..................... laturi.h) Decagonul are ..................... laturi.i) Dodecagonul are ..................... laturi.

2. Recunoaşteţi fiecare poligon:

12

3 4

3. Reprezentaţi:a) pentagonul concav ABCDE; b) heptagonul convex FGHIJKL;c) patrulaterul concav MNPQ;d) hexagonul RSTUVX.

4. Completaţi propoziţia:Diagonală a unui poligon este un segment determinat de ...................................................... ale poligonului.5. Patrulaterul ABCD are:

D E

F

Gvîrfurile ....................;laturile .......................;unghiurile ...................;diagonalele .................;laturile opuse ...............;

unghiurile opuse .................;laturile consecutive ...........................................

6. Completaţi propoziţia:Suma măsurilor unghiurilor unui patrulater convex este ...............................................................7. Aflaţi măsurile necunoscute ale unghiurilor patru-laterului convex:

a) cu unghiurile congruente;b) cu unghiurile două cîte două congruente şi un unghi de 57°;c) trei dintre unghiuri de măsuri 65°, 83°, 76°;d) trei dintre unghiuri de măsuri 43°, 108°, 74°;e) trei dintre unghiuri de măsuri 88°, 15°, 94°.

8. Completaţi propoziţiile:a) Paralelogramul este patrulaterul cu ...........................................................................b) Trapezul este patrulaterul cu ...........................................................................c) Patrulater ortodiagonal este un patrulater cu ...........................................................................

9. Recunoaşteţi fiecare patrulater:

A

B

C

H

I

D

E F

G

J

K

L

10. Completaţi tabelul:

Poligonul Numărul latu-rilor

Numărul dia-gonalelor

PatrulaterHexagonOctogonDecagon

11. Aflaţi măsurile unghiurilor unui patrulater con-vex, dacă ele sînt direct proporţionale cu numerele 3, 5, 7, 9.

12. Aflaţi suma măsurilor unui

A

B C

D

E

pentagon convex cu ajutorul sugestiei din desen.

13. Descoperiţi formula de calcul a numărului diago-nalelor unui poligon cu n laturi. Aflaţi:

a) numărul dreptelor diferite determinate de cîte 2 dintre n puncte, necoliniare oricare 3;b) numărul diagonalelor, diferenţa dintre numărul dreptelor şi numărul laturilor.

14. Aflaţi suma măsurilor A

B

C

D65°unghiurilor patrulaterului concav MNPQ cu ajutorulsugestiei oferite de desen.

128 Patrulatere

» Construcţii geometrice

º Proprietăţile paralelogramului

Laturile opuse ale paralelogramului. Teorema 1. Un patrulater convex este paralelogram dacă şi numai dacă are laturile opuse congruente.Teorema directă (Necesitatea)Ipoteza. Desenul, AB || DC,AD || BC.Concluzia. [AB] ≡ [DC],[AD] ≡ [BC].

Teorema reciprocă (Suficienţa)Ipoteza. Desenul, [AB] ≡ [DC],[AD] ≡ [BC].Concluzia. AB || DC,AD || BC.

Laturile opuse ale paralelogramului. Teorema 2. Un patrulater este paralelogram dacă şi numai dacă are două laturi paralele şi congruente.Unghiurile opuse ale paralelogramului. Teoremă. Un patrulater este paralelogram dacă şi numai dacă are unghiurile opuse congruente.Unghiurile alăturate ale paralelogramului. Teoremă. Un patrulater este paralelogram dacă şi numai dacă are unghiurile consecutive suplementare.Intersecţia diagonalelor paralelogramului. Teorema. Un patrulater este paralelogram dacă şi numai dacă diagonalele lui au acelaşi mijloc (se înjumătăţesc). Intersecţia diagonalelor este centrul lui de simetrie.Teorema directă (Necesitatea)Ipoteza. Desenul, AB || DC,AD || BC.Concluzia. [AO] ≡ [OC],[DO] ≡ [BO].

Teorema reciprocă (Suficienţa)Ipoteza. Desenul, [AO] ≡ [OC], [DO] ≡ [BO].Concluzia. ABCD esteparalelogram.

Demonstraţie. AB || DC (Ipoteza) implică ÐB2 ≡ ÐD2 (alterne interne), (1); AD || BC (Ipoteza) implică ÐD1 ≡ ÐB1 (alterne interne), (2).Conform ULU, [BD] latură comună, (1) şi (2) implică ∆ABD ≡ ∆DBC, de unde [AB] ≡ [DC], [AD] ≡ [BC], c.c.t.d.Demonstraţie. Conform LLL, [BD] latură co-mună, [AB] ≡ [DC] şi [AD] ≡ [BC] implică ∆ABD ≡ ∆DBC, de unde ÐB2 ≡ ÐD2 (1); AD || BC (Ipo-teza) implică ÐD1 ≡ ÐB1 (2). (1) alterne interne implică AB || DC, iar (2) alterne interne implică AD || BC, c.c.t.d.

A B

CD1

12

2

A B

CD1

12

2

A B

CD1

11

1

O

Demonstraţie. AB || DC, AD || BC (Ipoteza) im-plică [AB] ≡ [DC] (proprietate a paralelogramu-lui), (1). AB || DC (Ipoteza) implică ÐA1 ≡ ÐC1 (alterne interne) şi ÐB1 ≡ ÐD1 (alterne interne), (2). Conform ULU, (1) şi (2) implică ∆AOB ≡ ∆DOC, de unde [AO] ≡ [OC], [DO] ≡ [BO], c.c.t.d.Demonstraţie. Conform LUL, [AO] ≡ [OC], [DO] ≡ [BO], ÐAOB ≡ ÐDOC implică ∆AOB ≡ ∆DOC, de unde [AB] ≡ [DC] (1) şi ÐA1 ≡ ÐC1 (alterne interne) implică AB || DC (2). (1) şi (2) implică ABCD paralelogram (proprietate a para-lelogramului), c.c.t.d.A B

CD1

11

1

O

Construiţi paralelogramul: ABCD cu AB, BC şi mÐA date; EFGH cu EG, FH şi mÐEOF date.

AB B

D C

O O

E F

GH

129Patrulatere

E x e r c i ţ i i 1. Completaţi propoziţia:Un patrulater convex este paralelogram dacă şi nu-mai dacă are laturile opuse .....................2. Pentru fiecare paralelogram notaţi că are laturile opuse paralele.

A B

C

HI

D

E F

G

J K

L

3. Controlaţi dacă fiecare dintre paralelogramele de mai sus are laturile opuse congruente.4. Completaţi propoziţia:Un patrulater este paralelogram dacă şi numai dacă are două laturi ................... şi ........................5. Completaţi patrulaterele cu două laturi paralele şi congruente şi controlaţi dacă sînt paralelograme:

A

B

C

H I

DE

F

G

J

K

L

6. Completaţi propoziţia:Un patrulater este paralelogram dacă şi numai dacă are unghiurile opuse ..............................7. Controlaţi dacă paralelogramele din desenul ex. 2 au unghiurile opuse congruente.8. Completaţi propoziţia:Un patrulater este paralelogram dacă şi numai dacă are unghiurile consecutive .........................................9. Controlaţi dacă paralelogramele din desenul ex. 2 au unghiurile consecutive suplementare.10. Aflaţi măsurile celorlalte unghiuri ale paralelo-gramului cu un unghi de:

a) 78°; b) 53°; c) 29°; d) 38°; e) 41°.11. Construiţi paralelogramul ABCD cu:

a) AB = 5,2 cm, BC = 3,5 cm şi mÐB = 27°;b) AB = 6 cm, BC = 2,8 cm şi mÐB = 39°;c) AB = 5,7 cm, BC = 4,3 cm şi mÐB = 62°;d) AB = 4,8 cm, BC = 3,7 cm şi mÐB = 73°;e) AB = 6,4 cm, BC = 5,8 cm şi mÐB = 54°.

12. Completaţi propoziţia:Un patrulater este paralelogram dacă şi numai dacă diagonalele lui ............................................................Intersecţia diagonalelor este ......................................13. Construiţi paralelogramul ABCD ale cărui diagonale se intersectează în O:

a) AC = 4,8 cm, CD = 6,4 cm şi mÐO = 15°;b) AC = 5,6 cm, CD = 4,4 cm şi mÐO = 21°;c) AC = 6,2 cm, CD = 5 cm şi mÐO = 23°;d) AC = 8 cm, CD = 6 cm şi mÐO = 17°;e) AC = 7,2 cm, CD = 5,4 cm şi mÐO = 33°.

14. Prin vîrfurile triunghiului A

B C

D

EFABC se duc paralelele cu latu-rile opuse şi se obţine triun-ghiul DEF. Enumeraţi parale-logramele ce le descoperiţi înconstrucţie şi triunghiurile congruente.

15. Demonstraţi că mijloacele la-

A B

CD

M

N

P

Qturilor patrulaterului convex ABCD sînt vîrfurile unui paralelogram.16. Cercetaţi dacă patrulaterul concavare o proprietate similară celei descrise în ex. 15.

17. ABCD este paralelogram cu intersecţia diagonalelor (centrul de simetrie O) şi A

B C

DO

M

N

P

Q

M–O–P, Q–O–N ca în desen. Demonstraţi:

a) ∆BOM ≡ ∆DOP; b) O este mijlocul [MP];c) MNPQ este paralelo-gram cu centrul O.

18. Descoperiţi formula de calcul a sumei măsurilor

unghiurilor unui poligon convex cu n laturi. Aflaţi: a) numărul triunghiurilor în care se descompune un octogon convex; b) numărul triunghiurilor în care se descompune un poligon convex cu n laturi;c) suma măsurilor unghiurilor a n triunghiuri;d) suma măsurilor unghiurilor poligonului convex cu n laturi.

130 Patrulatere

¼ Proprietăţile dreptunghiuluil Paralelogramul cu un unghi drept se numeşte dreptunghi.l Dreptunghiul are toate proprietăţile paralelogramului.Unghiurile dreptunghiului. Teoremă. Un patrulater este dreptunghi dacă şi numai dacă are trei unghiuri drepte.Teorema directă (Necesitatea)Ipoteza. Desenul, AB || DC,AD || BC, mÐA = 90°.Concluzia. mÐA = mÐD = mÐB = 90°.Teorema reciprocă (Suficienţa)Ipoteza. Desenul, mÐA = mÐD = mÐB = 90°.Concluzia. AB || DC,AD || BC.Diagonalele dreptunghiului. Teoremă. Un paralelogram este dreptunghi dacă şi numai dacă are diago-nalele congruente.Teorema directă (Necesitatea)Ipoteza. Desenul, AB || DC,AD || BC, mÐA = 90°.Concluzia. [AC] ≡ [BD].

Teorema reciprocă (Suficienţa)Ipoteza. Desenul, AB || DC,AD || BC, [AC] ≡ [BD].Concluzia. mÐA = 90°.

Simetria dreptunghiului. Teoremă. Un paralelogram este dreptunghi dacă şi numai dacă me-diatoarea a două laturi opuse este o axă de simetrie.Corolar. Mediatoarele laturilor opuse ale unui dreptunghi sînt axele de simetrie ale dreptun-ghiului. Intersecţia diagonalelor unui dreptunghi este centrul cercului circumscris dreptunghiului.

A B

D C Demonstraţie. Ipoteza implică ABCD are unghiu-rile consecutive suplementare, (1). (1) şi mÐA = 90° implică mÐD = 90°, (2), şi mÐB = 90°, (3).(2) şi (3) implică mÐA = mÐD = mÐB = 90°, c.c.t.d.Demonstraţie. Conform Teoremei Paralelelor, mÐA = mÐD = 90° (ipoteză) implică mÐA + mÐD = 180°, de unde AB || DC. În acelaşi mod se deduce că AD || BC, c.c.t.d.

A B

D C

A B

D C Demonstraţie. Conform teoremei anterioare, ipo-teza implică mÐB = 90°, (1). Ipoteza implică [AD] ≡ [BC], (2). Conform CC, [AB] latură comună, (1) şi (2) implică ∆ABD ≡∆ABC, de unde [AC] ≡ [BD], c.c.t.d.Demonstraţie. Conform ipotezei, [AD] ≡ [BC], (1). Conform LLL, (1), [AC] ≡ [BD] (ipoteză), (1) şi [AB] latură comună implică ∆ABD ≡∆ABC, de unde mÐA = mÐD, (2). mÐA + mÐD = 180° (proprietate a paralelogramului), (3). (1) şi (2) implică mÐA = 90°, c.c.t.d.

A B

D C

l Construiţi un dreptunghi ABCD cu lungimile a două laturi date (m şi n). Rezolvare. Se construiesc: un unghi drept; două laturi consecutive ale dreptunghiului; celelalte laturi.

A B

D C

l Construiţi un dreptunghi ABCD cu lungimea unei diagonale (d) şi a unei laturi date (a). Rezolvare. Se construiesc: cercul de diametru d; o latură a dreptun-ghiului; celelalte lalturi ale lui.

m m m

n n n

A A ABB B

dd d

mmm

131Patrulatere

E x e r c i ţ i i 1. Completaţi propoziţiile:

a) Paralelogramul cu un unghi drept se numeşte ...................b) Dreptunghiul are toate proprietăţile .............

2. Identificaţi dreptunghiurile:

1

2

34

3. Controlaţi dacă fiecare dintre dreptunghiurile de mai sus are trei unghiuri congruente.4. Completaţi propoziţia:Un paralelogram este dreptunghi dacă şi numai dacă are diagonalele ..........................................5. Controlaţi dacă diagonalele dreptunghiurilor sînt congruente: A

B C

H

I

D

E F

G

J

KL

6. Construiţi dreptunghiul ABCD cu:a) AB = 2,4 cm şi BC = 3,1 cm;b) AB = 3,6 cm şi BC = 4,2 cm;c) AB = 6 cm şi BC = 7,2 cm;d) AB = 3,5 cm şi BC = 4,7 cm.

7. Construiţi dreptunghiul ABCD cu centrul O: a) AC = 8 cm, mÐO = 21°;b) AC = 3,8 cm, mÐO = 19°;c) AC = 7,2 cm, mÐO = 32°;d) AC = 4,6 cm, mÐO = 43°.

8. Completaţi propoziţiile:a) Un paralelogram este dreptunghi dacă şi numai dacă mediatoarea a două laturi opuse este ..........................................b) Mediatoarele laturilor opuse ale unui drept-unghi sînt axele .....................................................c) Intersecţia diagonalelor unui dreptunghi este centrul cercului .....................................................

9. Construiţi dreptunghiul ABCD cu: a) AC = 7 cm, AB = 4,6 cm;b) AC = 6,8 cm, AB = 2,4 cm; c) AC = 5,4 cm, AB = 3,1 cm;d) AC = 6 cm, AB = 2,5 cm.

10. Încercaţi să împăturiţi o foaie de hîrtie, astfel încît laturile lor opuse să se suprapună. După despăturire veţi obţine axele de simetrie ale dreptunghiului.11. Construiţi dreptunghiul ABCD cu:

a) AC = 5,6 cm, mÐCAB = 15°;b) AC = 4,8 cm, mÐCAB = 32°;c) AC = 5,2 cm, mÐCAB = 23°.

12. Construiţi dreptunghiul ABCDcu AB = 4,5 cm, mÐADB = 30°.13. Demonstraţi că mij- loacele laturilor patrula- terului ortodiagonalconvex ABCD sînt vîr-furile unui dreptunghi.

14. Demonstraţi că mijloacele laturilor

A

B

C

D

MN

PQpatrulaterului orto-

diagonal concav ABCD sînt vîrfurileunui dreptunghi.

15. Construiţi dreptunghiul ABCD cu mÐADB = 15° şi distanţa d(A, BD) = 3 cm.Sugestie. Aplicaţi proprietatea triunghiului dreptung-hic cu un unghi de 15°.16. M, N, P, Q, R, S sînt mijloacele laturilor şi mi-jloacele diagonalelor patrulaterului convex ABCD. Demonstraţi că:

a) MNPQ este para- lelogram;b) MSPR şi NRQSsînt paralelograme;c) cele trei paralelo-grame au acelaşicentru.

A

B

C

D

M N

PQ

RS

A

B

C

D

M N

PQ

132 Patrulatere

¾ Proprietăţi ale rombului

½ Patrulaterul ortodiagonal

l Patrulaterul ortodiagonal are diagonalele perpendiculare.Teoremă. Dacă un patrulater are două perechi de laturi consecutive congruente, atunci el este patrulater ortodiagonal.Ipoteza. Desenul.Concluzia. AC ^ BD.

A

B

C

DE

Demonstraţie. Fie [AE] mediana corespunzătoare bazei triunghiului isoscel ABD, (1). Conform proprietăţii triunghiului isoscel, AE ^ BD, (2).(1) implică [CE] este mediana corespunzătoare bazei triunghiului isoscel BCD, (3).(3) implică CE ^ BD, (4).(2) şi (4) implică mÐAEC = 180°, (5).(5) implică AC ^ BD, c.c.t.d.În desenul din stînga este ilustrat cazul patrula-terului concav, iar în dreapta este ilustrat cazul patrulaterului convex.

A

B

C

DE

l Rombul este paralelogramul cu două laturi consecutive congruente.Teoremă. Un patrulater este romb dacă şi numai dacă are laturile congruente.NecesitateaIpoteza. Desenul. AB || DC,AD || BC, [AB] ≡ [BC].Concluzia. [AB] ≡ [BC] ≡ [CD] ≡ [AD].SuficienţaIpoteza. Desenul.AB] ≡ [BC] ≡ [CD] ≡ [AD].Concluzia. AB || DC,AD || BC, [AB] ≡ [BC].Diagonalele rombului. Teorema 1. Un paralelogram este romb dacă şi numai dacă are diagonalele per-pendiculare.NecesitateaIpoteza. Desenul. AB] ≡ [BC] ≡ [CD] ≡ [AD].Concluzia. BD ^ AC.SuficienţaIpoteza. Desenul. AB || DC,AD || BC, BD ^ AC.Concluzia. [AB] ≡ [BC].Diagonalele rombului. Teorema 2. Un paralelogram este romb dacă şi numai dacă o diagonală este şi bisectoare.Diagonalele rombului. Corolarul 1. Diagonalele rombului sînt şi bisectoare.Diagonalele rombului. Corolarul 2. Diagonalele rombului sînt axele lui de simetrie.l Înălţime a unui paralelogram este perpendiculara (segmentul) dintr-un vîrf pe una dintre laturile lui. Înălţimile paralelogramului oarecare sînt necongruente. Distanţele dintre laturile opuse ale paralelogramu-lui oarecare sînt diferite.

A

B

C

D Demonstraţie. Proprietatea laturilor paralelo-gramului implică [AB] ≡ [DC] şi [AD] ≡ [BC], (1). Ipoteza şi (1) implică [AB] ≡ [BC] ≡ [CD] ≡ [AD], c.c.t.d.

Demonstraţie. Proprietatea laturilor paralelo-gramului implică AB || DC, AD || BC, (1). (1) şi ipoteza implică AB || DC, AD || BC, [AB] ≡ [BC], c.c.t.d.

A

B

C

D Demonstraţie. Conform Teoremei patrulaterelor, Ipoteza implică BD ^ AC, c.c.t.d.

Demonstraţie. Proprietatea diagonalelor paralelo-gramului şi ipoteza implică ABC este triunghi isoscel cu baza AC, AB || DC, AD || BC, (1). (1) şi ipoteza implică [AB] ≡ [BC], c.c.t.d.

O

133Patrulatere

E x e r c i ţ i i 1. Completaţi propoziţiile:

a) Patrulaterul ortodiagonal are diagonalele ..................................................b) Dacă un patrulater are două perechi de laturi consecutive congruente, atunci el este patrulater .................................................

2. Identificaţi patrulaterele ortodiagonale:

1

2

3

3. Completaţi propoziţiile:

a) Rombul este paralelogramul cu două laturi con-secutive ..................................................................b) Un patrulater este romb dacă şi numai dacă are laturile ...................................................................

4. Construiţi rombul ABCD cu: a) BC = 4 cm, mÐA = 62°;b) AB = 4 cm, mÐA = 29°;c) AD = 4 cm, mÐB = 34°;d) BC = 4 cm, mÐC = 48°;e) CD = 4 cm, mÐD = 51°.

5. Completaţi propoziţia:Un paralelogram este romb dacă şi numai dacă are diagonalele ...................................6. Examinaţi romburile şi controlaţi dacă diagonalele lor au acelaşi mijloc şi sînt perpendiculare:

A

B

C

H

I

D

E

F

G

J

K

L

7. Construiţi rombul ABCD cu diagonalele:

a) AC = 5 cm, BD = 6 cm; b) AC = 4,4 cm, BD = 5 cm; c) AC = 4 cm, BD = 2,2 cm; d) AC = 4,6 cm, BD = 3,8.

8. Completaţi propoziţiile:a) Un paralelogram este romb dacă şi numai dacă o diagonală este şi ...............................b) Diagonalele rombului sînt şi .......................c) Diagonalele rombului sînt şi axe ......................

9. Construiţi rombul ABCD cu: a) AB = 3,8 cm, AC = 5,2 cm; b) BC = 5,3 cm, AC = 6,3 cm;c) CD = 2,8 cm, AC = 4 cm;d) AD = 5,1 cm, AC = 4,8 cm.

10. Construţi un romb prin împăturirea unei foi de hîrtie.11. Construiţi rombul ABCD cu:

a) AC = 7,2 cm, mÐA = 68°; b) AC = 6,4 cm, mÐA = 76°;c) AC = 5,3 cm, mÐA = 84°; d) AC = 4,8 cm, mÐA = 56°.

12. Construiţi rom- A

B C

D

M

N

P

Qbul ABCD cu AC = 45 cm, mÐB = 108°.3. Demonstraţi că mijloacele laturilordreptunghiului ABCDsînt vîrfurile unui

romb.14. [AD este bisectoarea unghiului A al triunghiu-lui ABC; MD || AB,PD || AC.Demonstraţi că: PM ^ AD.

15. M, N, P sînt mij-

N

MP Q

A

B

CO

loacele laturilor triun-ghiului ABC; Q estesimetricul punctuluiP faţă de M. Demon-straţi că intersecţia

segmentelor AQ şi MN este mijlocul segmentului MN.16. ABC este triunghi isoscel cu baza BC, mÐA = mÐABD = 20°,mÐACE = 30°, DE || BC. Demonstraţi că triunghiul GBF este isoscel.

AD

B

C

E

F

G

A

B CD

MP

134 Patrulatere

¿ Proprietăţile pătratului

Înălţimile rombului. Teoremă. Un paralelogram este romb dacă şi numai dacă înălţimile lui sînt egale.NecesitateaIpoteza. Desenul. AB || DC,AD || BC, [AB] ≡ [BC],AE ^ BC, AF ^ CD. Concluzia. [AE] ≡ [AF].

SuficienţaIpoteza. Desenul. AB || DC,AD || BC, AE ^ BC,AF ^ CD, [AE] ≡ [AF].Concluzia. [AB] ≡ [BC].

Proprietatea riglei negradate. Corolar. Inter-secţiile a două perechi de drepte paralele con-struite cu ajutorul riglei negradate sînt vîrfurile unui romb.

A

B

C

FE

D

Demonstraţie. AB || DC (Ipoteza) implică ÐEAB ≡ ÐFAD (ÐBAD complementul comun), (1).Conform IU, ipoteza şi (1) implică DABE ≡ DADF, de unde [AE] ≡ [AF], c.c.t.d.

Demonstraţie. AB || DC (Ipoteza) implică ÐEAB ≡ ÐFAD (ÐBAD complementul comun), (1).Conform IU, ipoteza şi (1) implică DABE ≡ DADF, de unde [AB] ≡ [AD], c.c.t.d.

A

B

C

FE

D

l Construiţi bisectoarea unui unghi cu ajutorul riglei negradate.

l Construiţi mediatoarea unui segment cu ajutorul riglei negradate.

l Pătratul este dreptunghiul cu două laturi consecutive congruente sau rombul cu un unghi drept.

Proprietăţile pătratului. Teoremă. Pătratul are proprietăţile dreptunghiului şi ale rombului: 1) unghiurile drepte şi laturile congruente; 2) diagonalele congruente şi perpendiculare; 3) dia-gonalele sînt bisectoare; 4) diagonalele şi mediatoarele laturilor opuse sînt axele de simetrie ale pătratului; 5) intersecţia diago-nalelor pătratului este centrul cercului circumscris şi centrul cer-cul înscris (tangent laturilor) în pătrat.

135Patrulatere

E x e r c i ţ i i 1. Completaţi propoziţiile:

a) Înălţime a unui paralelogram este perpendicu-lara (segmentul) dintr-un vîrf pe una dintre .............................................b) Înălţimile paralelogramului oarecare sînt ............................ c) Distanţele dintre laturile opuse ale paralelo-gramului oarecare sînt ...........................

2. Construiţi înălţimile dintr-un vîrf al fiecărui para-lelogram:

A

B C

H

I

DE

F GJ

KL

3. Completaţi propoziţia:Un paralelogram este romb dacă şi numai dacă înălţimile lui sînt .........................4. Controlaţi dacă fiecare romb are înălţimile egale:

A

B

C

H

I

D

E

F

G

J

K

L

5. Completaţi propoziţia:Intersecţiile a două perechi de drepte paralele con-struite cu ajutorul riglei negradate sînt vîrfurile unui ..............................................................

6. Construiţi bisectoarele unghiurilor cu ajutorul riglei negradate:

A B C

7. Construiţi mediatoarea segmentului cu ajutorul riglei negradate:

D

E

FA B

C

8. Completaţi propoziţiile:a) Pătratul este dreptunghiul cu două laturi con-secutive ..................... sau rombul cu un ..........b) Pătratul are proprietăţile ...................... şi ale ...................

9. Construiţi pătratul ABCD cu:a) AC = 4,6 cm; b) BD = 6,1 cm;c) AC = 3,4 cm; d) AC = 5,2 cm.

10. Construiţi pătratul ABCD cu:a) AB = 4,2 cm; b) AB = 3,2 cm; c) BC = 4,6 cm; d) AD = 5,2 cm;e) CD = 2,8 cm.

11. ABCD este pătrat, triunghiul DE

A B

C ABE este echilateral. Aflaţi mă-surile unghiurilor: AED, AEB,BEC, DEC.

12. Construiţi cu ajutorul riglei negradate perpendic-ulara într-un punct pe o dreaptă.

Ad

13. ABCD este pătrat, tri-unghiurile ABE şi BEC sînt echilaterale.

a) Calculaţi mÐAED. b) Calculaţi mÐBEF.

c) Arătaţi că punctele D, E, F (unghiul DEF este alungit).

14. Construiţi cu ajutorul riglei negradate paralela printr-un punct la o dreaptă.

DE

F

A B

C

136 Patrulatere

11 Trapezul. Elementele lui. Tipuri de trapeze

l Trapezul are două laturi paralele numite baze (baza mare şi baza mică) şi două laturi neparalele. Unghiurile alăturate fiecărei laturi neparalele sînt suplementare.

AB || DC, [AB] este baza mare, [DC] este baza mică,[BC] şi [AD] sînt laturi neparalele, [CE] este înălţimea din C.

l Trapezul cu un unghi drept este un trapez dreptunghic.MN || PQ, [MN] este baza mare, [PQ] este baza mică,[NP] şi [MQ] sînt laturi neparalele, [MQ] este înălţimea din Q.Proprietatea trapezului dreptunghic. Teoremă. Trapezul dreptunghic are două unghiuri drepte.

M N

PQ

l Construiţi trapezul dreptunghic ABCD cu baza mare dată m, unghiul drept A, înălţimea dată h baza mică dată n.

Am

h

Am

h

B

D

Am

h

B

D n

Am

h

B

D n C

12 Trapezul isoscel

l Trapezul isoscel are laturile neparalele congruente.Proprietatea unghiurilor trapezului isoscel. Teoremă. Un trapez este isoscel dacă şi numai dacă are unghiurile alăturate unei baze congruente.NecesitateaIpoteza. Desenul. AB || DC,[AD] ≡ [BC].Concluzia. ÐDAB ≡ ÐCBA.

SuficienţaIpoteza. Desenul. AB || DC,ÐDAB ≡ ÐCBA.Concluzia. [AD] ≡ [BC].

Demonstraţie. Fie DE ^ AB, CF ^ AB, (1). Ipo-teza şi (1) implică DEFC dreptunghi, de unde [DE] ≡ [CF], (2) .Conform IC, (1) şi (2) implică DADE ≡ DBCF, de undeÐDAB ≡ ÐCBA, c.c.t.d.

Demonstraţie. Fie DE ^ AB, CF ^ AB, (1). Ipo-teza şi (1) implică DEFC dreptunghi, de unde [DE] ≡ [CF], (2) .Conform CU, ipoteza, (1) şi (2) implică DADE ≡ DBCF, de unde [AD] ≡ [BC], c.c.t.d.

CD

A BFECD

A BFE

Baza mare

Baza mică

Laturi neparalele

A B

CD

Înălţime

E

137Patrulatere

E x e r c i ţ i i 1. Completaţi propoziţiile: a) Trapezul are două laturi paralele numite ............. (.......... mare şi .......... mică) şi două ...............................................

b) Unghiurile alăturate fiecărei laturi neparalele sînt ...........................

2. Enumeraţi elementele fiecărui trapez:

A

B C

H

I

DE

F GJ

KL

3. Construiţi înălţimile fiecărui trapez:

A

B C

H

I

DE

F G

J

KL

4. Completaţi propoziţiile: a) Trapezul cu un unghi drept este .........................

b) Trapezul dreptunghic are ................... unghiuri drepte.

5. Enumeraţi elementele fiecărui trapez:A

B C

H

I

D E

F G J

KL

6. Construiţi trapezul dreptunghic ABCD cu:

a) baza mare AB = 5 cm, înălţimea AD = 3 cm, CD = 2,4 cm;

b) baza mare AB = 5,3 cm, înălţimea AD = 4,1 cm, CD = 3,2 cm;c) baza mare AB = 6,1 cm, înălţimea AD = 2,8 cm, CD = 3,5 cm;d) baza mare AB = 4,8 cm, înălţimea AD = 2,5 cm, CD = 3,1 cm.

7. Construiţi trapezul dreptunghic ABCD cu:a) baza mare AB = 6,4 cm, înălţimea AD = 3,5 cm, mÐB = 76°;b) baza mare AB = 6,2 cm, înălţimea AD = 2,2 cm, mÐB = 45°;c) baza mare AB = 4,6 cm, înălţimea AD = 2,3 cm, mÐB = 47°;d) baza mare AB = 5,1 cm, înălţimea AD = 3,2 cm, mÐB = 55°.

8. Completaţi propoziţiile:a) Trapezul isoscel are laturile neparalele .........................................................b) Un trapez este isoscel dacă şi numai dacă are unghiurile alăturate unei baze ..............................

9. Construiţi trapezul isoscel ABCD cu:a) baza mare AB = 6,4 cm, AD = 2,3 cm, mÐB = 72°;b) baza mare AB = 5,3 cm, AD = 2,5 cm, mÐB = 81°;c) baza mare AB = 5,8 cm, AD = 3,4 cm, mÐA = 63°;d) baza mare AB = 6,8 cm, AD = 3,7 cm, mÐA = 69°.

10. Construiţi trapezul isoscel ABCD cu:a) baza mare AB = 5,1 cm, AD = 2,1 cm, mÐD = 96°;b) baza mare AB = 6,9 cm, AD = 2,3 cm, mÐD = 97°;c) baza mare AB = 6,4 cm, AD = 2,5 cm, mÐD = 98°;d) baza mare AB = 7,4 cm, AD = 3,8 cm, mÐD = 111°.

14. Construiţi trapezul isoscel ABCD cu baza mare AB = 6,8 cm, CD = 3,4 cm, mÐA = 60°. Calculaţi perimetrul trapezului.15. Construiţi trapezul isoscel ABCD cu baza mică

CD = 4,2 cm, AD = 3,5 cm, mÐA = 60°. Calculaţi perimetrul trapezului.16. Aflaţi perimetrul trapezului isoscel ABCD cu baza mare AB = 8,6 cm, mÐA = 60°, mÐACD = 90°.

11. Construiţi trapezul dreptunghic ABCD u baza mare AB = 6,4 cm, CD = 3,2 cm, mÐB = 30°.12. Construiţi trapezul dreptunghic ABCD cu baza

mare AB = 5,1 cm, CD = 2,6 cm, mÐD = 60°.13. Construiţi trapezul isoscel ABCD cu baza mare AB = 5,5 cm, AD = 3,2 cm, înălţimea MD = 2,5 cm.

138 Patrulatere

Proprietatea diagonalelor trapezului isoscel. Teoremă. Un trapez este isoscel dacă şi numai dacă are diagonalele congruente.NecesitateaIpoteza. Desenul. AB || DC,[AD] ≡ [BC].Concluzia. [AC] ≡ [BD].

SuficienţaIpoteza. Desenul. AB || DC,[AC] ≡ [BD].Concluzia. [BC] ≡ [AD].

Axa de simetrie a trapezului isoscel. Teoremă. Un trapez este isoscel dacă şi numai dacă mediatoarea unei baze coincide cu mediatoarea celeilalte baze.NecesitateaIpoteza. Desenul. AB || DC,[AD] ≡ [BC], d mediatoarea[AB].Concluzia. [DF] ≡ [FC].

SuficienţaIpoteza. Desenul. AB || DC,d mediatoarea [AB] şi [CD].Concluzia. [BC] ≡ [AD].

Axa de simetrie a trapezului isoscel. Corolar. Un trapez este isoscel dacă şi numai dacă are o axă de simetrie.

CD

A BCD

A BFE

Demonstraţie. Ipoteza implică ÐDAB ≡ ÐCBA, (1). Conform LUL, (1), ipoteza ([AD] ≡ [BC]) şi [AB] latură comună implică DABD ≡ DBAC, de unde [AC] ≡ [BD], c.c.t.d.

Demonstraţie. Fie DE ^ AB, CF ^ AB, (1). Ipo-teza şi (1) implică DEFC dreptunghi, de unde [DE] ≡ [CF], (2) .Conform IC, ipoteza, (1) şi (2) implică DAFC ≡ DBED, de unde ÐCAF ≡ ÐDBE, (3).Conform LUL, ipoteza şi (3) implică DCAB ≡ DDBA, de unde [BC] ≡ [AD], c.c.t.d.

l Construiţi trapezul isoscel ABCD cu baza mare dată m unghiul A de măsură dată u, lungimea uneia dintre laturile neparalele n.

A A AB

DD D C

B B

C

uu umm m

nnn n

l Construiţi trapezul isoscel ABCD cu baza mare AB de lungime m, lungimea uneia dintre diagonale de lun-gime d, lungimea bazei mici p. Rezolvare. Se analizează trapezul cu proprietăţile din enunţ.

CD

A B

F

E

M

d

Demonstraţie. Fie M intersecţia AD şi BC, (1). Ipoteza (AB || DC, [AD] ≡ [BC]) implică ÐDAB ≡ ÐCBA, de unde ABM este triunghi isoscel cu baza AB, (2). Ipoteza (d mediatoarea [AB]) implică d = ME, (3). Ipoteza (AB || DC) şi (2) implică MDC este triunghi isoscel cu baza DC, (4). (3) şi (4) implică d este mediatoarea [DC], [DF] ≡ [FC], deci c.c.t.d.

Demonstraţie. Fie DN ^ AB, CP ^ AB, (1). Ipoteza (AB || DC) şi (1) implică DNPC dreptunghi, de unde [NE] ≡ [EP] şi [DN] ≡ [CP], (2). Ipoteza (d media-toarea [AB] şi [CD]) şi (2) implică [AN] ≡ [PB], (3).Conform CC, (1), (2) şi (3) implică DAND ≡ DBPC, de unde [AD] ≡ [BC], c.c.t.d.

CD

A B

F

E

d

PN

139Patrulatere

E x e r c i ţ i i 1. Completaţi propoziţiaUn trapez este isoscel dacă şi numai dacă are diago-nalele ................................................................ 2. Identificaţi trapezele isoscele:

A

B C

H

I

DE

F G

J

KL

3. Comparaţi unghiurile alăturate fiecărei baze:

A

B C

H

I

D E

F GJ

K

L

4. Comparaţi lungimile diagonalelor fiecărui trapez de mai sus.5. Construiţi înălţimile din fiecare vîrf al bazei mici:

A

B C

H

I

D

E

F G J

KL

6. Construiţi trapezul isoscel ABCD cu: a) baza mare AB = 5,1 cm, AC = 4,5 cm,

BC = 3 cm;b) baza mare AB = 5,6 cm, AC = 6,1 cm, BC = 4,8 cm; c) baza mare AB = 7,4 cm, AC = 6,3 cm, BC = 2,5 cm;d) baza mare AB = 6,5 cm, AC = 4,5 cm, BC = 3,1 cm.

7. Construiţi trapezul isoscel ABCD cu:a) baza mare AB = 5,2 cm, AC = 4,1 cm, mÐA = 15°;b) baza mare AB = 5,8 cm, AC = 4,2 cm, mÐA = 25°;c) baza mare AB = 6,5 cm, AC = 5,3 cm, mÐA = 28°;d) baza mare AB = 6,9 cm, AC = 5,6 cm,mÐA = 30°.

8. Construiţi trapezul isoscel ABCD cu:a) baza mare AB = 5,2 cm, mÐA = 16°,mÐB = 75°;b) baza mare AB = 5,6 cm, mÐA = 18°, mÐB = 72°;c) baza mare AB = 6,1 cm, mÐA = 23°,mÐB = 68°.

9. Construiţi trapezul isoscel ABCD cu: a) baza mică DC = 3,3 cm, AD = 4,2 cm, mÐB = 123°;b) baza mică DC = 4,4 cm, AD = 4,4 cm, mÐB = 112°;c) baza mică DC = 1,9 cm, AD = 5,7 cm, mÐB = 104°.

10. Construiţi trapezul isoscel ABCD cu:a) baza mică CD = 2,5 cm, AC = 6,4 cm, AD = 4,6 cm;b) baza mică CD = 1,7 cm, AC = 5,6 cm, AD = 4,6 cm;c) baza mică CD = 2,3 cm, AC = 6,8 cm, AD = 4,9 cm.

11. Construiţi trapezul isoscel ABCD cu baza mare AB = 6,5 cm, BC = 3,5 cm, mÐC = 105°.12. Construiţi trapezul isoscel ABCD cu baza mare

AB = 6,5 cm, înălţimea DM = 4,2 cm, mÐA = 66°.13. Construiţi trapezul isoscel ABCD cu baza mare AB = 5,8 cm, CD = 2,6 cm, înălţimea MD = 3,1 cm.

14. Construiţi trapezul isoscel ABCD cu baza mare AB = 6,5 cm, CD = 3,9 cm, AC = 6,2 cm.15. Construiţi trapezul isoscel ABCD cu baza mare AB = 7,4 cm, unghiul ACB este drept, mÐA = 67°.

16. Construiţi trapezul isoscel ABCD cu diagonalele perpendiculare, baza mare AB = 6,4 cm, CD = 4 cm.17. Construiţi trapezul isoscel ABCD cu AD ^ BC, baza mare AB = 5,8 cm, înălţimea MD = 1,9 cm.

140 Patrulatere

CD

A Bm

p CD

A Bm

d

p

d d d

p

d

E

Se construieşte paralelo-gramul BECD. Rezultă [AC] ≡ [CE].

Executarea construcţiei. Se construieşte: triunghiul isos-

cel AEC cu baza de lungime AE = AB + BE = m + p şi

AC = d; paralelogramul BECD; se completează

ABCD.

13 Linia mijlocie a trapezului

l Bimediană a unui patrulater este un segment determinat de mijloacele a două laturi opuse. Orice patru-later (convex sau concav) are două bimediane.Proprietatea bimedianei. Teoremă. Bimedianele unui patrulater au acelaşi mijloc.Bimedianele paralelogramului. Teoremă. Bimedianele paralelogramului sînt paralele cu laturile para-lelogramului şi se intersectează în centrul lui de simetrie.Bimedianele dreptunghiului şi pătratului. Teoremă. Bimedianele dreptunghiului şi pătratului sînt conţinute de mediatoarele laturilor opuse.l Linie mijlocie a unui trapez este bimediana determinată de mijloacele laturilor neparalele.Proprietatea liniei mijlocii. Teorema. Linia mijlocie a trapezului este paralelă cu bazele trapezului şi lungimea ei este egală cu semisuma lungimilor bazelor.Ipoteza. Desenul, AB || DC, M mijlocul [AD], N mijlocul [BC].Concluzia. MN || AB, MN = 0,5(AB + CD).

A B

CD

M NM P Q

Proprietatea liniei mijlocii. Corolar. 1) Linia mijlocie a trapezului conţine mijloacele diagonalelor. 2) Lungimea segmentului determinat de mijloacele diagonalelor este egală cu semidiferenţa lungimilor bazelor trapezului.

Demonstraţie. Fie MP || DC, (1). Proprietatea liniei mijlocii a triunghiului implică [AP] ≡ [PC], (2). PQ || AB, implică [QB] ≡ [QD], (3). Conform proprietăţii liniei mijlocii a triunghiului, ipoteza şi (3) implică NQ || CD, (4). Conform Postulatului Paralelelor, AB || DC, (1), (2), (3), (4) implică MN || AB, (5).Conform proprietăţii liniei mijlocii a triunghiului, (1) implică MP = 0,5 • DC, (6). În acelaşi mod rezultă PN = 0,5 • AB, (7). (6) şi (7) implică MN = 0,5(AB + CD), c.c.t.d.

14 Unghiuri cu laturile respectiv paralele sau perpendiculare Unghiuri cu laturile respectiv paralele. Teoremă. Unghiurile cu laturile respectiv paralele sînt congru-ente sau suplementare.Ipoteza. AB || DC,AD || BC, M–A–D.Concluzia. ÐBAD ≡ ÐBCD, mÐMAB + mÐBCD = 180°.Unghiuri cu laturile respectiv perpendiculare. Teoremă. Unghiurile cu laturile respectiv perpendiculare sînt congruente sau suplementare.Ipoteza. Desenul. AC ^ BD, AE ^ BC.Concluzia. ÐCAE ≡ ÐCBD, mÐMAE + mÐCBD = 180°.

A

B

D

C

M Demonstraţie. AB || DC, AD || BC, (Ipoteză) implică ABCD este paralelogram, de unde ÐBAD ≡ ÐBCD (unghiuri opuse), (1). (1) implică mÐMAB + mÐBCD = 180°, c.c.t.d.

A

B

DCM

E

Demonstraţie. ÐACE ≡ ÐBCD (unghiuri opuse la vîrf), (1). (1) şi ipoteza implică ÐCAE ≡ ÐCBD, (2). (2) implică mÐMAE + mÐCBD = 180°, c.c.t.d.

141Patrulatere

E x e r c i ţ i i 1. Completaţi propoziţiile:

a) Bimediană a unui patrulater este un segment determinat de mijloacele a două .........................b) Orice patrulater (convex sau concav) are ..........bimediane.

2. Construiţi bimedianele:

A

B C

H

I

D E

F

G J

KL

3. Completaţi propoziţia:Bimedianele unui patrulater au acelaşi ......................4. Controlaţi dacă bimedianele au acelaşi mijloc:

A

B C

H

I

D E

F G J

K

L

5. Completaţi propoziţiile:

a) Bimedianele paralelogramului sînt ......................... cu laturile paralelogramului şi se intersectează în ............................ de simetrie.b) Bimedianele dreptunghiului şi pătratului sînt conţinute de ...........................................................c) Linie mijlocie a unui trapez este bimediana determinată de mijloacele .....................................

6. Controlaţi proprietăţile bimedianelor paralelogra-melor:

A

B C

H

I

DE

F GJ

K

L

7. Controlaţi proprietăţile bimedianelor dreptunghi-ului şi pătratului:

A

B C

H

I

DE

F G

J

KL

8. Completaţi propoziţiile:a) Linia mijlocie a trapezului este paralelă cu ba-zele trapezului şi lungimea ei este egală cu ..............................................................................................b) Linia mijlocie a trapezului conţine mijloacele .............................................................................c) Lungimea segmentului determinat de mijloacele diagonalelor este egală cu ................................................................................. bazelor trapezului.

9. Trapezul ABCD are linia mijlocie MN, P şi Q mijlo-acele diagonalelor. Aflaţi

A B

C

M

D

NP Q

lungimea liniei mijlocii şi PQ, dacă:

a) DC = 3,5 cm, AB = 5,7 cm;b) DC = 2,7 cm, AB = 4,3 cm;c) DC = 1,1 cm, AB = 3,7 cm;d) DC = 4,1 cm, AB = 5,9 cm.

12. Fie triunghiul dreptunghic ABC cu ipotenuza BC şi înălţimea AD. Enumeraţi unghiurile cu laturile respectiv perpendiculare.13. Construiţi trapezul dreptunghic ABCD cu baza

mare AB = 8,6 cm astfel încît triunghiul ACB este dreptunghic isoscel cu ipotenuza AB. Aflaţi lungimea liniei mijlocii şi distanţa dintre mijloacele diago-nalelor trapezului ABCD.

10. Fie paralelogramul ABCD. Enu-

A B

CDmeraţi unghiurile cu laturile respectivparalele.

11. Construiţi trapezul isoscel ABCD cu lungimea li-niei mijlocii MN = 6,5 cm, distanţa dintre mijloacele diagonalelor PQ = 2,5 cm şi înălţimea de 4,2 cm.

142 Patrulatere

E VA L U A R EI I I

1. Patrulaterul convex ABCD

A B

CDare măsurile unghiurilor A, B,

C respectiv egale cu 47°, 86°,59°. Aflaţi măsura unghiului D.2. Paralelogramul DEFG are DE = 5 cm şi EF = 5,7 cm. Aflaţi lungimile celorlalte laturi ale paralelo-gramului.3. Paralelogramul DEFG are unghiul D de 41°. Aflaţi măsurile celorlalte unghiuri ale paralelogramului.4. Paralelogramul DEFG are centrul de simetrie O. Aflaţi lungimile segmentelor OD şi OE, dacă DF = 4,8 cm şi GE = 3,8 cm.

D E

G FO

5. Trapezul dreptunghic

J K

M LJKLM are unghiul K de 24°.Aflaţi măsurile celorlalteunghiuri ale trapezului.6. Trapezul MNPQ are bazele MN de 5,4 cm şi PQ de 3,6 cm. Aflaţi: a) lungimea liniei mijlocii a trapezului;b) distanţa dintre mijloacele diagonalelor lui.7. Trapezul isoscel MNPQ

M N

Q P

are NP = 3,2 cm, MP = 4,1 cmşi unghiul P de 115°. Aflaţi:a) MQ şi NQ; b) măsurile celorlalte unghiuri.8. Construiţi: a) paralelogramul cu diagonalele de 6 cm, 4,8 cm şi un unghi format de ele de 34°;b) rombul cu diagonalele de 4,6 cm şi 5,2 cm.9. Construiţi trapezul isoscel ABCD cu baza mare AB de 3,8 cm, unghiul A de 45° şi CD = 2,1 cm.10. Construiţi trapezul isoscel ABCD cu baza mare AB de 5,6 cm, unghiul B de 60° şi diagonalele perpen-diculare. Aflaţi perimetrul trapezului.

1. Patrulaterul convex ABCD

A B

CDare măsurile unghiurilor A, B,

C respectiv egale cu 42°, 93°,71°. Aflaţi măsura unghiului D.2. Paralelogramul DEFG are DE = 4,8 cm şi EF = 6 cm. Aflaţi lungimile celorlalte laturi ale paralelo-gramului.3. Paralelogramul DEFG are unghiul D de 52°. Aflaţi măsurile celorlalte unghiuri ale paralelogramului.4. Paralelogramul DEFG are centrul de simetrie O. Aflaţi lungimile segmentelor OD şi OE, dacă DF = 5,2 cm şi GE = 2,4 cm.

D E

G FO

5. Trapezul dreptunghic

J K

M LJKLM are unghiul K de 37°.Aflaţi măsurile celorlalteunghiuri ale trapezului.6. Trapezul MNPQ are bazele MN de 8,8 cm şi PQ de 6,2 cm. Aflaţi: a) lungimea liniei mijlocii a trapezului;b) distanţa dintre mijloacele diagonalelor lui.7. Trapezul isoscel MNPQ

M N

Q P

are NP = 2,6 cm, MP = 4,9 cmşi unghiul P de 123°. Aflaţi:a) MQ şi NQ; b) măsurile celorlalte unghiuri.8. Construiţi: a) paralelogramul cu diagonalele de 4 cm, 5,8 cm şi un unghi format de ele de 23°;b) rombul cu diagonalele de 5,8 cm şi 4,4 cm.9. Construiţi trapezul isoscel ABCD cu baza mare AB de 4,5 cm, unghiul A de 45° şi CD = 2,6 cm.10. Construiţi trapezul isoscel ABCD cu baza mare AB de 6,4 cm, unghiul B de 60° şi diagonalele perpen-diculare. Aflaţi perimetrul trapezului.

1

1

1

1

1

1

1

1

1

1

143Geometrie. Rezolvări. Indicaţii 143143143143

Geometrie. Rezolvări. Indicaţii. Răspunsuri

Cap.1. 15. (P. 75) Se controlează dacă suma a două distanţe este egală cu a treia distanţă. 16. 10 drepte.Cap.2. 11. (P. 77) 10 segmente; 20 semidrepte. 15. 1 + 2 + 3 + 4 + 5 etc. 16. 3, 3 + 9, 3 + 9 + 9 etc. 12. (P. 79) 6 unghiuri. 13. 1 + 2 + 3 + ... + 7 etc. 12. (P. 81) De exemplu: Dacă un unghi are 60°, atunci bisectoarea formează cu laturile lui unghiuri de 30°.26. (P. 84) a) 36°, 54°. b) 70°, 20°. 27. a) 54°, 126°.Cap.3. 15. (P. 87) a) Dreaptă secantă. b) Exterioară. 9. (P. 89) a) Cercuri exterioare. b) Cercuri exterioare. Cap.4. 43. (P. 95) a) 1, 1 + 2 = 3, 1 + 2 + 3 = 6, ..., 1 + 2 + 3 + ... + 15 + 16 = 16 • 17 : 2 = 136. 45. 3x + 7 = 5x + 3, de unde x = 2. Perimetrul este 6 cm. 18. (P. 98) Triunghiul are laturile de 4,5 cm. 23. Examinaţi ex. 44, p. 95.Cap.5. 24. (P. 103) Conform LLL, ABC ≡ DCB, de unde [AB] ≡ [DC], ABC ≡ BCD. 25. Se de-monstrează, aplicînd LLL, că BDC ≡ BCD. 26. Se demonstrează, aplicînd ULU, că BDC ≡ BCD. 21. (P. 106) Se construieşte: 1) unghiul drept A; 2) latura AB; 3) unghiul B. 22. Se aplică proprietatea medi-anei AD: mediana din vîrful triunghiului isoscel este perpendiculară pe baza triunghiului. Se poate construi o infi nitate de triunghiuri congruente. 23. Unghiuri-le alăturate bazei au 37°. 27. Se aplică LUL. 28. Se aplică ULU. 29. Se aplică CC. 30. Se aplică CC.Cap.6. 9. (P. 109) b) BAC < C1; c) ABC < A1; d) ABC < C1; e) ACB < A1; f) ACB < B1.14. (P. 113) Deoarece m || n, 5x + 19° = 7x + 3° (mă-surile a două unghiuri corespondente), de unde x = 8°.15. Deoarece 57° + 123° = 180°, m || n (măsurile a două unghiuri alterne externe), de unde 3x + 13° + 36x + 11° etc. Răspuns: x = 4°. 17. Măsura ACB = 180° – 117° + 180° – 129° etc. 20. Se construieşte paralela prin P cu dreapta m şi se obţine o situaţie similară cu cea din ex. 17. 21. Se aplică ex. 20. 21. Se aplică ex. 18 după ce se construieşte paralela prin B cu m.17. (P. 115) Există două triunghiuri isoscele necongru-ente cu proprietăţile din enunţ. 11. (P. 117) Perpen-dicularele din O sînt mediatoarele triunghiului. 11. Bisectoarele unghiurilor triunghiului.29. (P. 121) a) Se aplică proprietatea centrului de greutate al triunghiului şi se deduce că [BG] ≡ [CG], de unde EBC ≡ DCB. 35. Ipotenuza are 94 mm şi cateta opusă unghiului de 30° are 47 mm. 28. (P. 124) Cazurile: 1) baza este de 28 mm; 2) una dintre laturile congruente are 28 mm. 29. Două soluţii.Cap.7. 14. (P. 127) 130°. 16. (P. 131) a) Teorema li-niei mijlocii a unui triunghi implică: [MN] ≡ [PQ] şi MN || PQ etc. c) Paralelogramele au fi ecare două o

diagonală comună. 16. (P. 133) Afl aţi măsurile un-ghiurilor triunghiului CBG. 11. (P. 137) Se construieşte: 1) triunghiul dreptung-hic ABM cu ipotenuza BM, AB = 6,4 cm, mB = 30°; 2) N între A şi B astfel încît AN = CD = 3,2 cm; 3) perpendiculara în N pe AB; 4) paralela cu AB prin intersecţia perpendicularei cu BM. Paralela cu AB conţine baza mică CD. 12. Se procedează ca la ex. 11. 13. Se construieşte: 1) [AB]; 2) paralela cu AB la distanţa 2,5 cm de AB; 3) intersecţia D a paralelei cu un arc de cerc de centru A şi rază 3,2 cm; 4) intersecţia C a paralelei cu un arc de cerc de centru B şi rază 3,2 cm. 5) Se construiesc celelalte laturi ale trapezului ABCD. 14. Se construieşte: 1) triunghiul echilateral ABM; 2) punctul N între A şi B astfel încît AN = 3,4 cm; 3) paralela cu AM prin punctul N; 4) intersecţia C a paralelei cu BM; 5) paralela cu AB prin C; 6) intersecţia D a paralelei cu AM. 5) Se constru-iesc celelalte laturi ale trapezului ABCD. 11. (P. 139) mA = 75°. Se construieşte: 1) baza AB; 2) se construieşte unghiul A; 3) se construieşte unghiul B; 5) se construiesc laturile neparalele; 6) se completează trapezul. 12. Se construieşte: 1) baza AB; 2) paralela cu AB la distanţa 4,2 cm; 3) unghiul A etc. 13. Se construieşte: 1) baza AB; 2) paralela cu AB la distanţa 3,1 cm; 3) mediatoarea bazei mari; 4) baza mică CD etc. 14. Se construieşte: 1) baza AB; 2) punctul M al semidreptei AB, astfel încît [BM] ≡ [CD]; 3) triunghiul isoscel ACM; 4) baza mică CD etc. 15. Se construieşte: 1) triunghiul dreptunghic ACB cu ipotenuza AB; 2) triunghiul BAD congruent cu triunghiul ACB etc. 16. Se construieşte: 1) triun-ghiul dreptunghic isoscel cu ipotenuza AB; 2) pre-lungirea fi ecărei catete; 3) segmentul MN de 4 cm cu M şi N puncte ale laturilor catetelor şi MN || AB; 4) perpendiculara prin M pe AB; 5) perpendiculara prin N pe AB; 6) trapezul ABCD (C şi D sînt intersecţiile prelungirile catetelor cu perpendicularele pe AB). 12. (P. 141) Două perechi de unghiuri cu laturile res-pectiv perpendiculare. 13. Înălţimea trapezului este 4,3 cm. 14. Se construieşte: 1) segmentul AE, B–A–E, AE = 6,5 cm + 3,9 cm; 2) triunghiul isoscel ACE cu baza AE; 3) paralelogramul BECD etc.